0% found this document useful (0 votes)
68 views111 pages

Electric Field and Charges Module

The document discusses the concepts of electric charges and fields, focusing on frictional electricity, electrostatics, and the nature of electric charges. It explains the differences between conductors and insulators, the process of electrostatic induction, and the basic properties of electric charge. Key experiments demonstrate the principles of charge attraction and repulsion, as well as the methods for charging objects through contact and induction.

Uploaded by

ak02100singla
Copyright
© © All Rights Reserved
We take content rights seriously. If you suspect this is your content, claim it here.
Available Formats
Download as PDF, TXT or read online on Scribd
0% found this document useful (0 votes)
68 views111 pages

Electric Field and Charges Module

The document discusses the concepts of electric charges and fields, focusing on frictional electricity, electrostatics, and the nature of electric charges. It explains the differences between conductors and insulators, the process of electrostatic induction, and the basic properties of electric charge. Key experiments demonstrate the principles of charge attraction and repulsion, as well as the methods for charging objects through contact and induction.

Uploaded by

ak02100singla
Copyright
© © All Rights Reserved
We take content rights seriously. If you suspect this is your content, claim it here.
Available Formats
Download as PDF, TXT or read online on Scribd
You are on page 1/ 111

MITTAL INSTITUTE

CHAPTER 1 ELECTRIC CHARGES AND FIELD

1.1 FRICTIONAL ELECTRICITY Large-scale matter that consists of


equal number of electrons and protons is
1. What is frictional electricity ? When
electrically neutral. If there is an excess of
is a body said to be electrified or charged
electrons, the body has a negative charge
?
and an excess of protons results in a
Frictional electricity. If a glass rod is positive charge.
rubbed with a silk cloth, or a fountain-pen
1.3 ELECTROSTATICS
with a coat-sleeve, it is able to attract
small pieces of paper, straw, lint, light 4. What is electrostatics ? Mention
feathers, etc. Similarly, a plastic comb some of its important applications.
passed through dry hair can attract such
Electrostatics. Electrostatics is the
light objects. In all these examples, we
study of electric charges at rest. Here we
can say that the rubbed substance has
study the forces, fields and potentials
become electrified or electrically charged.
associated with static charges.
It is because of friction that the
substances get charged on rubbing. 1.4 TWO KINDS OF ELECTRIC
CHARGES
The property of rubbed substances
due to which they attract light objects is 5. How will you show experimentally
called electricity. The electricity that (i) there are only two kinds of electric
developed by rubbing or friction is called charges and (ii) like charges repel and
frictional or static electricity. The unlike charges attract each other ?
rubbed substances which show this Two kinds of electric charges. About
property of attraction are said to have 100 years ago, Charles Du Fay of France
become electrified or electrically
showed that electric charges on various
charged.
objects are of only two kinds. The
. following simple experiments prove this
fact.
1.2 ELECTRIC CHARGE
EXPERIMENT 1
3. What is electric charge ? Is it a
scalar or vector quantity ? Name its SI (i) Rub a glass rod with silk and suspend it
unit. from a rigid support by means of a silk
thread. Bring another similarly charged
Electric charge. Electric charge is an
rod near it. The two rods repel each other
intrinsic property of the elementary
[Fig. 1.1(a)].
particles like electrons, protons, etc., of
which all the objects are made up of. It is
because of these electric charges that
various objects exert strong electric forces
of attraction or repulsion on each other.
Electric charge is an intrinsic property
of elementary particles of matter which
gives rise to electric force between various
objects.
Electric charge is a scalar quantity. Its
SI unit is coulomb (C). A proton has a
positive charge (+ e) and an electron has
a negative charge (- e), where
e = .6 × 10-19 coulomb
Fig. 1.1 Like charges repel and unlike

CENTER- DSS 100, SECTOR 9 MARKET, NEAR AGGARWAL BHAVAN , AMBALA CITY
CONTACT NO. - 9896441144 Page 1
MITTAL INSTITUTE

charges attract each other. demonstrate that the charges are


transferred from the rods to the pith balls
(ii) Bring a plastic rod rubbed with wool
on contact. We say that the pith balls have
near the charged glass rod. The two rods
been electrified or charged by contact.
attract each other [Fig. 1.1(b)].
This property which distinguishes the two
(iii) Now rub a plastic rod with wool and kinds of charges is called the polarity of
suspend it from a rigid support. Bring charge.
another similarly charged plastic rod near
7. What are positive and negative
it. There will be a repulsion between the
charges ? What is the nature of charge on
two rods [Fig. 1.1(c)].
an electron in this convention ?
EXPERIMENT 2. If a glass rod, rubbed
Positive and negative charges.
with silk, is made to touch two small pith
Benjamin Franklin (1706-1790), an
balls (or polystyrene balls) which are
American pioneer of electrostatics
suspended by silk threads, then the two
introduced the present-day convention by
balls repel each other, as shown in Fig.
replacing the terms vitreous and resinous
1.2(a). Similarly, two pith balls touched
by positive and negative, respectively.
with a plastic rod rubbed with fur are found
According to this convention :
to repel each other [Fig. 1.2(b)]. But it is
seen that a pith ball touched with glass 1. The charge developed on a glass rod
rod attracts another pith ball touched with when rubbed with silk is called positive
a plastic rod [Fig. 1.2(c)]. charge.
2. The charge developed on a plastic rod
when rubbed with wool is called negative
charge.
The above convention is consistent
with the fact that when two opposite kinds
of charges are brought in contact, they
tend to cancel each other's effect.
According to this convention, the charge
on an electron is negative.
Table 1.1 gives a list of the pairs of
objects which get charged on rubbing
against each other. On rubbing, an object
of column I will acquire positive charge
Fig. 1.2 Like charges repel and unlike
while that of column II will acquire
charges attract.
negative charge.
From the above experiments, we note
Table 1.1 Two kinds of charges developed
that the charge produced on a glass rod is
on rubbing
different from the charge produced on a
plastic rod. Also the charge produced on a
Column I (Positive Column II (Negative
pith ball touched with a glass rod is
charge) charge)
different from the charge produced on pith
ball touched with a plastic rod. We can
Glass rod Silk cloth
conclude that :
1. There are only two kinds of electric Flannel or cat skin Ebonite rod
charges - positive and negative.
Woollen cloth Amber rod
2. Like charges repel and unlike charges
attract each other. Woollen coat Plastic seat
The statement 2 is known as the
fundamental law of electrostatics. Woollen carpet Rubber shoes
The above experiments also Obviously, any two charged objects

2
MITTAL INSTITUTE

belonging to the same column will repel The material with lower work function
each other while those of two different loses electrons and becomes positively
columns will attract each other. charged.
. As an electron has a finite mass,
therefore, there always occurs some
1.5 ELECTRONIC THEORY OF
change in mass during charging. The
FRICTIONAL ELECTRICITY
mass of a positively charged body slightly
8. Describe the electronic theory of decreases due to loss of some electrons.
fictional electricity. Are the frictional forces The mass of a negatively charged body
electric in origin ? slightly increases due to gain in some
Electronic theory of frictional electrons.
electricity. All matter is made of atoms. An 1.6 CONDUCTORS AND INSULATORS
atom consists of a small central nucleus
9. How do the conductors differ from
containing protons and neutrons, around
the insulators ? Why cannot we electrify a
which revolve a number of electrons. In
metal rod by rubbing it while holding it in
any piece of matter, the positive proton
our hand ? Hozv can we charge it ?
charges and the negative electron
charges cancel each other and so the Conductors. The substances through
matter in bulk is electrically neutral. which electric charges can flow easily are
called conductors. They contain a large
The electrons of the outer shell of an
number of free electrons which make
atom are loosely bound to the nucleus.
them good conductor of electricity. Metals,
The energy required to remove an
human and animal bodies, graphite, acids,
electron from the surface of a material is
alkalies, etc. are conductors.
called its 'work function'. When two
different bodies are rubbed against each Insulators. The substances through
other, electrons are transferred from the which electric charges cannot flow easily
material with lower work function to the are called insulators. In the atoms of such
material with higher work function. For substances, electrons of the outer shell
example, when a glass rod is rubbed with are tightly bound to the nucleus. Due to
a silk cloth, some electrons are transferred the absence of free charge carriers, these
from glass rod to silk. The glass rod substances offer high resistance to the
develops a positive charge due to flow of electricity through them. Most of
deficiency of electrons while the silk cloth the non- metals like glass, diamond,
develops an equal negative charge due to porcelain, plastic, nylon, wood, mica, etc.
excess of electrons. The combined total are insulators.
charge of the glass rod and silk cloth is
An important difference between
still zero, as it was before rubbing i.e.,
conductors and insulators is that when
electric charge is conserved during
some charge is transferred to a conductor,
rubbing.
it readily gets distributed over its entire
Electric origin of frictional forces. The surface. On the other hand, if some
only way by which an electron can be charge is put on an insulator, it stays at
pulled away from an atom is to exert a the same place. We shall discuss this
strong electric force on it. As electrons are distinguishing feature in the next chapter.
actually transferred from one body to
A metal rod held in hand and rubbed
another during rubbing, so frictional forces
with wool does not develop any charge.
must have an electric origin.
This is because the human body is a good
For Your Knowledge conductor of electricity, so any charge
developed on the metal rod is transferred
The cause of charging is the actual
to the earth through the human body. We
transfer of electrons from one material to
can electrify the rod by providing it a
another during rubbing. Protons are not
plastic or a rubber handle and rubbing it
transferred during rubbing.
without touching its metal part.

3
MITTAL INSTITUTE

10. What is meant by earthing or


grounding in household circuits ? What is
its importance ?
Earthing and safety. When a charged
body is brought in contact with the earth
(through a connecting conductor), its
entire charge passes to the ground in the
form of a momentary current. This process
in which a body shares its charges with
the earth is called grounding or earthing.
Fig. 1.4 Electrostatic induction.
Electrostatic induction is the
phenomenon of temporary electrification
of a conductor in which opposite charges
appear at its closer end and similar
charges appear at its farther end in the
presence of a nearby charged body.
Fig. 1.3 (a) Positively charge (b) The positive and negative charges
Negatively charge, earthed body. produced at the ends of the conducting
rod are called induced charges and the
The electricity from the mains is charge on the glass rod which induces
supplied to our houses using a three-core these charges on conducting rod is called
wiring : live, neutral and earth wires. The inducing charge.
live wire red in colour brings in the current.
The black neutral wire is the return wire. 12. Describe how two metal spheres
The green earth wire is connected to a can be oppositely charged by induction.
thick metal plate buried deep into the Charging of two spheres by induction.
earth. The metallic bodies of the electric Figure 1.5 shows the various steps
appliances such as electric iron, involved in inducing opposite charges on
refrigerator, TV, etc. are connected to the two metal spheres.
earth wire. When any fault occurs or live
wire touches the metallic body, the charge
flows to the earth and the person who
happens to touch the body of the
appliance does not receive any shock.
1.7 ELECTROSTATIC INDUCTION
11. What is meant by electrostatic
induction ?
Electrostatic induction. As shown in
Fig. 1.4, hold a conducting rod AB over an
insulating stand. Bring a positively
charged glass rod near its end A. The free
electrons of the conducting rod get
attracted towards the end A while the end
B becomes electron deficient. The closer Fig. 1.5 Two metal spheres get oppositely
end A acquires a negative charge while charged by induction.
the remote end B acquires an equal
positive charge. As soon as the glass rod (a) Hold the two metal spheres on
is taken away, the charges at the ends A insulating stands and place them in
and B disappear. contact, as shown in Fig. 1.5(a).
(b) Bring a positively charged glass
rod near the left sphere. The free

4
MITTAL INSTITUTE

electrons of the spheres get attracted the ground, its positive charge at the near
towards the glass rod. The left surface of end remains held there due to the
the left sphere develops an excess of attractive force of the external charge.
negative charge while the right side of the
(d) When the plastic rod is removed, the
right sphere develops an excess of
positive charge spreads uniformly on the
positive charge. However, all of the
sphere.
electrons of the spheres do not collect at
the left face. As the negative charge
begins to build up at the left face, it starts
repelling the new incoming electrons.
Soon an equilibrium is established under
the action of force of attraction of the rod
and the force of repulsion due to the
accumulated electrons. The equilibrium
situation is shown in Fig. 1.5(b).
(c) Holding the glass rod near the left
sphere, separate the two spheres by a
small distance, as shown in Fig. 1.5(c). Fig. 1.6 Charging by induction.
The two spheres now have opposite
charges. Similarly, the metal sphere can be
negatively charged by bringing a positively
(d) Remove the glass rod. The charged glass rod near it.
charges on the spheres get redistributed.
Their positive and negative charges face For Your Knowledge
each other, as shown in Fig. 1.5(d). The Gold-leaf electroscope. It is a device used
two spheres attract each other. for detecting an electric charge and
(e) When the two spheres are identifying its polarity. It consists of a
separated quite apart, the charges on vertical conducting rod passing through a
them get uniformly distributed, as shown rubber stopper fitted in the mouth of a
in Fig. 1.5(c). glass vessel. Two thin gold leaves are
attached to lower end of the rod. When a
Thus the two metal spheres get charged object touches the metal knob at
charged by a process called charging by the outer end of the rod, the charge flows
induction. In contrast to the process of down to the leaves. The leaves diverge
charging by contact, here the glass rod due to repulsion of the like charges they
does not lose any of its charge. have received. The degree of divergence
13. How can you charge a metal of the leaves gives a measure of the
sphere positively without touching it ? amount of charge.
Charging of a sphere by induction. Fig.
1.6 shows the various steps involved in
inducing a positive charge on a metal
sphere.
(a) Hold the metal sphere on an insulating
stand. Bring a negatively charged plastic
rod near it. The free electrons of the
sphere are repelled to the farther end. The
near end becomes positively charged due 1.8 BASIC PROPERTIES OF ELECTRIC
to deficit of electrons. CHARGE
(b) When the far end of the sphere is It is observed from experiments that
connected to the ground by a connecting electric charge has following three basic
wire, its free electrons flow to the ground. properties :
(c) When the sphere is disconnected from 1. Additivity 2. Quantization 3.

5
MITTAL INSTITUTE

Conservation. Quantization of electric charge. It is


found experimentally that the electric
We shall discuss these properties in
charge of any body, large or small, is
detail in the next few sections.
always an integral multiple of a certain
1.9 ADDITIVITY OF ELECTRIC CHARGE minimum amount of charge. This basic
14. What do you mean by additive charge is the charge on an electron, which
nature of electric charges ? is denoted by e and has magnitude 1.6 ×
10-19 coulomb. Thus the charge on an
Additive nature of electric charges. electron is - e, on a proton is + e and that
Like mass, electric charge is a scalar on α-particle is + 2e. The experimental
quantity. Just as the mass of an extended fact that electric charges occur in discrete
body is the sum of the masses of its amounts instead of continuous amounts is
individual particles, the total charge of an called quantization of electric charge. The
extended body is the algebraic sum (i.e., quantization of electric charge means that
the sum taking into account the positive the total charge (q) of a body is always an
and negative signs) of all the charges integral multiple of a basic quantum of
located at different points inside it. Thus, charge (e), i.e.,
the electric charge is additive in nature.
q = ne, where n = 0, ±1, ±2, ±3,
Additivity of electric charge means that
the total charge of a system is the Cause of quantization. The basic
algebraic sum of all the individual charges cause of quantization of electric charge is
located at different points inside the that during rubbing only an integral
system. number of electrons can be transferred
from one body to another.
If a system contains charges q1, q2,
….., qn, then its total charge is Quantization of electric charge is an
experimentally verified law :
q = q1+ q2 + ….. + qn
1. The experimental laws of electrolysis
The total charge of a system discovered by Faraday first suggested the
containing four charges 2 μC, - 3 μC, 4 μC quantization of electric charge.
and - 5 μC is
2. Millikan's oil drop experiment in 1912 on
q = 2μC - 3μC + 4μC - 5μC = -2 μC the measurement of electric charge further
1.10 QUANTIZATION OF ELECTRIC established the quantization of electric
CHARGE charge.

15. What is meant by quantization of a 17. Can we ignore the quantization of


physical quantity ? electric charge ? If yes, under what
conditions ?
Quantization of a physical quantity.
The quantization of a physical quantity When can we ignore the quantization
means that it cannot vary continuously to of electric charge. While dealing with
have any arbitrary value but it can change macroscopic charges (q = ne),
discontinuously to take any one of only a we can ignore the quantization of electric
discrete set of values. For example, a charge. This is because the basic charge
building can have different floors (ground, e is very small and n is very large in most
first, second, etc.) from the ground floor practical situations, so q behaves as if it
upwards but it cannot have a floor of the were continuous i.e., as if a large amount
value in-between. Thus the energy of an of charge were flowing. For example,
electron in atom or the electric charge of a when we switch on a 60 W bulb, nearly 2
system is quantized. The minimum × 1018 electrons pass through its filament
amount by which a physical quantity can per second. Here the graininess or
change is called its quantum. structure of charge does not show up i.e.,
16. What is meant by quantization of the bulb does not flicker with the entry of
electric charge ? What is the cause of each electron. Quantization of charge
quantization of electric charge ? becomes important at the microscopic

6
MITTAL INSTITUTE

level, where the charges involved are of ∴ Number of electronic charges in 1


the order of a few tens or hundreds of e. coulomb,
For Your Knowledge n= = = 6.25 × 1018.
The smallest amount of charge or basic
quantum of charge is the charge on an Example 2. A comb drawn through
electron or a proton. Its exact magnitude person's hair on a dry day causes 1022
is e = 1.602192 × 10-19 C electrons to leave the person's hair and
stick to the comb. Calculate the charge
Quantization of electric charge cannot be carried by the comb.
explained on the basis of classical
electrodynamics or even modem physics. Solution. Here n = 1022, e = 1.6 × 10-19 C
However, the physical and chemical ∴ q = ne = 1022 × 1.6 × 10-19 = 1.6 × 103 C
properties of atoms, molecules and bulk
matter cannot be explained without As the comb has excess of electrons,
considering the quantization of electric ∴ Charge on comb = - 1.6 × 103 C.
charge.
Example 3. If a body gives out 109
Recent discoveries in high energy physics electrons every second, how much time is
have indicated that the elementary required to get a total charge of 1 C from it
particles like protons and neutrons are ? [NCERT]
themselves built out of more elementary
units, called quarks, which have charges Solution. Number of electrons given
(2/3) e and (- 1/3) e Even if quark-model is out by the body in one second = 109
established in future, the quantization of Charge given out by the body in one second
charge will still hold. Only the quantum of
charge will reduce from e to e/3. = ne = 109 × 1.6 × 10-19 C

Quantization is a universal law of nature. = 1.6 × 10-10 C


Like charge, energy and angular Time required to get a charge of 1.6 × 10-10 C
momentum of an electron are also
quantized. However, quantization of mass =1s
is yet to be established. Time required to get a charge of 1 C
Examples based on = s = 6.25 × 109 s
Quantisation of Electric Charge
= years = 198.18 years.
Formulae Used
1. q = ne Thus from a body emitting 109
electrons per second, it will take nearly
2. Mass transferred during charging = me 200 years to get a charge of 1 C from that
×n body. This shows how large is one
Units Used coulomb as the unit of charge.

q and e are in coulomb, n is pure integer. Example 4. How much positive and
negative charge is there in a cup of water
Constants Used ? [NCERT]
e = 1.6 × 10-19 C, me = 9.1 × 10-31 kg Solution. Suppose the mass of water
Example 1. Which is bigger -a coulomb or contained in a cup is 250 g. The molecular
a charge on an electron ? How many mass of water is 18 g.
electronic charges form one coulomb of Number of molecules present in 18 g of water
charge ? [Haryana 01] = Avogadro's number = 6.02 × 1023
Solution. One coulomb of charge is ∴ Number of molecules present in a
bigger than the charge on an electron. cup (or 250 g) of water
Charge on one electron, e = 1.6 × 10-
19
C n= = 8.36 × 1024

7
MITTAL INSTITUTE

Each molecule of water (H2O) contains ∴ q = + 26 e


2+8=10 electrons as well as 10 protons. Tot ss
5. ss o n tron
Total number of electrons or protons
present in a cup of water, q = -ne = - × 1031 × 1.6 × 10-19 = -1.33 ×
25
n' = n × 10 = 8.36 × 10 1013 C.
Total negative charge carried by 6. Number of molecules in 2.0 mole of H2
electrons or total positive charge carried gas = 2.0 × 6.02 × 1023
by protons in a cup of water,
As each H2 molecule contains 2
q = n’ e electrons/protons, so
= 8.36 × 1025 × 1.6 × 10-19 C = 1.33 × 107 n = 2 × 2.0 × 6.02 × 1023 = 24.08 × 1023
C
∴ q = ne = 24.08 × 1023 × 1.6 × 10-19
Problems For Practice
= 0.3853 × 106 C = 0.3853 MC.
1. Calculate the charge carried by 12.5 ×
108 electrons. [CBSE D 92] [1 MC = 106C]
(Ans. 2 × 10-10 C) 7. Proceed as in Example 4.
2. How many electrons would have to be 1.11 CONSERVATION OF CHARGE
removed from a copper penny to leave 18. State the laze of conservation of
it with a positive charge of 10-7 C ? charge. Give some examples to illustrate
(Ans. 6.25 × 1011 electrons) this law.
3. Calculate the charge on an alpha Law of conservation of charge. If some
particle. Given charge on a proton = amount of matter is isolated in a certain
1.6 × 10-19 C. region of space and no matter either
(Ans. + 3.2 × 10-19 C) enters or leaves this region by moving
across its boundary, then whatever other
4. Calculate the charge on nucleus. changes may occur in the matter inside,
Given charge on a proton = 1.6 × 10-19 its total charge will not change with time.
C. (Ans. + 4.16 × 10-18 C) This is the law of conservation of charge
5. Determine the total charge on 75.0 kg which states :
of electrons. 1. The total charge of an isolated system
13
(Ans. - 1.33 × 10 C) remains constant.

6. How many mega coulombs of positive 2. The electric charges can neither be
(or negative) charge are present in 2.0 created nor destroyed, they can only be
mole of neutral hydrogen (H2) gas ? transferred from one body to another.

7. Estimate the total number of electrons The law of conservation of charge is


present in 100 g of water. How much obeyed both in large scale and
is the total negative charge carried by microscopic processes. In fact, charge
these electrons ? Avogadro's number conservation is a global phenomenon i.e.,
= 6.02 × 1023 and molecular mass of total charge of the entire universe remains
water = 18. (Ans. 5.35 × 106 C) constant.
HINTS Examples :

3. An alpha particle contains 2 protons 1. When a glass rod is rubbed with a silk
and 2 neutrons. cloth, it develops a positive charge. But at
the same time, the silk cloth develops an
∴ q = + 2e. equal negative charge. Thus the net
4. nucleus contains 26 protons and charge of the glass rod and the silk cloth
30 neutrons. is zero, as it was before rubbing.
2. The rocksalt ionises in aqueous solution

8
MITTAL INSTITUTE

as follows : electric charge and mass


NaCl Na+ + Cl-
Electric charge Mass
As the total charge is zero before and
after the ionisation, so charge is 1. Electric charge Mass of a body is
conserved. may be positive, always positive.
negative or zero.
3. Charge is conserved during the fission
of a nucleus by a neutron. 2. Electric charge is Quantization of
always quantized : mass is not yet
+ → + + 3 +
q = ne established.
Energy
Total charge before fission (0 + 92) 3. Charge on a body Mass of a body
does not depend increases with its
= Total charge after fission (56 + 36 + 3 ×
on its speed. speed.
0)
4. Electric charge is conserved during the 4. Charge is strictly Mass is not
phenomenon of pair production in which a conserved. conserved by itself
y-ray photon materialises into an electron- as some of the
positron pair. mass may get
changed into
γ – ray → electron + positron energy or vice
zero charge (-e) (+e) versa.

5. In annihilation of matter, an electron 5. Electrostatic forces Gravitational forces


and a positron on coming in contact between two between two
d stroy h oth r, produ ing two γ-ray charges may be masses are always
photons, each of energy 0.51 MeV. attractive or attractive.
repulsive.
electron + positron → 2 γ – rays
6. Electrostatic forces Gravitational forces
(-e) (+e) zero charge between different between different
charges may bodies never
For Your Knowledge cancel out. cancel out.
Conservation of charge implies that
electric charges can be created or 7. A charged body A body possessing
destroyed always in the form of equal and always possesses mass may not
opposite pairs but never in isolation. For some mass. have any net
example, in the beta decay of a neutron charge.
(zero charge), a proton (charge + e) and
an electron (charge^- e) are produced. 20. How does the speed of an
Total charge remains zero before and electrically charged particle affect its (i)
after the decay. mass and (ii) charge ?
The law of conservation of charge is an Effect of speed on mass and electric
exact law of nature. It is valid in all charge. According to the special theory of
domains of nature. Even in the domains of relativity, the mass of a body increases
high energy physics, where mass changes with its speed in accordance with the
into energy and vice-versa, the law of relation :
conservation of charge strictly holds good.
1.12 ELECTRIC CHARGE VS MASS √
19. Compare the properties of electric
charge with those of mass of a body. where, m0 = rest mass of the body, c =
speed of light, and m = mass of the body
Table 1.2 Comparison of the properties of

9
MITTAL INSTITUTE

when moving with speed v.


As v < c, therefore, m > m0. wh r ε0 is called permittivity of free
space. So we can express Coulomb's law
In contrast to mass, the charge on a
in SI units as
body remains constant and does not
change as the speed of the body changes. F=
1.13 COULOMB'S LAW OF ELECTRIC
FORCE Units of charge, (i) The SI unit of
charge is coulomb. In the above equation,
21. State Coulomb's law in if q1 = q2- 1C and r = 1 m, then
electrostatics. Express the same in SI
units. Name and define the units of F= =9 × 109 N
electric charge.
So one coulomb is that amount of
Coulomb's law. In 1785, the French charge that repels an equal and similar
physicist Charles Augustin Coulomb charge with a force of 9 × 109 N when
(1736-1806) experimentally measured the placed in vacuum at a distance of one
electric forces between small charged metre from it.
spheres by using a torsion balance. He
formulated his observations in the form of (ii) In electrostatic cgs system, the unit
Coulomb's law which is electrical of charge is known as electrostatic unit of
analogue of Newton's law of Universal charge (e.s.u. of charge) or statcoulomb
Gravitation in mechanics. (stat C).

Coulomb's law states that the force of One e.s.u. of charge or one
attraction or repulsion between two statcoulomb is that charge which repels
stationary point charges is (i) directly an identical charge in vacuum at a
proportional to the product of the distance of one centimetre from it with a
magnitudes of the two charges and (ii) force of 1 dyne.
inversely proportional to the square of the 1 coulomb = 3 × 109 statcoulomb
distance between them. This force acts
along the line joining the two charges. = 3 × 109 e.s.u. of charge
(iii) In electromagnetic cgs system, the
unit of charge is abcoulomb or
electromagnetic unit of charge (e.m.u.
Fig. 1.7 Coulomb's law. of charge).
If two point charges q1 and q2 are
1 coulomb = abcoulomb = e.m.u. of
separated by distance r, then the force F
of attraction or repulsion between them is charge
such that For Your Knowledge
F ∝ q,q2 and F∝ When the linear sizes of charged bodies
are much smaller than the distance
∴ ∝ or between them, their sizes may be ignored
and the charged bodies are called point
where A: is a constant of proportionality,
charges.
called electrostatic force constant. The
value of k depends on the nature of the Coulomb's law is valid only for point
medium between the two charges and the charges.
system of units chosen to measure F2 q1, In SI units, the exact value of the
q2 and r. combination 4π ε0 is
For the two charges located in free
space and in SI units, we have 4πε0 = C2N-1m-2

k= = 9 × 109 Nm2 C-2 where c is the speed of light in vacuum


having the exact value 299792458 × 108

10
MITTAL INSTITUTE

ms-1. where ̂ =

, is a unit vector in the
Electrostatic force constant, direction from q2 to q1.
k = 8.98755 × 109 Nm2C-2 ≃ 9 × 109 The coulombian forces between unlike
Nm2C-2. charges (q1q2 <0) are attractive, as shown
Permittivity of free space, in Fig. 1.9.

ε0 = 8.8551485 × 10-2 C2N-1m-2


≃ 9 × 10-2C2N-1 m-2.
SI unit of permittivity
Fig. 1.9 Attractive coulombian forces for
= = C2N-1m-2 q1q2 < 0.
The unit C2N-1m-2 is usually expressed as Importance of vector form. The vector
farad per metre (Fm-1). form of coulomb's law gives the following
More strictly, the SI unit of charge 1 additional information :
coulomb is equal to 1 ampere-second, 1. As ̂ = - ̂ , therefore ⃗ 21 = - ⃗ .
where 1 ampere is defined in terms of the
magnetic force between two current This means that the two charges exert
carrying wires. equal and opposite forces on each other.
So Coulombian forces obey Newton's third
1.14 COULOMB'S LAW IN VECTOR law of motion.
FORM
2. As the Coulombian forces act along ⃗ 12
22. Write Coulomb's law in vector
form. What is the importance of or ⃗ 21 i.e., along the line joining the
expressing it in vector form ? centres of two charges, so they are central
forces.
Coulomb's law in vector form. As
shown in Fig. 1.8, consider two positive 23. What is the range over which
point charges q1 and q2 placed in vacuum Coulombian forces can act ? State the
at distance r from each other. They repel limitations of Coulomb's law in
each other. electrostatics.
Range of coulombian forces.
Coulombian forces act over an enormous
range of separations (r), from nuclear
dimensions (r = 10-15 m) to macroscopic
distances as large as 1018 m. Inverse
Fig. 1.8 Repulsive coulombian forces for square is valid over this range of
q1 q2 >0. separation to a high degree of accuracy.
In vector form, Coulomb's law may be Limitations of Coulomb's law.
expressed as Coulomb's law is not applicable in all
situations. It is valid only under the
⃗ 21 = Force on charge q2 due to q1
following conditions :
= . ̂ 1. The electric charges must be at rest.
⃗ 2. The electric charges must be point
where ̂ = , is a unit vector in the charges i.e., the extension of charges
direction from q1 to q2- must be much smaller than the separation
between the charges.
Similarly, ⃗ 12 = Force on charge q1 due
to q2 3. The separation between the charges
must be greater than the nuclear size (10-
= . ̂ 15
m), because for distances < 10-15m, the
strong nuclear force dominates over the

11
MITTAL INSTITUTE

electrostatic force. follows :


1.15 DIELECTRIC CONSTANT : The dielectric constant or relative
permittivity of a medium may be defined
RELATIVE PERMITTIVITY
as the ratio of the force between two
24. What do you mean by permittivity charges placed some distance apart in
of a medium ? Define dielectric constant in free space to the force between the same
terms of forces between two charges. two charges when they are placed the
Permittivity : An introduction. When same distance apart in the given medium.
two charges are placed in any medium Clearly, when a material medium of
other than air, the force between them is dielectric constant K is placed between the
greatly affected. Permittivity is a property charges, the force between them
of the medium which determines the becomes 1 / K times the original force in
electric force between two charges vacuum. That is,
situated in that medium. For example, the
v
force between two charges located some d
distance apart in water is about l/80th of
the force between them when they are Hence the Coulomb's law for any material
separated by same distance in air. This is medium may be written as
because the absolute permittivity of water
is about 80 times greater than the d
absolute permittivity of air or free space.
Dielectric constant or relative K (vacuum) = 1
permittivity. According to Coulomb's law, K (air) = 1.00054
the force between two point charges q1
K (water) = 80.
and q2, placed in vacuum at distance r
from each other, is given by Examples based on

v ...(1) Coulomb's Law


Formulae Used
When the same two charges are
placed same distance apart in any 1. v
medium other than vacuum, the force
between them becomes 2. d F - * ^2
d …(2) Units Used
Th qu ntity ε is d bso ut q1,q2 are in coulomb, F in newton and r in
permittivity or just permittivity of the metre.
intervening medium. Dividing equation (1)
Constant Used
by equation (2), we get
k= = 9 × 109 Nm2C-2
v
Example 5. The electrostatic force of
d
repulsion between two positively charged
ions carrying equal charges is 3.7 × 10-9
Th r tio (ε / ε0) o th p r ittivity (ε) o
N, when they are separated by a distance
the m diu to th p r ittivity (ε0) of free of 5 Å. How many electrons are missing
sp is d r tiv p r ittivity (εr) or from each ion ?
dielectric constant (K) of the given
medium. Thus Solution. Here F = 3.7 × 10-9 N,
v r = 5 Å = 5 × 10-10 m, q1 = q2, =q (say)
or
d As
So one can define dielectric constant in
terms of forces between charges as

12
MITTAL INSTITUTE

∴ √ .

or Solution. Suppose charge - q1 moves


around the charge + q2 with speed v along
or q = 3.2 × 10-19 C the circular path of radius r. Then
Number of electrons missing from each Force of attraction between the two
ion is charges
= Centripetal force

Example 6. A free pith-ball A of 8 g carries or or √


a positive charge of 5 × 10-8 C. What must
be the nature and magnitude of charge The period of revolution of charge - q1
that should be given to a second pith-ball around + q2 will be
B fixed 5 cm below the former ball so that
the upper ball is stationary ? [Haryana 01] √ or
Solution. The pith-ball B must be of
√ .
positive charge i.e., of same nature as that
of A so that the upward force of repulsion
Example 8. Two particles, each having a
balances the weight of pith-ball A
mass of 5 g and charge 1.0 × 10-7 C, stay
When the pith-ball A remains stationary, in limiting equilibrium on a horizontal table
with a separation of 10 cm between them.
F = m1g
The coefficient of friction between each
or p rti nd th t b is th s . Find μ.
Solution. Here q1 = q2 =1.0 × 10-7 C,
But m1 = 8 g = 8 × 10-3 kg
r = 10 cm = 0.10 m, m = 5 g = 5 × 10-3 kg
q1 = 5 × 10-8 C
The mutual electrostatic force between the
r = 5 cm = 0.05 m
two particles is

= 0.009 N

The limiting force of friction between a


particle and the table is
= μ × mg = μ × 5 × 10-3 × 9.8 = 0.049 μ N
As the two forces balance each other,
therefore
Fig. 1.10 0.049 μ = 0.009

∴ or μ = = 0.18.
Example 9. (a) Two insulated charged
or copper spheres A and B have their
= 4.36 × 10-7 C (positive). centres separated by a distance of 50 cm.
What is the mutual force of electrostatic
Example 7. A particle of mass m and repidsion if the charge on each is 6.5 × 10-
7
carrying charge - q1 is moving around a C ? The radii of A and B are negligible
charge + q2 along a circular path of radius compared to the distance of separation.
r. Prove that the period of revolution of the Also compare this force with their mutual
charge - q1 about + q2 is given by gravitational attraction if each weighs 0.5
kg.

13
MITTAL INSTITUTE

(b) What is the force of repulsion if (i) each q = 6.5 × 10-7C


sphere is charged double the above
When a similar but uncharged sphere C is
amount, and the distance between them is
placed in contact with sphere A, each
halved ; (ii) the two spheres are placed in
sphere shares a charge q / 2, equally.
water ? (Dielectric constant of water = 80).
[NCERT]
Solution, (a) Here q1 = q2 = 6.5 × 10-7 C,
r = 50 cm = 0.50 m
Using Coulomb's law,

ir

Fig. 1.11
= 9 × 109. N
Now when the sphere C (with charge q/2)
-2
= 1.5 × 10 N. is placed in contact with sphere B (with
charge q), the charge is redistributed
The mutual gravitational attraction,
equally, so that

Charge on sphere B or C = ( )

∴ New force of repulsion between A and


B is
N
Clearly, Fc << Fair.
(b) (i) When charge on each sphere is
doubled, and the distance between them = × 1.5 × 10-2 N = 0.5625 × 10-2 N 8
is halved, the force of repulsion becomes
≃ 5.7 × 10-3 N.
ir Example 11. Two similarly equally
charged identical metal spheres A and B
= 16 × 1.5 × 10-2 = 0.24 N. repel each other with a force of 2.0 × 10-5
(ii) The force between two charges placed N. A third identical uncharged sphere C is
in a medium of dielectric constant K is touched to A, then placed at the midpoint
given by between A and B. Calculate the net
electrostatic force on C. [CBSE OD 03]
Solution. Let the charge on each of the
spheres A and B be q. If the separation
For water, K = 80 between A and B is r, then electrostatic
force between spheres A and B will be

= 1.875 × 10-4 N ≃ 1.9 × 10-4 N. N
Example 10. Suppose the spheres A and When sphere C is touched to A, the
B in Example 9 have identical sizes. A spheres share charge q /2 each, because
third sphere of the same size but both are identical.
uncharged is brought in contact with the
first, then brought in contact with the ∴ Force on C due to A
second, and finally removed from both.
What is the new force of repulsion ong
between A and B ? [NCERT]
Force on C due to B
Solution. Charge on each of the spheres A
and B is

14
MITTAL INSTITUTE

Solution. Suppose the three charges are


ong placed as shown in Fig. 1.13. Let the
charge q be positive.
Since these forces act in opposite
directions, therefore net force on C is 2 a2
a2 a2

Fig. 1.13
N ong
For the equilibrium of charge + q, we must
Example 12. Two identical charges, Q
have Force of repulsion F1 between + 4e
each, are kept at a distance r from each
and + q
other. A third charge q is placed on the
line joining the above two charges such = Force of repulsion F2 between + e and +
that all the three charges are in q
equilibrium. What is the magnitude, sign
and position of the charge q ? [CBSE OD or
94, 98]
or 4 (a - x)2 = x2
Solution. Suppose the three charges be
or 2 (a - x) = ± x
placed in the manner, as shown in Fig.
1.12. ∴x= or 2 a
As the charge q is placed between + 4c
and + c, so only x = 2a/3 is possible.
Hence for equilibrium, the charge q must
be placed at a distance 2a/3 from the
Fig. 1.12 charge + 4c.
The' charge q will be in equilibrium if the We have considered the charge q to be
forces exerted on it by the charges at A positive. If we displace it slightly towards
and C are equal and opposite. charge c, from the equilibrium position,
then F1 will decrease and F2 will increase
or and a net force (F2 - F1) will act on q
towards left i.e., towards the equilibrium
or x = r - x or x = position. Hence the equilibrium of positive
q is stable.
Since the charge at A is repelled by the
similar charge at C, so it will be in Now if we take charge q to be negative,
equilibrium if it is attracted by the charge q the forces F1 and F2 will be attractive, as
at B, i.e., the sign of charge q should be shown in Fig. 1.14.
opposite to that of charge Q.
∴ Force of repulsion between charges at
A and C
= Force of attraction between charges at A
Fig. 1.14
and B
The charge -q will still be in equilibrium at
or or . x = 2a/3. However, if we displace charge -
q slightly towards right, then F1 will
Example 1 3. Two point charges + 4c and decrease and F2 will increase. A net force
+ e are fixed' a distance 'a' apart. Where (F2 - F1) will act on - q towards right i.e.,
should a third point charge q be placed on away from the equilibrium position. So the
the line joining the two charges so that it equilibrium of the negative q will be
may be in equilibrium ? In which case the unstable.
equilibrium will be stable and in which
unstable ? Example 14. Two 'free' point charges + 4e

15
MITTAL INSTITUTE

and + e are placed a distance 'a' apart.


Where should a third point charge q be
placed between them such that the entire
system may be in equilibrium ? What Fig. 1.16
should be the magnitude and sign of q ? For the equilibrium of charge q, the charge
What type of a equilibrium will it be ? Q must have the same sign as that of q or
Solution. Suppose the charges are placed 4q, so that the forces FA and FB are equal
as shown in Fig. 1.15. and opposite.
As FA = FB

or Q = 4 q.
Fig. 1.15 Example 16. A charge Q is to be divided
As the charge + e exerts repulsion F on on two objects. What should be the values
charge + 4e, so for the equilibrium of of the charges on the two objects so that
charge + 4e, the charge -q must exert the force between the objects can be
attraction F' on +4e. This requires the maximum ?
charge q to be negative. Solution. Let q and Q - q be the charges
For equilibrium of charge +4e, on the two objects. Then force between
the two objects is
F = F'

where r is the distance between the two


or objects.
For F to be maximum,
For equilibrium of charge - q,
Attraction F1 between +4e and - q
= Attraction f2 between + e and – q
or

or
or x2 = 4 (a – x)2
∴ x = 2a/3 or Q - 2q =0

Hence q = or q =

The equilibrium of the negative charge q i.e., the charge should be divided equally
will be unstable. on the two objects.
Example 15. Two point charges of charge Example 17. Two identical spheres,
values Q and q are placed at distances x having charges of opposite sign attract
and x / 2 respectively from a third charge each other with a force of 0.108 N when
of charge value 4q, all charges being in separated by 0.5 m. The spheres are
the same straight line. Calculate the connected by a conducting wire, which
magnitude and nature of charge Q, such then removed, and thereafter they repel
that the net force experienced by the each other with a force of 0.036 N. Wlwt
charge q is zero. were the initial charges on the spheres ?
[CBSE D 98] Solution. Let + q1 and - q2 be the initial
charges on the two spheres.
Solution. Suppose the three charges are
placed as shown in Fig. 1.16. (a) When the two spheres attract each
other,

16
MITTAL INSTITUTE


(b) When the two spheres are connected
by the wire, they share the charges
equally.
∴ Charge on each sphere =

Force of repulsion between them is


Fig. 1.17
( )( ) As the forces are in equilibrium, the three
forces on sphere A can be represented by
th thr sid s o Δ AOC t k n in th
i.e. same order. Hence


or q1 - q2 = 2 × 10-6 ...(i) or F = mg × ...(ii)
2 2
Now (q1 + q2) = (q1 - q2) + 4 q1q2
From (i) and (ii), we have
= (2 × 10-6)2 + 4 × 3 × 10-12
= 16 × 10-12
∴ q1 + q2 = 4 × 10-6 ...(ii) But AC = l sin θ, OC = os θ, AB = 2 AC
On solving equations (i) and (ii), we get = 2l sin θ

q1 = 3 × 10-6 C and q2 = 10-6 C ∴


which are the initial charges on the two
or q2 = (4 mg I2 sin2 θ t n θ) 4πε0.
spheres.
Problems For Practice
Example 18. Two small spheres each
having mass m kg and charge q coulomb 1. Obtain the dimension or u o ε0.
are suspended from a point by insulating -1 -3 4
(Ans. M L T A ) 2
threads each l metre long but of negligible
ss. I θ is th ng , h thr d k s 2. Calculate coulomb force between two
with the vertical when equilibrium has α-particles separated by a distance of 3.2
been attained, show that × 10-15 m in air.
q2 =(4 mgl2 sin2 θ t n θ) 4πε0 [Punjab 95] [CBSE OD 92] (Ans. 90 N)
Solution. The given situation is shown in 3. Calculate the distance between two
Fig. 1.17. Each of the spheres A and B is protons such that the electrical repulsive
acted upon by the following forces : force between them is equal to the weight
of either. [CBSE D 94]
(i) its weight mg, (ii) tension T in the string
(Ans. 1.18 cm)
(iii) the force of repulsion F given by
4. How far apart should the two electrons
…(i) be, if the force each exerts on the other is
equal to the weight of the electron ? Given
that e = 1.6 × 10-19 C and me = 9.1 × 10-31
kg. [Haryana 02]
(Ans. 5.08 m)

17
MITTAL INSTITUTE

5. A pith-ball A of mass 9 × 10-5 kg carries 11. Two point charges q1 = 5 × 10-6C and
h rg o 5 μC. Wh t ust b th q2 = 3 × 10-6C are located at positions (1
magnitude and sign of the charge on a m, 3 m, 2 m) and (3 m, 5 m, 1 m)
pith-ball B held 2 cm directly above the respectively. Find the forces and
pith-ball A, such that the pith-ball A using vector form of Coulomb's law.
remains stationary ?
[Ans. ̂ ̂ ̂ ,
(Ans. 7.84 pC, sign opposite to that of A)
̂ ̂ ̂ ]
6. Two identical metal spheres having
equal and similar charges repel each 12. Three equally charged small objects
other with a force of 103 N when they are are placed as shown in Fig. 1.18. The
placed 10 cm apart in a medium of object A exerts an electric force on object
dielectric constant 5. Determine the B equal to 3.0 × 10-6N.
charge on each sphere. (Ans. 23.9 × 10-6
C)
7. The distance between the electron and
proton in hydrogen atom is 5.3 × 10-11 m. Fig. 1.18
Determine the magnitude of the ratio of
electrostatic and gravitational force (i) What electric force does C exert on B ?
between them. (ii) What is the net electric force on B ?
-31 -27
Given me = 9.1 × 10 kg, = 1.67 × 10 [Ans. (i) 12.0 × 10-6N, along BA (ii) 9.0 ×
kg, e = 1.6 × 10-19 C and G = 6.67 × 10-11 10-6 N, along BA]
Nm2 kg-2.
13. Two identical metallic spheres A and
(Ans. Fe / FG = 2.27 × 1039) B, each carrying a charge q, repel each
8. Two identical metallic spheres, having other with a force F. A third metallic
unequal, opposite charges are placed at a sphere C of the same size, but uncharged,
distance 0.90 m apart in air. After bringing is successively made to touch the spheres
them in contact with each other, they are A and B, and then removed away. What is
again placed at the same distance apart. the force of repulsion between A and B ?
Now the force of repulsion between them (Ans. 3F/ 8)
is 0.025 N. Calculate the final charge on 14. Two point charges + 9e and + e are
each of them. [CBSE D 02C] kept at a distance a from each-other.
(Ans. 1.5 × 10-6C) Where should we place a third charge q
on the line joining the two charges so that
9. A small brass sphere having a positive it may be in equilibrium ?
charge of 1.7 × 10-8 C is made to touch
another sphere of the same radius having (Ans. from + 9e charge)
a negative charge of 3.0 × 10-9 C. Find the
force between them when they are 15. Two point electric charges of values q
separated by a distance of 20 cm. What and 2q are kept at a distance d apart
will be the force between them when they from each other in air. A third charge Q is
are immersed in an oil of dielectric to be kept along the same line in such a
constant 3 ? way that the net force acting on q and 2q
is zero. Calculate the position of charge
(Ans. 1.1 × 10-5 N ; 0.367 × 10-5 N) Qin terms of q and d. [CBSE D 98]
10. Th su o two point h rg s is 7 μC. (Ans. At a distance of ( √ ) d from
They repel each other with a force of 1 N charge q)
when kept 30 cm apart in free space.
Calculate the value of each charge. 16. A charge q is placed at the centre of
[CBSE F 09] the line joining two equal charges Q.
Show that the system of three charges will
(Ans. 5 μC, 2 μC) be in equilibrium if q = - Q/ 4.
[CBSE OD 05]

18
MITTAL INSTITUTE

17. Two pith-balls each weighing 10-3kg


are suspended from the same point by
or
means of silk threads 0.5 m long. On
charging the balls equally, they are found But m1 = 9 × 10-5 kg,
to repel each other to a distance of 0.2 m.
Calculate the charge on each ball. q1 - 5 μC = 5 × 10-6 C,
[Haryana 2002] r = 2 cm = 0.02 m
(Ans. 2.357 × 10-6C) ∴
HINTS
or q2 = 7.84 × 10-12 C = 7.84 pC.
1. or
6. ∴

or C.
2. Here 7. Proceed as in illustrative problem on
page 1.18.
∴ 8. The two spheres will share the final
charge equally. Let q be the charge on
N. each sphere.
3. For a proton, kg ∴
C.
or
Weight of Proton Electrical repulsive
force or
or
or C.
∴ 9. Charge shared by each sphere

or cm.
4.
N.
or
10. Here F = 1N, r = 30 m
As
5. The pith-ball B must have charge
opposite to that of A so that the ∴
upward force of attraction balances the
or
weight of pith-ball A.
When the pith-ball A remains stationary, But …(i)

Now

or – …(ii)
On solving (i) and (ii), we get
and
Fig. 1.19

19
MITTAL INSTITUTE

11. Here ⃗⃗⃗ ̂ ̂ ̂ m, ⃗⃗⃗ ̂ ̂


̂ )m

∴ ⃗⃗⃗⃗⃗ ⃗⃗⃗ ⃗⃗⃗ ̂ ̂ ̂ ̂ ̂ ̂


Fig. 1.21
̂ ̂ ̂
For equilibrium of charge q,
⃗⃗⃗⃗⃗ √
…(i)
̂ ̂ ̂ ̂
̂ For equilibrium of charge 2q,
⃗⃗⃗⃗⃗
⃗⃗⃗⃗⃗⃗ = …(ii)
̂

̂ ̂ ̂ From (i) and (ii), we get,

̂ ̂ ̂ .
or =
Also, ⃗⃗⃗⃗⃗ = ⃗⃗⃗⃗⃗ = ̂ ̂ ̂ .
or √
12. Here AB = 2 cm = 0.02 m, BC = 1 cm
= 0.01 m or √

i.e., the charge Q must be placed at a


distance of (√ - 1) d from the charge q.
16. Suppose the three charges are placed
Fig. 1.20 as shown in Fig. 1.22.
Let q be the charge on each object.

Fig. 1.22
or
Clearly, the net force on charge q is zero.
or So it is in equilibrium, the net force on
other two charges should also be zero.
(i) Total force on charge Q at point B is

, along BA.
(ii) Net force on charge at B,
or

or q = Q/4.
, along BA.
17. In Δ OCA of forces, we have
13. Proceed as in Example 10 on page
1.12.
14. Force between + 9e and q = Force
between + e and q

or or
15. For equilibrium of charges q and 2q,
the charge Q must have sign opposite to
that of q or 2q. Suppose it is placed at
distance x from charge q.

20
MITTAL INSTITUTE

Fig. 1.23 square dependence on the distance


between charges/masses.
∴ F=
(a) Compare the strength of these forces
by determining the ratio of their
magnitude (i) for an electron and a
proton and (ii) for two protons.
or √
(b) Estimate the accelerations for electron
q = 2.357 × 10-6 C. and proton due to the electrical force
of their mutual attraction when they
1.16 COMPARING ELECTROSTATIC
are 1 •Å (= 10-10 m) apart.
AND GRAVITATIONAL FORCES
How much is the electrostatic force
25. Give a comparison of the
stronger than the gravitational force ?
electrostatic and gravitational forces.
(a)(i) From Coulomb's law, the
Electrostatic force vs gravitational
electrostatic force between an electron
force. Electrostatic force is the force of
and a proton separated by distance r is
attraction or repulsion between two
charges at rest while the gravitational
force is the force of attraction between two
bodies by virtue of their masses.
Negative sign indicates that the force
Similarities : is attractive. From Newton's law of
1. Both forces obey inverse square law gravitation, the corresponding gravitational
i.e., attraction is

2. Both forces are proportional to product where mp and me are the masses of the
of masses or charges. proton and electron.
3. Both are central forces i.e., they act Hence
along the line joining the centres of the
two bodies. | |
4. Both are conservative forces i.e., the
work done against these forces does But k = 9 × 109 Nm2 C-2, e = 1.6 × 10-19 C,
not depend upon the path followed. mp = 1.67× 10-27 kg, me = 9.1 × 10-31 kg,
5. Both forces can operate in vacuum. G = 6.67 × 10-11 Nm2 kg-2
Dissimilarities :
∴ | |
1. Gravitational force is attractive while
electrostatic force may be attractive or = 2.27 × 1039
repulsive.
(a) (ii) Similar to that in part (i), the
2. Gravitational force does not depend on ratio of the magnitudes of electric force to
the nature of the medium while the gravitational force between two
electrostatic force depends on the protons at a distance r is given by
nature of the medium between the two
charges.
| |
3. Electrostatic forces are much stronger
than gravitational forces.
Illustrative Problem. Coulomb's law for
electrical force between two charges and = 1.24 × 1036
Newton's law for gravitational force
Thus the large value of the
between two masses, both have inverse-
(dimensionless) ratio of the two forces

21
MITTAL INSTITUTE

indicates that the electrostatic forces are 1.17 FORCES BETWEEN MULTIPLE
enormously stronger than the gravitational CHARGES : THE SUPERPOSITION
forces. PRINCIPLE
(b) The magnitude of the electric 27. State the principle of superposition
force exerted by a proton on an electron is of electrostatic forces. Hence write an
equal to the magnitude of the force expression for the force on a point charge
exerted by an electron on a proton. The due to a distribution of N -1 point charges
magnitude of this force is in terms of their position vectors.
Principle of superposition of
electrostatic forces. Coulomb's law gives
force between two point charges. The
[∵ r = 1 Å = 10-10 m] principle of superposition enables us to
= 2.3 × 10-8 N find the force on a point charge due to a
group of point charges. This principle is
Acceleration of the electron due to the based on the property that the forces with
mutual attraction with the proton, which two charges attract or repel each
other are not affected by the presence of
other charges.
The principle of superposition states
Acceleration of the proton due to the that when a number of charges are
mutual attraction with the electron, interacting, the total force on a given
charge is the vector sum of the forces
exerted on it due to all other charges. The
force between two charges is not affected
Clearly, the acceleration of an electron by the presence of other charges.
or a proton due to the electric force is
As shown in Fig. 1.24, consider N
much larger than the acceleration due to
point charges q1, q2, q3,…, qN placed in
gravity. So, we can neglect the effect of
vacuum at points whose position vectors
gravitational field on the motion of the
w.r.t. origin O are ⃗⃗⃗⃗ , ⃗⃗⃗⃗ , ⃗⃗⃗⃗ , ..., ⃗⃗⃗⃗
electron or the proton.
respectively.
26. Give two examples which illustrate
According to the principle of
that the electrical forces are enormously
superposition, the total force on charge ql
stronger than the gravitational forces.
is given by
Examples : (i) A plastic comb passed
⃗⃗⃗⃗⃗ , = + ⃗⃗ + …... + ⃗⃗
through hair can easily lift a piece of paper
upwards. The electrostatic attraction where + ⃗⃗ + …... + ⃗⃗ are the forces
between the comb and the piece of paper exerted on charge q1 by the individual
overcomes the force of gravity exerted by charges q2, q3, …..., qN respectively.
the entire earth on the paper.
(ii) When we hold a book in our hand,
the electric (frictional) forces between the
palm of our hand and the book easily
overcome the gravitational force on the
book due to the entire earth.
In the words of Feynman, if you stand
at arm's length from your friend and
instead of being electrically neutral each
of you had an excess of electrons over
protons by just one per cent, then the
force of repulsion between you would be
enough to lift the entire earth.

22
MITTAL INSTITUTE

Fig. 1.24 Superposition principle : Force ⃗⃗⃗



⃗⃗⃗
on charge q1 exerted by q2 and q3.
According to Coulomb's law, the force Examples based on
exerted on charge q1 due to q2 is Principle of Superposition of Electric
⃗⃗⃗⃗⃗
Forces
̂
Formulae Used
⃗⃗⃗ ⃗⃗⃗
⃗⃗⃗ ⃗⃗⃗ ⃗⃗⃗ ⃗⃗⃗
⃗⃗⃗ ⃗⃗⃗ √
⃗⃗⃗ ⃗⃗⃗
Units Used
⃗⃗⃗⃗ ⃗⃗⃗⃗
where ̂ a unit vector pointing
⃗⃗⃗⃗ ⃗⃗⃗⃗ Forces are in newton, charges in coulomb
from q2 to q1 and r12 = ⃗⃗⃗ ⃗⃗⃗ = distance and distances in metre.
of q2 from q1.
Example 19. An infinite number of charges
Hence the total force on charge q1 is each equal to 4 μC are placed along x-
axis at x = 1 m, x = 2 m, x = 4 m, x = 8 m
⃗⃗⃗ * ̂ ̂ ̂ + and so on. Find the total force on a charge
of 1 C placed at the origin.
or ⃗⃗⃗ ∑ ̂
[IIT 95]
-6
In terms of position vectors, Solution. Here q = 4 μC = 4 × 10 C, q0 =
⃗⃗⃗ ⃗⃗⃗ ⃗⃗⃗ ⃗⃗⃗ 1C
⃗⃗⃗ *
⃗⃗⃗ ⃗⃗⃗ ⃗⃗⃗ ⃗⃗⃗ By the principle of superposition, the
⃗⃗⃗ ⃗⃗⃗ total force acting on a charge of 1 C
+ placed at the origin is
⃗⃗⃗ ⃗⃗⃗
⃗⃗⃗ ⃗⃗
or ⃗⃗⃗ ∑
⃗⃗⃗ ⃗⃗
* +

In general, force ⃗⃗⃗ on th charge located


at ⃗⃗⃗ due to all other (N - 1) charges may * +
be written as
Sum of the infinite geometric progression
⃗⃗⃗ = Total force on th charge
⃗⃗⃗ ⃗⃗
⃗⃗⃗ ∑
⃗⃗⃗ ⃗⃗

Total force on charge ∴


⃗⃗⃗ ⃗⃗⃗
∑ ∑
⃗⃗⃗ ⃗⃗⃗ Example 20. Consider three charges q1,
q2, q3 each equal to q at the vertices of an
where a = 1,2,3,..., N. equilateral triangle of side l. What is the
force on a charge Q (with the same sign
It may be noticed that for each choice as q) placed at the centroid of the triangle
of a, the summation on b omits the value ?
a. This is because summation must be
taken only over other charges. The above [NCERT]
expression can be written in a simpler way Solution. Suppose the given charges
as follows : are placed as shown in Fig. 1.25(a).
= Total force on charge q due to many
point charges q’

23
MITTAL INSTITUTE

Solution. At any vertex, the charge will


be in equilibrium if the net electric force
due to the remaining three charges is
zero.

Fig. 1.26
Let Q be the charge required to be
Fig. 1.25 kept at the centroid G. Then,
Let AO = BO = CO = r ⃗⃗⃗ = Force at A due to the charge at B
Force on charge Q due to q1, , along ⃗⃗⃗⃗⃗
⃗⃗⃗ ̂ ⃗⃗⃗ = Force at A due to charge at
, along ⃗⃗⃗⃗⃗
Force on charge Q due to q2,

⃗⃗⃗ ̂ ⃗⃗⃗ + ⃗⃗⃗ = 2F1 cos30°, along ⃗⃗⃗⃗⃗ = √ ,


along ⃗⃗⃗⃗⃗
Force on charge Q due to q3 ,
Force at A due to charge at G
⃗⃗⃗ ̂


By the principle of superposition, the
total force on charge Q is This must be equal and opposite to ( 1 +
⃗⃗⃗ ⃗⃗⃗ ⃗⃗⃗ 2).

∴ √ or √
[ ̂ ̂ ̂] ∵
Example 22. Consider the charges q, q
As shown in Fig. 1.25(b), the angle and - q placed at the vertices of an
between each pair of the unit vectors ̂ , equilateral triangle, as shown in Fig. 1.27.
̂ and ̂ is 120°, so they form a triangle What is the force on each charge ?
of cyclic vectors. Consequently, [NCERT]
̂ ̂ ̂ =0 Solution. The forces of attraction or
repulsion between different pairs of
Hence = 0 i.e., the total force on charge charges are shown in Fig. 1.27. Each
Q is zero. such force has magnitude,
Example 21. Three point charges +q each
are kept at the vertices of an equilateral
triangle of side ‘ ’. Determine the
magnitude and sign of the charge to be
kept at its centroid so that the charges at
the vertices remain in equilibrium.
[CBSE F 2015]

24
MITTAL INSTITUTE

Fig. 1.27
By the parallelogram law, the net force Fig. 1.28
on charge q1 is
The charge at C attracts the charge at
By the parallelogram law, the net force on charge
A with a force
is

⃗⃗⃗ √ ̂

√ ̂ ̂
= 180 N, along AC.
where ̂ is a unit vector along BC. By the parallelogram law of vector
Similarly, total force on charge q2 is addition, the magnitude of resultant force
on charge at A is
⃗⃗⃗ ̂

where ̂ is a unit vector along AC. √

Total force on charge q3 is √


⃗⃗⃗ √ ̂ √ ̂

where ̂ is a unit vector along the direction
bisecting . Let the resultant force F make an
ng β with th or F2. Then
Example 23. Charges of + 5 μC, + 10 μC
and -10 μC are placed in air at the corners t nβ=
A, B and C of an equilateral triangle ABC,
having each side equal to 5 cm. √
Determine the resultant force on the √
charge at A.
Solution. The charge at B repels the ∴ β = 60°
charge at A with a force, i.e., the resultant force is parallel to BC.
Example 24. Four equal point charges
each 16 μC are placed on the four corners
= 180 N, along BA of a square of side 0.2 m. Calculate the
force on any one of the charges.
Solution. As shown in Fig. 1.29,
suppose the four charges are placed at
the corners of the square ABCD. Let us
calculate the total force on q4.

25
MITTAL INSTITUTE

Fig. 1.30
Solution. As shown in Fig. 1.30(b), the
Fig. 1.29 or x rt d on h rg + 2 μC by h rg
at B,
Here AB = BC = CD = AD = 0.2 m
q1 = q2 = q3 = q4 = 16 μC = 16 × 10-6 C
Force exerted on q4 by q1 is

= 1.35 N, along AB
= 57.6 N, along AD produced For x rt d on h rg +2μC by
Force exerted on q4 by q2 is charge at C,

= 28.8 N, along BD produced = 1.35 N, along AC


Force exerted on q4 by q3 is Resultant force of F1 and F2

= 57.6 N, along CD produced √

As F1 and F3 are perpendicular to each √ , along AM


other, so their resultant force is For the charge at A to be equilibrium,
the charge q to be placed at point M must
√ √
be a positive charge so that it exerts a
or on +2 μC h rg ong MA.
= 57.6√ = 81.5 N, in the direction of
F2. Hence total force on q4 is Now, √
F = F2 + F’ = 28.8 + 81.5 √ √ cm
= 110.3 N, along BD produced. √
Example 25. Three point charges of +2 Net force on charge at A will be zero if
μC, -3 μC and -3 μC are kept at the
vertices A, B and C respectively of an
equilateral triangle of side 20 cm as ( √ )
shown in Fig. 1.30(a). What should be the
sign and magnitude of the charge to be or
placed at the midpoint (M) of side BC so
that the charge at A remains in equilibrium Problems for Practice
? [CBSE D 05] 1. Ten positively charged particles are
kept fixed on the x-axis at points x =
10 cm, 20 cm, 30 cm, ..., 100 cm. The

26
MITTAL INSTITUTE

first particle has a charge 1.0 × 10-8 C, 1. By the principle of superposition, the
the second 8 × 10-8 C, third 27 × 10-8 total force on the 1 C charge placed at the
C, and so on. The tenth particle has a origin is
charge 1000 × 10-8 C. Find the

magnitude of the electric force acting
on a 1 C charge placed at the origin.
* +
(Ans. 4.95 × 105N)
2. Charges q↑ =1.5 mC, q2 = 0.2 mC and *
q3 = - 0.5 mC are placed at the points
A, B and C respectively, as shown in +
Fig. 1.31. If r1 = 1.2 m and r2 = 0.6 m,
calculate the magnitude of resultant
force on q2. (Ans. 3.125 × 103 N) N.

2.

, along AB produced

, along BC AB
As , so the resultant force on q2
Fig. 1.31 is
3. Two equal positive charges, each of 2 √ N.
μC int r t with third positiv h rg 3. Here
o 3μC situ t d s shown in Fig. 1.32.
Find the magnitude and direction of
the force experienced by the charge of
3 μC. (Ans. 3.456 × 10-3 N, along OC √
produced)

Fig. 1.33,
Fig. 1.32 Force exerted by charge q4 on qC,
4. Four charges + q, + q, -q and -q are
placed respectively at the four corners A,
B, C and D of a square of side a.
Calculate the force on a charge Q placed
at the centre of the square. = 2.16 × 10-3 N, along AC produced
(Ans. √
parallel to AD or BC) Similarly, force exerted by charge qB on
q c,
HINTS FB = 2.16 × 10-3 N, along BC produced
Clearly, FA = FB (in magnitude)

27
MITTAL INSTITUTE

The components of FA and FB along Y- charged bodies even without any direct
axis will cancel out and get added along contact between them. The nature of this
X-axis. action- at-distance force can be
understood by introducing the concept of
∴ Tot or on 3 μC h rg ,
electric field.

= 3.456 × 10-3 N, along CX.


4. Here AB = BC = CD = DA = a
AO = BO = CO = DO = √ =
√ Fig. 1.35 A charged body produces an
electric field around it.
Consider a charged body carrying a
positive charge q placed at point O. It is
assumed that the charge q produces an
electrical environment in the surrounding
space, called electric field.
To test the existence of electric field at
any point P, we simply place a small
positive charge q0, called the test charge
Fig. 1.34 at the point P. If a force is exerted on
Let FA, FB, FC and FD be the forces the test charge, then we say that an
exerted by charges at points A, B, C and electric field ⃗ exists at the point P. The
D on charge Q at point O. Then charge q is called the source charge as it
produces the field ⃗ .
29. Define electric field at a point. Give
its units and dimensions.

Electric field. An electric field is said to
The resultant of the forces and
exist at a point if a force of electrical origin
is exerted on a stationary charged body
placed at that point. Quantitatively, the
electric field or the electric intensity or the
or , along OL
electric field strength ⃗ at a point is
Similarly, resultant of the forces and defined as the force experienced by a unit
positive test charge placed at that point,
, along OM without disturbing the position of source
charge.
Hence the resultant force on charge is
As shown in Fig. 1.35, suppose a test

√ , along ON charge q0 experiences a force at the
point P. Then the electric field at that point
As the forces F1 and F2 are equal in will be
magnitude, so their resultant F will act
along the bisector of COD i.e., parallel to ⃗
AD or BC.
1.18 ELECTRIC FIELD There is a difficulty in defining the
electric field by the above equation. The
28. Briefly develop the concept of
test charge q0 may disturb the charge
electric field.
distribution of the source charge and
Concept of electric field. The hence change the electric field ⃗ which we
electrostatic force acts between two want to measure. The test charge q0 must

28
MITTAL INSTITUTE

be small enough so that it does not Electrostatic force = Charge × Electric


change the value of ⃗ . It is better to define field.
electric field as follows : Thus an electric field plays an
The electric field at a point is defined intermediary role in the forces between
as the electrostatic force per unit test two charges :
charge acting on a vanishingly small Charge Electric field Charge.
positive test charge placed at that point.
Hence It is in this sense that the concept of
electric field is useful. Electric field is a
⃗ characteristic of the system of charges

and is independent of the test charge that
we place at a point to determine the field.
The electric field ⃗ is a vector quantity
whose direction is same as that of the Examples based on
force exerted on a positive test charge. Relation between Electric Field
Units and dimensions of electric field. Strength and Force
As the electric field is force per unit Formulae Used
charge, so its SI unit is newton per
coulomb (NC-1). It is equivalent to volt per ⃗ or ⃗
metre (Vm_1).
Units Used
The dimensions for ⃗ can be
determined as follows : When force is in newton charge in
coulomb and distance in metre, electric
field strength is in newton per coulomb
(NC-1) or equivalently in volt per metre
(Vm-1).
*∵ +
Example 26. Calculate the electric field
30. Give the physical significance of strength required to just support a water
electric field. drop of mass 10-3 kg and having a charge
1.6 × 10-19 C. [CBSE OD 99]
Physical significance of electric field.
The force experienced by the test charge Solution. Here m = 10-3 kg, q = 1.6 ×
-19
q0 is different at different points. So ⃗ also 10 C
varies from point to point. In general, ⃗ is Let E be the strength of the electric
not a single vector but a set of infinite field required to just support the water
vectors. Each point is associated with a drop. Then
unique vector ⃗ (r). So electric field is an Force on water drop due to electric
example of vector field. field
By knowing electric field at any point, = Weight of water drop
we can determine the force on a charge
placed at that point. The Coulomb force or qE = mg
on a charge q0 due to a source charge q ∴
may be treated as two stage process :
.
(i) The source charge q produces a
Example 27. Calculate the voltage needed
definite field ⃗ (r) at every point .
to balance an oil drop carrying 10
(ii) The value of ⃗ (r) at any point electrons when located between the
determines the force on charge q0 at plates of a capacitor which are 5 mm
that point. This force is apart. The mass of oil drop is 3 × 10-16 kg.
Take g = 10 ms-2.

[CBSE OD 95C]

29
MITTAL INSTITUTE

Solution. Here q = 10 e = 10 × 1.6 × of 2 × 104 Vm-1. Find the tension in the


-19
10 C, thread of the pendulum and the angle it
makes with the vertical.
d = 5 mm = 5 × 10-3 m, m = 3 × 10-16 kg, g
= 10 ms-2 Solution. Here m = 80 mg = 80 × 10-6 kg,
q = 2 × 10-8 C, E = 2 × 104 Vm-1.

Fig. 1.36
When the drop is held stationary,
Upward force on oil drop due to electric
field = Weight of oil drop
qE = mg
or *
+ Fig. 1.38
Let T be the tension in the thread and
∴ θ b th ng it k s with v rti , s
shown in Fig. 1.38. When the bob is in
= 9.375 V. equilibrium,
Example 28. How many electrons should T sin θ = qE ; T os θ = mg
be removed from a coin of mass 1.6 g, so
that it may just float in an electric field of ∴t nθ=
intensity 109 NC-1, directed upward ?. [Pb.
98C] Solution. Here m = 1.6 g = 1.6 × 10-3
kg,
E = 109 NC-1 or θ = 27°

Also,
= 8.81 × 10-4 N.
Example 30. An electron moves a
distance of 6 cm when accelerated from
Fig. 1.37 rest by an electric field of strength 2 × 104
Let n be the number of electrons NC-1. Calculate the time of travel. The
removed from the coin. mass and charge of electron are 9 × 10-31
kg and 1.6 × 10-19 C respectively. [CBSE D
Then charge on the coin, 91]
q = + ne Solution. Force exerted on the electron
When the coin just floats, by the electric field,

Upward force of electric field = Weight of F = eE


coin :. Acceleration,
qE or neE = mg
∴ Now u = 0, s = 6.0 cm = 0.06 m, a = 0.35
Example 29. A pendulum of mass 80 × 1016 ms-2
milligram carrying a charge of 2 × 10-8 C is As
at rest in a horizontal uniform electric field

30
MITTAL INSTITUTE

∴ 0.06 = 0 + × 0.35 × 1016 × t2


√ √
or t =√ = 0.585 × 10-8
s.
Example 31. An electron falls through a √
distance of 1.5 cm in a uniform electric
field of magnitude 2.0 × 104 NC-1 [Fig. Thus the heavier particle takes a
1.39(a)]. The direction of the field is greater time to fall through the same
reversed keeping its magnitude distance. This is in contrast to the situation
unchanged and a proton falls through the of ‘ ree fall under gravity' where the time of
same distance [Fig. 1.39(b)]. Compute the fall is independent of the mass of the
time of fall in each case. Contrast the body. Here the acceleration due to gravity
situation (a) with that of 'free fall under 'g', being negligibly small, has been
gravity'. [NCERT] ignored.
Example 32. An electron is liberated from
the lower of the two large parallel metal
plates separated by a distance of 20 mm.
The upper plate has a potential of +2400
V relative to the lower plate. How long
does the electron take to reach the upper
plate ? Take of electrons 1.8 × 10-11 C
kg-1.
Solution. Here V = 2400 V, d = 20 mm =
0.02 m, = 1.8 × 10-11 C kg-1.
Upward force on the electron exerted
by electric field is

Fig. 1.39
Solution, (a) The upward field exerts a ∴ Acceleration,
downward force eE on the electron.
∴ Acceleration of the electron,
= 2.16 × 1016 ms-2
As
Using, s = , we get
∴ Time of fall of the electron is
√ √ √
√ √ √
Example 33. A stream of electrons moving
= 2.9 × 10-9 s. with a velocity of 3 × 107 ms-1 is deflected
(b) The downward field exerts a by 2 mm in traversing a distance of 0.1 m
downward force eE on the proton. in a uniform electric field of strength 18 V
cm-1. Determine e/m of electrons.
∴ √ Solution. Here v0 = 3 × 107 ms-1,
y = 2 mm = 2 × 10-3m, x = 0.1 m,
Time of fall of the proton is
E = 18 V cm-1 = 1800 V m-1

31
MITTAL INSTITUTE

ma = eE or a = and t = vx = 1000 ms-1, is constant


The velocity of the particle, along the
∴ and t = direction of field, after 10 s, is given by

or
=2000 ms-1
= 2 ×1011Ckg_1.
Example 34. An electric field E is set up The net velocity after 10 s,
between the two parallel plates of a √ √ √
capacitor, as shown in Fig. 1.40. An
electron enters the field symmetrically Displacement, along the x-axis, after 10 s,
between the plates with a speed vQ. The x = 1000 × 10 m = 10000 m
length of each plate is l. Find the angle of
deviation of the path of the electron as it Displacement along y-axis (in the
comes out of the field. direction of field) after 10 s,

=10000 m
Net displacement,
Fig. 1.40
√ √
Solution. Acceleration of the electron √
in the upward direction,
r = ^x + y = V(10000)2 + (10000)2 =
2 2

Problems For Practice


1. If an oil drop of weight 3.2 × 10-13 N is
Time taken to cross the field, balanced in an electric field of 5 × 103 Vm-
1
, find the charge on the oil drop. [CBSE D
Upward component of electron velocity 93] (Ans. 0.64 × 10-18 C)
on emerging from field region,
2. Calculate the magnitude of the electric
field, which can just balance a deutron
of mass 3.2 × 10-27 kg. Take g = 10
Horizontal component remains same, vx = ms-2.
vQ [Punjab 99]
I θ is th ng o d vi tion o th p th (Ans. 2.0 × 10- 7 NC-1)
of the electron, then
3. A charged oil drop remains stationary
t nθ= or θ = tan-1 . when situated between two parallel
plates 20 mm apart and a p.d. of 500
Example 35. A charged particle, of charge V is applied to the plates. Find the
2μC nd ss 10 i igr , oving with charge on the drop if it has a mass of
velocity of 1000 m/s entres a uniform 2 × 10-4 kg. Take g - 10 ms-2. (Ans.
electric field of strength 103 NC-1 directed 8 × 10 -13 C)
perpendicular to its direction of motion.
Find the velocity and displacement, of the 4. In Millikan's experiment, an oil drop of
particle after 10 s. radius 10-4cm remains suspended
between the plates which are 1 cm
[CBSE Sample Paper 11] apart. If the drop has charge of 5e
Solution. The velocity of the particle, over it, calculate the potential
normal to the direction of field. difference between the plates. The

32
MITTAL INSTITUTE

density of oil may be taken as 1.5 gcm- 31. Obtain an expression for the
3
. (Ans. 770 V) electric field intensity at a point at a
distance r from a charge q. What is the
5. A proton falls down through a distance
nature of this field ?
of 2 cm in a uniform electric field of
magnitude 3.34 × 103NC-1. Determine Electric field due to a point charge. A
(i) the acceleration of the electron (ii) single point charge has the simplest
the time taken by the proton to fall electric field. As shown in Fig. 1.41,
through the distance of 2 cm, and (Hi) consider a point charge q placed at the
the direction of the electric field. Mass origin O. We wish to determine its electric
of a proton is 1.67 × 10-27kg. field at a point P at
(Ans. 3.2 × 1011 ms-2, 3.54 × 10 -7
s,
vertically downwards)
6. A particle of mass 10-3 kg and charge
5 μC is thrown t sp d o 20 s-1
against a uniform electric field of Fig. 1.41 Electric field of a point charge.
strength 2 × 105 NC-1. How much a distance r from it. For this, imagine a
distance will it travel before coming to test charge q0 placed at point P. According
rest momentarily ? to Coulomb's law, the force on charge q0
(Ans. 0.2 m) is
HINTS ̂
1. Use W = qE.
2.
where ̂ is a unit vector in the direction
3. mg = qE or mg = q from q to q0. Electric field at point P is

∴ ⃗ ̂

4. Use The magnitude of the field ⃗ is


5. (i)

Clearly, ∝ .This means that at all


(ii) points on the spherical surface drawn
around the point charge, the magnitude of
∴ √ √ ⃗ is same and does not depend on the
direction of . Such a field is called
(iii) The field must act vertically spherically symmetric or radial field, i.e., a
downwards so that the positively charged field which looks the same in all directions
proton falls downward. when seen from the point charge.
6. 1.20 ELECTRIC FIELD DUE TO A
SYSTEM OF POINT CHARGES
As the particle is thrown against the field,
so 32. Deduce an expression for the
electric field at a point due to a system of
N point charges.

As ∴ Electric field due to a system of point


charges. Consider a system of N point
or charges q1, q2, ……, qN having position
1.19 ELECTRIC FIELD DUE TO A vectors , , ......, , with respect to the
POINT CHARGE

33
MITTAL INSTITUTE

Fig. 1.42 Notations used in the


determination of electric field at a point
due to two point charges.
origin O. We wish to determine the electric
field at point P whose position vector is .
According to Coulomb's law, the force on Fig. 1.43 Electric field at a point due to a
charge test q0 due to charge ql is system of charges is the vector sum of
the electric fields at the point due to
̂ individual charges.
In terms of position vectors, we can write
where ̂ is a unit vector in the direction
from q1 to P and ̂ is the distance ⃗
⃗ ∑
between q1 and P. Hence the electric field ⃗ ⃗
at point P due to charge q1 is
or ⃗ ∑ ⃗⃗⃗
⃗ .
⃗ ̂ Examples based on

Similarly, electric field at P due to Electric Fields of Point Charges


charge q2 is Formulae Used
⃗ ̂ 1.

2. By the principle of superposition,


According to principle of superposition
electric field due to a number of point
of electric fields, the electric field at any
charges,
point due to a group of charges is equal to
the vector sum of the electric fields ⃗ ⃗⃗⃗⃗ ⃗⃗⃗⃗ ⃗
produced by each charge individually at
Units Used
that point, when all other charges are
assumed to be absent. When q is in coulomb and r in metre; E is
in NC_1 or Vm-1.
Hence, the electric field at point P due
to the system of N charges is Example 36. Assuming that the charge on
⃗ ⃗⃗⃗⃗ ⃗⃗⃗⃗ ⃗
an atom is distributed uniformly in a
sphere of radius 10-10 m, what will be the
electric field at the surface of the gold
* ̂ ̂ ̂ +
atom ? For gold, Z =79.
or ⃗ ∑ ̂ Solution. The charge may be assumed
to be concentrated at the centre of the
sphere of radius 10-10 m.
∴ r = 10-10 m, q = Ze = 79 × 1.6 × 10-
19
C

34
MITTAL INSTITUTE

Determine the position of the point at


which the resultant field is zero.
= 1.138 × 1013 NC-1. Solution. Let P be the point at distance
x cm from A, where the net field is zero.
Example 37. Two point charges of 2.0
×10-7 C and 1.0 × 10-7 C are 1.0 cm apart.
What is the magnitude of the field
produced by either charge at the site of
the other ? Use standard value of 1/4πε0.
Fig. 1.45
Solution. Here q1 = 2.0 × 10-7 C,
At point P,
q2 = 1.0 × 10-7 C, r = 1.0 cm = 0.01 m
Electric field due to q1 at the site of q2,

or

= 1.8 × 107 NC-1. or


Electric field due to q2 at the site of q1, or cm, 32
cm

At , both E1 and E2 will be in the


= 9 ×106 NC-1. same 7 direction, therefore, net electric
Example 38. Two point charges +4μC and field cannot be zero.
+1μC are separated by a distance of 2 m Hence x = 32 cm
in air. Find the point on the line-joining
charges at which the net electric field of i.e., electric field is zero at a point 24 cm
the system is zero. [CBSE OD 17C] to the right of - 9 μC h rg .
Solution. Example 40. Two point charges q1 = + 0.2
C and q2 = + 0.4 C are placed 0.1 m apart.
Calculate the electric field at
(a) the midpoint between the charges.
(b) a point on the line joining q1 and q2
Fig. 1.44 such that it is 0.05 m away from q2 and
The electric field at point P will be zero 0.15 m away from q↑.
if Solution, (a) Let O be the midpoint
between the two charges.

or or
or Fig. 1.46
At , both E1 and E2 will be in the Electric field at O due to q1
same direction, therefore, net electric field
cannot be zero. NC-1,

Hence electric field will be zero at acting along AO


to the right of + 4μC charge.
Electric field at O due to q2,
Example 39. Two point charges of +16 μC
and -9 μC are placed 8 cm apart in air.

35
MITTAL INSTITUTE

, acting along BO The electric field vector ⃗ at A due to


Net field at the negative charge q2 points towards the
right and it has a magnitude,
, acting along BO
(b) Electric field at due to
Magnitude of the total electric field at A
, acting along AP

Electric field at due to = 3.6 × 104 + 3.6 × 104 = 7.2 × 104 NC-
1
acting along BP
⃗ is directed towards the right.

The electric field vector ⃗ at B due to


the positive charge q1 points towards the
left and it has a magnitude,
Fig. 1.47 -1
NC
Net electric field at point P is
The electric field vector ⃗ at B due to
[ ] the negative charge points towards the
right and it has a magnitude,
= 1.52 × 1012 NC-1, acting along
AP.
Example 41. Two point charges q1 and q2 Magnitude of the total electric field at B
of 10-8 C and -10-8 C respectively are
placed 0.1 m apart. Calculate the electric
fields at points A, B and C shown in Fig. ⃗ is directed towards the left.
1.48.
Magnitude of each electric field vector,
[NCERT] at point C, of charges q1 and q2 is
-1
NC

The directions in which these two


vectors point are shown in Fig. 1.48. The
resultant of these vectors is given by

Fig. 1.48 Since ⃗ and ⃗ are equal in


magnitude, so their resultant ⃗ acts
Solution. The electric field vector ⃗ at A
due to the positive charge q1 points along the bisector of the angle between ⃗
towards the right and it has a magnitude, and ⃗ , i.e., towards right.
Example 42. ABCD is a square of side 5
m. Charges of + 50 C, - 50 C and + 50 C
are placed at A, C and D respectively.
= 3.6 ×104 NC-1 Find the resultant electric field at B.
Solution. Electric field at B due to + 50
C charge at A is

36
MITTAL INSTITUTE

, along AB √

√ ( ) ( ) √
√ √

NC-1 NC-1
I th r su t nt i d E k s ng β
with x-axis, then


or

Example 43. Four charges + q, + q, - q, - q


are placed respectively at the four corners
Fig. 1.49 A, B, C and D of a square of side 'a'.
Calculate the electric field at the centre of
Electric field at B due to -50 C charge at C
the square.
is
[Punjab 96C]
, along BC
Solution. Let EA, EB, EC and ED be the
Electric field at B due to + 50 C charge at electric fields at the centre O of the square
D is due to the charges at A, B, C and D
respectively. Their directions are as
, along DB shown in Fig. 1.50(a).

Component of E1 along x-axis = 2k


(as it acts along x-axis)
Component of E2 along x-axis = 0
(as it acts along y-axis)
Component of E3 along x-axis

√ √ Fig. 1.50
∴ Total electric field at B along x-axis Since all the charges are of equal
magnitude and at the same distance r
from the centre O, so
√ √
Now,
Component of E1 along y-axis = 0 √

Component of E2 along y-axis = 2k
Because EA and EC act in the same
Component of E3 along y-axis
direction, so their resultant is
sin
√ √

But the components of E2 and E3 act in


opposite directions, therefore, total electric Similarly, resultant of EB and ED is
field at B along y-axis

√ √ Now, the resultant of E1 and E2 will be a


∴ Resultant electric field at B will be

37
MITTAL INSTITUTE


√ √
( )

√ ,

directed parallel to AD or BC, as shown in √


∴ ( )
Fig. 1.50(b).
Problems For Practice

√ 1. An electron is separated from the
i.e., the resultant field is inclined at an proton through a distance of 0.53 Å.
angle of 45o with AC. Calculate the electric field at the location
of the electron. (Ans. 5.1 × 1011 NC-1)
Example 44. Two point charges +6q and -
8q are placed at the vertices 'E’ and 'C’ of 2. Determine the electric field produced
n qui t r tri ng ABC o sid ‘ ’ s by a helium nucleus at a distance of 1
shown in Fig. 1.51(a). Obtain the Å from it.
expression for (i) the magnitude and (ii) (Ans. 2.88 ×1011 NC-1)
the direction of the resultant electric field
at the vertex A due to these two charges. 3. Two point charges + q and + 4q are
[CBSE OD 14C] separated by a distance of 6a. Find
the point on the line joining the two
Solution, (i) As shown in Fig. 1.51(b), the charges where the electric field is
fields at point A due to the charges at B zero.
and C are ⃗ BA and ⃗ AC respectively.
(Ans. At a distance 2a from charge + q)
4. Two point charges q1 and q2 of 2 × 10-
8
C and - 2 × 10-8C respectively are
placed 0.4 m apart. Calculate the
electric field at the centre of the line
joining the two charges. [CBSE F
94C]
(Ans. 900 NC-1, towards the -ve charge)
5. Two point charges + q and -2q are
placed at the vertices 'B' and ‘C’ of an
Fig. 1.51 equilateral triangle ABC of side V as
given in the figure. Obtain the
Their magnitudes are expression for (i) the magnitude and
, where (ii) the direction of the resultant electric
field at the vertex A due to these two
charges. [CBSE OD 14C]

[Ans. (i) (ii) 30° with AC]
The magnitude of the resultant field is

√ ( )


√ Fig. 1.52
6. Find the magnitude and direction of
(ii) I th r su t nt i d k s n ng β electric field at point P in Fig. 1.53.
with AC, then

38
MITTAL INSTITUTE

Electric field is zero at distance 2a


from the charge + q.
4. Proceed as in Q. 1.8 on page 1.82.
5. Proceed as in the solution of Example
44 on page 1.33.
6. Here EA and EC are equal and
opposite and hence cancel out.
sin √
Fig. 1.53 Hence √
(Ans. , along BP produced)
, along BP produced.
7. Three charges, each equal to q are
placed at the three comers of a square 7. Refer to Fig. 1.55.
of side a. Find the electric field at the
fourth corner. (Ans. √
)

8. Figure 1.54 shows four point charges


at the comers of a square of side 2
cm. Find the magnitude and direction
of the electric field at the centre O of
th squ r , i Q = 0.02 μC.

Fig. 1.55



* + √
Fig. 1.54
Use = 9 × 109 Nm2 C-2. [ISCE 8. Here, cm
98] √
∴ √
(Ans. 9√ ×105NC-1, parallel to BA)
HINTS √
1. Electric field at the location of the
electron,

2. Here and r = 1Å = 10-10 m.


3. Suppose the electric field is zero at
distance x from the charge + q. Then

Fig. 1.56
or or ∴ , along OA
or

39
MITTAL INSTITUTE

, along OD ⃗⃗⃗⃗⃗ ̂
, along OC ̂
where ̂ = , is a unit vector pointing from
and , along OB the small charge dq towards the point
charge q0. By the principle of
Net electric field along OA, superposition, the total force on charge q0
will be the vector sum of the forces
exerted by all such small charges and is
given by
Net electric field along OD,

Hence, the resultant electric field at point


O,


, parallel to side BA
Fig. 1.57 Force on a point charge q 0 due
-6
But, Q = 0.02 μC = 0.02 × 10 C to a continuous charge distribution.

∫ ⃗⃗⃗⃗⃗ ∫ ̂

√ , parallel to side BA. or ∫ ̂
1.21 CONTINUOUS CHARGE
DISTRIBUTION 34. Name the different types of
continuous charge distributions. Define
33. What is a continuous charge their respective charge densities. Write
distribution ? How can we calculate the expression for the electric field produced
force on a point charge q due to a by each type of charge distribution. Hence
continuous charge distribution ? write expression for the electric field of a
Continuous charge distribution. In general source charge distribution.
practice, we deal with charges much Different types of continuous charge
greater in magnitude than the charge on distributions. There are three types of
an electron, so we can ignore the continuous charge distributions:
quantum nature of charges and imagine
that the charge is spread in a region in a (a) Volume charge distribution. It is a
continuous manner. Such a charge charge distribution spread over a three
distribution is known as a continuous dimensional volume or region V of space,
charge distribution. as shown in Fig. 1.57. We define the
volume charge density at any point in this
Calculation of the force on a charge volume as the charge contained per unit
due to a continuous charge distribution. volume at that point, i.e.,
As shown in Fig. 1.57, consider a point
charge q0 lying near a region of =
continuous charge distribution. This
The SI unit for is coulomb per cubic
continuous charge distribution can be
metre (Cm 3).
imagined to consist of a large number of
small charges dq. According to Coulomb's For example, if a charge q is
law, the force on point charge q0 due to distributed over the entire volume of a
small charge dq is sphere of radius R, then its volume charge
density is

40
MITTAL INSTITUTE

= conductor of radius R, then its surface


charge density is
= Cm-2
The charge contained in small area dS is
dq = σ dS
Total electrostatic force exerted on
charge q0 due to the entire surface S is
given by

∫ ̂
Fig. 1.58 Volume charge distribution
The charge contained in small volume Electric field due to the surface charge
dV is distribution at the location of charge q0 is

dq = dV ⃗ ∫ ̂
Total electrostatic force exerted on
charge q0 due to the entire volume V is
(c) Line charge distribution. It is a
given by
charge distribution along a one-
dimensional curve or line L in space, as
∫ ̂ ∫ ̂
shown in Fig. 1.60. We define the line
charge density at any point on this line as
Electric field due to the volume charge the charge per unit length of the line at
distribution at the location of charge q0 is that point, i.e.,
=
⃗ ∫ ̂
The SI unit for  is Cm-1.
(b) Surface charge distribution. It is a
charge distribution spread over a two-
dimensional surface S in space, as shown
in Fig. 1.59. We define the surface charge
density at any point on this surface as the
charge per unit area at that point, i.e.,
=
Th SI unit or σ is Cm-2.
Fig. 1.60 Line charge distribution.
For example, if a charge q is uniformly
distributed over a ring of radius R, then its
linear charge density is

The charge contained in small length is

Total electrostatic force exerted on charge


Fig. 1.59 Surface charge distribution.
due to the entire length is given by
For example, if a charge q is uniformly
distributed over the surface of a spherical ⃗⃗⃗ ∫ ̂

41
MITTAL INSTITUTE

Electric field due to the line charge Cm-2 . When its charge is increased by
distribution al the location of charge is 0.44 C, the charge density changes by
⃗⃗⃗
0.14 Cm-2. Find the radius of the sphere
⃗⃗⃗⃗ ∫ ̂ and initial charge on it.
So ution, σ =
The total electric field due to a continuous
charge distribution is given by In first case: 0.7 = ...(i)
⃗ ⃗⃗⃗⃗ ⃗⃗⃗⃗ ⃗⃗⃗⃗
In second case:
or ⃗ *∫ ̂ ∫ ̂ ∫ ̂+
0.7 + 0.14 =
General charge distribution. A general
charge distribution consists of continuous or 0.84 = ...(ii)
as well as discrete charges. Hence total Dividing (ii) by (i), we get,
electric field due to a general charge
distribution at the location of charge q0 is or
given by
⃗ ⃗ ⃗
∴ Initial charge, q = 2.2 C.

or ⃗ ∑ ̂ ∫ ̂ From (i), r = √ √
∫ ̂ ∫ ̂

In all the above cases, ̂ = / r is a √


variable unit vector directed from each
point of the volume, surface or line charge Example 46. Sixty four drops of radius
distribution towards the location of the 0.02 m and each carrying a charge of 5
point charge q0. μC are combined to form a bigger drop.
Examples based on Find how the surface density of
electrification will change if no charge is
Continuous Charge Distributions lost.
Formulae Used Solution. Volume of each small drop
1. Volume charge density, = = π(0.02)3 m3
2. Sur h rg d nsity, σ = Volume of 64 small drops

3. Linear charge density,  =


Let R be the radius of the bigger drop
4. Force exerted on a charge q0 due to a formed. Then
continuous charge distribution,

∫ ̂
or R3 = (0.02 )3 × 43
5. Electric field due to a continuous
charge distribution, ∴ R = 0.02 × 4 = 0.08 m
Ch rg on s drop = 5 μC = 5 × 10-6 C
⃗ ∫ ̂
Surface charge density of small drop,
Units Used
is in Cm-3, σ in Cm-2,  in Cm_1 and E in
NC_1. Surface charge density of bigger drop,
Example 45. A charged spherical
conductor has a surface density of 0.7 Cm

42
MITTAL INSTITUTE

∫ ∫
Example 48. A charge is distributed
uniformly over a ring of radius 'a'. Obtain * +
an expression for the electric intensity E at
a point on the axis of the ring. Hence
show that for points at large distances
from the ring, it behaves like a point
charge. [CBSE D 16] ∵

Solution. Suppose that the ring is or


placed with its plane perpendicular to the
Special case
x-axis, as shown in Fig 1.62. Consider a
small element dl of the ring. For points at large distances from the ring,
As the total charge q is uniformly
distributed, the charge dq on the element ∴
dl is
This is the same as the field due to a
point charge, indicating that for far off axial
points, the charged ring behaves as a
point charge.
Example 49. A thin semicircular ring of
radius a is charged uniformly and the
charge per unit length is . Find the
electric field at its centre. [CBSE PMT
2000, AIEEE 2010]
Solution. Consider two symmetric
Fig. 1.62 elements each of length dl at A and B.
The electric fields of the two elements
∴ The magnitude of the field ⃗ perpendicular to PO get cancelled while
produced by the element dl at the field those along PO get added.
point P is
Electric field at O due to an element of
⃗⃗⃗⃗⃗ length dl is
[Along PO]
As shown in Fig. 1.62, the field ⃗⃗⃗⃗⃗ has
two components :
1. the axial component dE os θ, nd
2. the perpendicular component dE [dl = ad ]
sin θ.
Since the perpendicular components
of any two diametrically opposite elements
are equal and opposite, they all cancel out
in pairs. Only the axial components will
add up to produce the resultant field ⃗ at
point P, which is given by

[∵Only the axial components contribute


towards E] Fig. 1.63
Total electric field at the centre O is

43
MITTAL INSTITUTE

whose magnitude is either charge times


∫ ∫
the separation between the two opposite
charges and the direction is along the
sin dipole axis from the negative to the
positive charge.
Problems for Practice
As shown in Fig. 1.64, consider an
1. A uniformly charged sphere carries a electric dipole consisting of charges + q
tot h rg o 2π × 10-12C. Its radius and - q and separated by distance 2 a.
is 5 cm and is placed in vacuum. The line joining the charges is called
Determine its surface charge density. dipole axis.
(Ans. 2 × 10-10 Cm-2)
2. What charge would be required to
electrify a sphere of radius 15 cm so Fig. 1.64
as to get a surface charge density of
Dipole moment = Either charge × a vector
μC ? drawn from negative to positive charge
(Ans. 1.8 × 0-7 C) or
3. A metal cube of length 0.1 m is Thus the dipole moment is a vector
h rg d by 12 μC. C u t its quantity. Its direction is along the dipole
surface charge density. axis from - q to + q and its magnitude is
(Ans. 2 × 10-4 Cm-2) P = q × 2a
4. Two equal spheres of water having The SI unit of dipole moment is
equal and similar charges coalesce to coulomb metre (Cm). When both the
form a large sphere. If no charge is charge q and separation 2 a are finite, the
lost, how will the surface densities of dipole has a finite size (equal to 2 a), a
electrification change ? (Ans. location (midpoint between + q and - q), a
σ1: σ2 = 22/3 : 2) direction and a strength.
HINTS Examples of electric dipoles. Dipoles
1. Use σ = . are common in nature. In molecules like
H2O, HC1, C2H5OH, CH3COOH, etc., the
2. Use q = 4πr2σ. centre of positive charges does not fall
3. Surface area of cube = 6 × l2 = 6 × exactly over the centre of negative
0.01 = 0.06 m2. charges. Such molecules are electric
dipoles. They have a permanent dipole
4. π R3 = 2 × π r3 or R = 21/3 r moment.
Ideal or point dipole. We can think of a
dipole in which size 2 a → 0 and charge q
→ ∞ in su h w y th t th dipo
1.22 ELECTRIC DIPOLE moment, p = q × 2a has a finite value.
Such a dipole of negligibly small size is
35. What is an electric dipole ? Define called an ideal or point dipole.
dipole moment and give its SI unit. Give
some examples of electric dipoles. What Dipoles associated with individual
are ideal or point dipoles ? atoms or molecules may be treated as
ideal dipoles. An ideal dipole is specified
Electric dipole. A pair of equal and only by its location and a dipole moment,
opposite charges separated by a small as it has no finite size.
distance is called an electric dipole.
1.23 DIPOLE FIELD
Dipole moment. It measures the
strength of an electric dipole. The dipole
moment of an electric dipole is a vector

44
MITTAL INSTITUTE

36. What is a dipole field ? Why does ⃗ ̂ (towards right)


the dipole field at large distance falls off
faster than 1/r2 ? Hence the resultant electric field at
point P is
Dipole field. The electric field produced
by an electric dipole is called a dipole ⃗ ⃗ ⃗
field. This can be determined by using (a)
the formula for the field of a point charge [ ]̂
and (b) the principle of superposition.
Variation of dipole field with distance. ̂
The total charge of an electric dipole is
zero. But the electric field of an electric or ⃗ ̂
dipole is not zero. This is because the
charges + q and - q are separated by Here dipole moment.
some distance, so the electric fields due to
them when added do not exactly cancel For r can be neglected
out. However, at distances much larger compared to r2.
than the dipole size (r >> 2a), the fields of ⃗ ̂ (towards
+ q and - q nearly cancel out. Hence we
expect a dipole field to fall off, at larger right)
distance, faster than 1/r2, typical of the Clearly, electric field at any axial point
field due to a single charge. In fact a of the dipole acts along the dipole axis
dipole field at larger distances falls off as from negative to positive charge i.e., in the
1/r3. direction of dipole moment .
1.24 ELECTRIC FIELD AT AN AXIAL 1.25 ELECTRIC FIELD AT AN
POINT OF A DIPOLE EQUATORIAL POINT OF A DIPOLE
37. Derive an expression for the 38. Derive an expression for the
electric field at any point on the axial line electric field at any point on the equatorial
of an electric dipole. line of an electric dipole.
Electric field at an axial point of an Electric field at an equatorial point of a
electric dipole. As shown in Fig. 1.65, dipole. As shown in Fig. 1.66, consider an
consider an electric dipole consisting of electric dipole consisting of charges -q
charges + q and - q, separated by and + q, separated by distance 2a and
distance 2 a and placed in vacuum. Let P placed in vacuum. Let P be a point on the
be a point on the axial line at distance r equatorial line of the dipole at distance r
from the centre O of the dipole on the side from it.
of the charge + q.
i.e., OP = r

Fig. 1.65 Electric field at an axial point of


dipole.
Electric field due to charge - q at point
P is
⃗ ̂ (towards left)

where ̂ is a unit vector along the dipole


axis from -q to +q.
Electric field due to charge + q at point P
is Fig. 1.66 Electric field at an equatorial
point of a dipole.

45
MITTAL INSTITUTE

Electric field at point P due to + q charge


is
⃗ , directed along BP Clearly, =2

Electric field at point P due to - q Hence the electric field of a short


charge is dipole at a distance r along its axis is twice
the electric field at the same distance
⃗ , directed along PA along the equatorial line.
1.26 TORQUE ON A DIPOLE IN A
Thus the magnitudes of ⃗ and ⃗ UNIFORM ELECTRIC FIELD
are equal i.e.,
40.Derive an expression for the torque
on an electric dipole placed in a uniform
electric field. Hence define dipole moment.
Clearly, the components of ⃗ and Torque on a dipole in a uniform
⃗ normal to the dipole axis will cancel electric field. As shown in Fig. 1.67(a),
out. The components parallel to the dipole consider an electric dipole consisting of
axis add up. The total electric field ⃗ charges +q and -q and of length 2a placed
is opposite to . in a uniform electric field ⃗ making an
ng θ with it. It h s dipo o nt o
∴⃗ ̂ magnitude,
̂ p = q × 2a
. ̂ . Force exerted on charge + q by field ⃗

= ⃗
[∵ ]
√ (along ⃗ )
or ⃗ ̂ Force exerted on charge - q by field
⃗ ⃗
where p = 2qa, is the electric dipole
moment. (opposite to ⃗ )
If the point P is located far away from ⃗ ⃗
the dipole, r >> a, then
⃗ ̂

Clearly, the direction of electric field at


any point on the equatorial line of the
dipole will be antiparallel to the dipole
moment .
39. Give a comparison of the (a)
magnitudes of electric fields of a short
dipole at axial and equatorial points.
Comparison of electric fields of a short
dipole at axial and equatorial points. The
magnitude of the electric field of a short
dipole at an axial point at distance r from
its centre is

Electric field at an equatorial point at Fig. 1.67 (a) Torque on a dipole in a


the same distance r is uniform electric field.

46
MITTAL INSTITUTE

(b) Direction of torque as given by right Dipole in a non-uniform electric field. In a


hand screw rule. non-uniform electric field, the + q and -
q charges of a dipole experience
Hence the net translating force on a
different forces (not equal and
dipole in a uniform electric field is zero.
opposite) at slightly different positions
But the two equal and opposite forces act
at different points of the dipole. They form in the field and hence a net force ⃗ acts
a couple which exerts a torque. on the dipole in a non-uniform field.
Also, a net torque acts on the dipole
Torque = Either force × Perpendicular which depends on the
distance between the two forces
location of the dipole in the non-uniform
 = qE × 2a sin θ = (q × 2a) E sin θ field.
or  = pE sin θ (p-q×2a) = × ⃗ ( )
As the direction of torque  is where is the position vector of the centre
perpendicular to both ⃗ and ⃗ , so we of the dipole.
can write
When the dipole is parallel or
 ⃗ antiparallel to ⃗ . In a non-uniform field, if
The direction of vector  is that in which a is parallel to ⃗ or antiparallel to ⃗ , the net
right handed screw would advance torque on the dipole is zero (because the
when rotated from ⃗ to ⃗ . As shown in forces on charges ±q become linear).
Fig. 1.67(b), the direction of vector  is However, there is a net force on the
perpendicular to, and points into the dipole. As shown in Fig. 1.68, when is
plane of paper. parallel to ⃗ , a net force acts on the dipole
in the direction of increasing ⃗ . When is
When the dipole is released, the torque
 tends to align the dipole with the field antiparallel to ⃗ , a net force acts in the
⃗ i.e., tends to reduce ang θ to 0. direction of decreasing ⃗ .
When the dipole gets aligned with ⃗ ,
the torque  becomes zero.
Clearly, the torque on the dipole will be
maximum when the dipole is held
perpendicular to ⃗ . Thus
max = sin 90° = pE.
Dipole moment. We know that the torque,
 = pE sin θ
If E = 1 unit, θ = 90°, then  = p
Hence dipole moment may be defined
as the torque acting on an electric dipole,
placed perpendicular to a uniform electric
field of unit strength.
1.27 DIPOLE IN A NON-UNIFORM
ELECTRIC FIELD
41. What happens when an electric
Fig. 1.68 Forces on a dipole (a) when is
dipole is held in a non-uniform electric
field ? What will be the force and the parallel to ⃗ and (b) When is
torque when the dipole is held parallel or antiparallel to ⃗ .
anti-parallel to the electric field ? Hence A comb run through dry hair attracts
explain why does a comb run through dry small pieces of paper. As the comb runs
hair attract pieces of paper ?

47
MITTAL INSTITUTE

through hair, it acquires charge due to Clearly, EA < EB < EC.


friction. When the charged comb is
 The direction of the electric field at an
brought closer to an uncharged piece of
axial point of an electric dipole is same
paper, it polarises the piece of paper i.e.,
as that of its dipole moment and at an
induces a net dipole moment in the
equatorial point it is opposite to that of
direction of the field. But the electric field
dipole moment.
due to the comb on the piece of paper is
 The strength of electric field at an axial
not uniform. It exerts a force in the
point of a short dipole is twice the
direction of increasing field i.e., the piece
strength at the same distance on the
of paper gets attracted towards the comb.
equatorial line.
42. Give the physical significance of  At larger distances, the dipole field (E
electric dipoles. 1 / r3) decreases more rapidly than
Physical significance of electric the electric field of a point charge (E
dipoles. Electric dipoles have a common 1/ r2).
occurrence in nature. A molecule Examples based on
consisting of positive and negative ions is
an electric dipole. Moreover, a Dipole Moment, Dipole Field and
complicated array of charges can be Torque on a Dipole
described and analysed in terms of Formulae Used
electric dipoles. The concept of electric
1. Dipole moment, p = q × 2a; where 2a
dipole is used (i) in the study of the effect
is the distance between the two
of electric field on an insulator, and (ii) in
charges.
the study of radiation of energy from an
antenna. 2. Dipole field at an axial point at
distance r from the centre of the dipole
For Your Knowledge
is
 In a uniform electric field, an electric
dipole experiences no net force but a
non zero torque.
 As the net force on a dipole in a When r >> a,
uniform electric field is zero, therefore, 3. Dipole field at an equatorial point at
no linear acceleration is produced. distance r from the centre of the dipole
 Torque on a dipole becomes zero when is
it aligns itself parallel to the field.
 Torque on a dipole is maximum when
it is held perpendicular to the field ⃗ .
 In a non-uniform electric field, a dipole When r >> a,
experiences a non zero force and non
zero torque. In the special case when 4. Torque,  = pE sin θ, wh r θ is th
the dipole moment is parallel or angle between and ⃗ .
antiparallel to the field, the dipole Units Used
experiences a zero torque and a non
zero force. Charge q is in coulomb, distance 2a in
 A non-uniform or specifically an metre, dipole moment p in coulomb metre
increasing E-field may be represented (Cm), field E in NC-1 or Vm-1.
by field lines as shown. Example 50. Two charges, one +5μC and
another -5μC are placed 1 mm apart.
Calculate the dipole moment.
[CBSE OD 94C]
Solution. Here q = 5 μC = 5 × 10-6 C,
Fig. 1.69 2a =1 mm = 10-3 m
Direction of increasing E-field Dipole moment,

48
MITTAL INSTITUTE

p = q × 2a = 5 × 10-6 × 10-3 = 5 × 10-9 Cm.


Example 51. An electric dipole, when held
at 30° with respect-to a uniform electric
field of 104 NC-1 experiences a torque of 9
× 10-26 Nm Calculate dipole moment of the
dipole. [CBSE D 96]
So ution. H r θ =30°, E = 104 NC-1,
Example 55. Two charges ±10μC are
 = 9 × 10-26
Nm placed 5.00 mm apart. Determine the
electric field at (a) a point P on the axis of
As  = pE sin θ the dipole 15 cm away from its centre O
∴ Dipole moment, on the side of the positive charge, (b) a
point Q, 15 cm away from O on a line
 passing through O and normal to the axis
of the dipole. [NCERT]
= 1.8 × 10-29 Cm. Solution. Here
Example 52. An electric dipole consists of mm
two opposite charges of magnitude 1 / 3 ×
cm
10-7 C, separated by 2 cm. The dipole is
placed in an external field of 3× 107 NC-1. (a) Field at the axial point P of the dipole
What maximum torque does the electric is
field exert on the dipole ?

Solution. Here q = × 10-7 C, 2a = 2 cm =
0.02 m,
NC-1
7 -1
E = 3 × 10 NC
, along ⃗⃗⃗⃗⃗ .
max = pE sin 90° = q × 2a × E × 1
This field is directed along the
= × 10-7 × 0.02 × 3 × 107 × 1 = 0.02 Nm. direction of dipole moment vector, i.e.,
from -q to +q, as shown in Fig. 1.70(a).
Example 53. Calculate the electric field
due to an electric dipole of length 10 cm
having charges of 1 μC at an equatorial
point 12 cm from the centre of the dipole.
Solution. Here q =1 μC = 10-6 C, r =12 cm
= 0.12 m, 2a = 10 cm, a = 5 cm = 0.05 m

= 4.096 × 105 NC-1.


Example 54. Two point charges, each of 5
μC but opposite in sign, are placed 4 cm
apart. Calculate the electric field intensity Fig. 1.70
at a point distant 4 cm from the midpoint
on the axial line of the dipole. [Punjab 02] (b) Field at the equatorial point Q of
the dipole is
Solution. Here
⃗⃗⃗⃗

49
MITTAL INSTITUTE

× 10-10m is situated in an electric field


of intensity 3 × 10-5 NC-1 at an angle of
30° with the field. Calculate the dipole
, along ⃗⃗⃗⃗⃗ moment and the torque acting on it.
This field is directed opposite to the Charge on an electron = 1.602 × 10-19
direction of the dipole moment vector, i.e., C.
from + q to -q, as shown in Fig. 1.70(b). [Kerala 94]
Example 56. The force experienced by a (Ans. 6.41 × 10 -29
C m, 9.615 × 10-24 Nm)
unit charge when placed at a distance of
0.10 m from the middle of an electric 4. An electric dipole is placed at an angle
dipole on its axial line is 0.025 N and of 60° with an electric field of
when it is placed at a distance of 0.2 m, magnitude 4×105 NC-1. It experiences
the force is reduced to 0.002 N. Calculate a torque of 8√ Nm. If the length of the
the dipole length. dipole is 4 cm, determine the
magnitude of either charge of the
Solution. dipole. (Ans. 10-3 C)
In first case : 5. An electric dipole consists of two
opposite charges of magnitude 2 × 10-
∴ 0.025 …(i) 6
C each and separated by a distance
of 3 cm. It is placed in an electric field
In second case :
of 2 × 103 NC-1. Determine the
∴ 0.002 …(ii) maximum torque on the dipole. (Ans.
1.2 ×10-2 N m)
Dividing (i) by (ii), we get 6. Two point h rg s o +0.2 μ μC nd -
0.2 μ μC r s p r t d by 10-8m.
Determine the electric field at an axial
point at a distance of 0.1 m from their
or idpoint. Us th st nd rd v u o ε0.
[Punjab 97]
or
(Ans. 3.6 × 10-9 NC-1)

7. Calculate the field due to an electric
Dipole length dipole of length 10 cm and consisting
Problems For Practice of charges of ±100 μC t point 20
cm from each charge.
1. An tri dipo is or d by +4μC
and -4μC charges at 5 mm distance. (Ans. 1.125 ×107 NC-1)
Calculate the dipole moment and give its HINTS
direction. [Haryana
01] 1. p = q ×2a = 4 × 10-6 × 5 × 10-3 = 2 ×10-8
Cm.
(Ans. 2 × 10“8 Cm, from -ve to +ve charge)
2.  = pE sin θ = 4 × 10-5 × 10-3 × sin 30°
2. An electric dipole of dipole moment
4 × 10-3C m is placed in a uniform electric = 2×10-8 Nm.
field of 10-13 NC-1 making an angle of 30° 3. Here q = e = 1.602 × 10-19 C, 2a = 4 ×
with the direction of the field. Determine 10-10 m,
the torque exerted by the electric field on
the dipole. E = 3 ×105 NC-1, 0 = 30°
[Haryana 02] p = q × 2a = 1.602 × 10-19 × 4 × 10-
10
(Ans. 2 × 10-8 Nm)
= 6.41 ×10-29 Cm.
3. A dipole consisting of an electron and
a proton separated by a distance of 4

50
MITTAL INSTITUTE

 = pE sin θ = 6.41 ×10-29 × 3 × 105 × sin 43. What are electric lines of force ?
30° Give their important properties.
= 9.615 ×10-24 Nm. Electric lines of force. Michael Faraday
(1791-1867) introduced the concept of
4.  = pE sin θ = q × 2a × E sin θ lines of force to visualize the nature of
 √ electric (and magnetic) fields. A small

positive charge placed in an electric field
= 10-3 C. experiences a force in a definite direction
and if it is free to move, it will start moving
5. Here q = 2 × 10-6 C, 2a = 3 cm = 3 × 10- in that direction. The path along which this
2
m, charge would move will be a line of force.
E = 2 × 105 NC-1 An electric line of force may be defined
max = p E 90° = q × 2a × E × 1 as the curve along which a small positive
charge would tend to move when free to
= 2 × 10-6 × 3 × 10-2 × 2 do so in an electric field and the tangent to
×105 which at any point gives the direction of
= 1.2 ×10-2 Nm. the electric field at that point.

6. Here r > > a In Fig. 1.72, the curve PQR is an electric


line of force. The tangent drawn to this
curve at the point P gives the direction of
the field ⃗ P at the point P. Similarly, the
tangent at the point Q gives the direction
of the field ⃗ Q at the point Q, and so on.

= 3.6 ×10-9 NC-1.


7. Here q = 100 μC = 10-4C, 2a = 10 cm =
0.10 m
p = q × 2a = 10-4 × 0.10 = 10-5 Cm

Fig. 1.72 An electric line of force.


The lines of force do not really exist, they
are imaginary curves. Yet the concept of
lines of force is very useful. Michael
Fig. 1.71 Clearly,
Faraday gave simple explanations tor
(r2 + a2)1/2 = 20cm = 0.20 m many of his discoveries (in electricity and
magnetism) in terms of such lines of force.
For Your Knowledge
The lines of force are imaginary curves,
but the field which they represent is real.

= 1.125 ×107 NC-1. The term 'lines of force' is misleading. It


will be more appropriate to call them
1.28 ELECTRIC FIELD LINES electric (or magnetic) 'field lines'.

51
MITTAL INSTITUTE

A Field line is a space curve i.e., a curve pressure on neighbouring lines of force.
in three dimensions. This explains repulsion between two
similar charges.
Properties of Electric Lines of Force
8. The relative closeness of the lines of
1. The lines of force are continuous
force gives a measure of the strength of
smooth curves without any breaks.
the electric field in any region. The lines of
2. The lines of force start at positive force are
charges and end at negative charges -
(i) close together in a strong field.
they cannot form closed loops. If there is a
single charge, then the lines of force will (ii) far apart in a weak field.
start or end at infinity.
(iii) parallel and equally spaced in a
3. The tangent to a line of force at any uniform field.
point gives the direction of the electric field
9. The lines of force do not pass through a
at that point.
conductor because the electric field inside
4. No two lines of force can cross each a charged conductor is zero.
other.
1.29 ELECTRIC FIELD LINES FOR
Reason. If they intersect, then there will DIFFERENT CHARGED CONDUCTORS
be two tangents at the point of intersection
44. Sketch and explain the field lines of (i)
(Fig. 1.73) and hence two directions of the
a positive point charge, (ii) a negative
electric Field at the same point, which is
point charge, (iii) two equal and opposite
not possible.
charges, (iv) two equal positive charges
and (v) a positively charged plane
conductor.
Electric Field lines for different charge
systems :
(i) Field lines of a positive point charge.
Fig. 1.74 shows the lines of force of an
isolated positive point charge. They are
directed radially outwards because a small
positive charge would be accelerated in
the outward direction. They extend to
infinity. The field is spherically symmetric
i.e., it looks same in all directions, as seen
from the point charge.
Fig. 1.73
5. The lines of force are always normal to
the surface of a conductor on which the
charges are in equilibrium,
Reason. If the lines of force are not
normal to the conductor, the component of
the field ⃗ parallel to the surface would
cause the electrons to move and would
set up a current on the surface. But no
current flows in the equilibrium condition. Fig. 1.74 Field lines of a positive point
charge.
6. The lines of force have a tendency to
contract lengthwise. This explains
attraction between two unlike charges.
7. The lines of force have a tendency to
expand laterally so as to exert a lateral

52
MITTAL INSTITUTE

charges. The field ⃗ is zero at the middle


point N of the join of two charges. This
point is called neutral point from which no
line of force passes. This field also has
cylindrical symmetry.

Fig. 1.75 Field lines of a negative point


charge.
(ii) Field lines of a negative point
charge. Like that of a positive point
charge, the electric field of a negative
point charge is also spherically symmetric
but the lines of force point radially inwards
as shown in Fig. 1.75. They start from
infinity. Fig. 1.77 Field lines of two equal positive
charges.
(iii) Field lines of two equal and
opposite point charges. Fig. 1.76 shows (v) Field lines of a positively
the electric lines of force of an electric charged plane conductor. Fig. 1.78
dipole i.e., a system of two equal and shows the pattern of lines of force of
opposite point charges (± q) separated by positively charged plane conductor. A
a small distance. They start from the small positive charge would tend to move
positive charge and end on the negative normally away from the plane conductor.
charge. The lines of force seem to Thus the lines of force are parallel and
contract lengthwise as if the two charges normal to the surface of the conductor.
are being pulled together. This explains They are equispaced, indicating that
attraction between two unlike charges. electric field ⃗ is uniform at all points near
The field is cylindrically symmetric the plane conductor.
about the dipole axis i.e., the field pattern
is same in all planes passing through the
dipole axis. Clearly, the electric field at all
points on the equatorial line is parallel to
the axis of the dipole.

Fig. 1.78 Field pattern of a positively


charged plane conductor.
45. What is the relation between the
density of lines of force and the electric
field strength ? Illustrate it in a diagram.
Relation between electric field strength
Fig. 1.76 Field lines of an electric dipole. and density of lines of force. Electric field
strength is proportional to the density of
(iv) Field lines of two equal and lines of force i.e., electric field strength at
positive point charges. Fig. 1.77 shows a point is proportional to the number of
the lines of force of two equal and positive lines of force cutting a unit area element
point charges. They seem to exert a placed normal to the field at that point. As
lateral pressure as if the two charges are illustrated in Fig. 1.79, the electric field at
being pushed away from each other. This P is stronger than at Q.
explains repulsion between two like

53
MITTAL INSTITUTE

i.e., ∝ .
1.30 AREA VECTOR
47. What is an area vector ? How do we
specify the direction of a planar area
vector ? How do we associate a vector to
the area of a curved surface ?
Area vector. We come across many
Fig. 1.79 Density of lines of force is
situations where we need to know not only
proportional to the electric field strength.
the magnitude of a surface area but also
field ⃗ , then the component of ⃗ normal to its direction. The direction of a planar area
ΔS wi b E os θ, so th t th tri ux vector is specified by the normal to the
is plane. In Fig. 1.81 (a), a planar area
element dS has been represented by a
∆E = Normal component of E × Surface
normal vector ⃗⃗⃗⃗ . The length of vector ⃗⃗⃗⃗
area
represents the magnitude dS of the area
= E os θ × ∆S element. If ̂ is unit vector along the
normal to the planar area, then
or ∆E = E ∆S os θ = ⃗ . ⃗⃗⃗⃗
⃗⃗⃗⃗ = dS ̂
46. Show that the 1/r2 dependence of
electric field of a point charge is consistent
with the concept of the electric field lines.
Consistency of the inverse senate law with
the electric field lines. As shown in Fig.
1.80. the number of radial lines of force
originating from a point charge q in a
given solid angle ∆Ω is constant. Consider
two points P1 and P2 at distances r1 and r2
from the charge q. The same number of
lines (say n) cut an element of area ∆Ω Fig. 1.81 (a) A planar area element. (b) An
at P1 and an element of area ΔΩ t P2. area element of a curved surface.
In case of a curved surface, we can
imagine it to be divided into a large
number of very small area elements. Each
small area element of the curved surface
can be treated as a planar area. By
convention, the direction of the vector
associated with every area element of a
closed surface is along the outward drawn
normal. As shown in Fig. 1.81(b), the area
Fig. 7.80 element ⃗⃗⃗⃗ at any point on the closed
∴ Number of lines of force cutting unit surface is equal to dS ̂, where dS is the
area element at P1 = ∆ magnitude of the area element and ̂ is a
unit vector in the direction of outward
Number of lines of force cutting unit area normal.
element at P2 = ∆ 1.31 ELECTRIC FLUX

As electric field strength ∝ Density of lines 48. Define the term electric flux. How is it
of force related to electric field intensity ? What is
its SI unit ?
∴ Electric flux. The term flux implies some
kind of flow. Flux is the property of any

54
MITTAL INSTITUTE

vector field. The electric flux is a property


of electric field.
The electric flux through a given area held
inside an electric field is the measure of
the total number of electric lines of force
passing normally through that area.
As shown in Fig. 1.82, if an electric field ⃗
passes normally through an area element Fig. 1.84 Electric flux through a closed
∆S, then the electric flux through this area surface S.
is Then the electric flux through the surface
∆ϕE = E ∆S S will be
ϕ ⃗⃗⃗⃗ ⃗ ….. ⃗

∑⃗

When the number ofarea elements


becomes infinitely large (N → ∞) nd ΔS
→ 0, the above sum approaches a surface
integral taken over the closed surface.
Thus
Fig. 1.82 Electric flux through normal area.
ϕ ∑ ⃗ ⃗ ∮ ⃗ ⃗⃗⃗⃗
As shown in Fig. 1.83, if the normal drawn ∆
to the ar nt ∆S k s n ng θ Thus the electric flux through any surface
with the uniform , open or closed, is equal to the surface
integral of the electric field ⃗ taken over
the surface .
Electric flux is a scalar quantity.
Unit of ϕ = Unit of E × unit of S
∴ SI unit of electric flux
= NC-1.m2 = Nm2C_1.
Equivalently, SI unit of electric flux
= Vm_1.m2 = Vm.
Fig. 1.83 Flux through an inclined area.
1.32 GAUSS'S THEOREM
In case the field ⃗ is non-uniform, we 49. State and prove Gauss's theorem.
consider a closed surface S lying inside
the field, as shown in Fig. 1.84. We can Gauss's theorem. This theorem gives
divide the surface S into small area a relationship between the total flux
elements : ⃗⃗⃗⃗ Δ 2, ⃗⃗⃗⃗ ,..., ⃗⃗⃗⃗ . Let the passing through any closed surface and
corresponding electric fields at these the net charge enclosed within the
surface.
elements be ⃗ ⃗ ⃗
Gauss theorem states that the total
flux through a closed surface is 1/ε0 times
the net charge enclosed by the closed
surface.
Mathematically, it can be expressed as

55
MITTAL INSTITUTE

ϕ ∮ ⃗⃗ ⃗⃗⃗⃗⃗  The net flux through a closed surface


due to a charge lying outside the
closed surface is zero.
Proof. For the sake of simplicity, we
 The charge q appearing in the Gauss's
prove Gauss's theorem for an isolated
theorem includes the sum of all the
positive point charge q. As shown in Fig.
charges located anywhere inside the
1.85, suppose the surface S is a sphere of
closed surface.
radius r centred on q. Then surface S is a
 The electric field ⃗ appearing in
Gaussian surface.
Gauss's theorem is due to all the
charges, both inside and outside the
closed surface. However, the charge q
appearing in the theorem is only
contained within the closed surface.
 Gauss's theorem is based on the
inverse square dependence on
distance contained in the coulomb's
law. In fact, it is applicable to any field
obeying inverse square law. It will not
hold in case of any departure from
inverse square law.
Fig. 1.85 Flux through a sphere enclosing
 For diu o bso ut p r ittivity ε
a point charge.
or dielectric constant , the Gauss's
Electric field at any point on S is theorem can be expressed as

∮⃗ ∮ ⃗ ⃗⃗⃗⃗

This field points radially outward at all 1.33 GAUSSIAN SURFACE


points on S. Also, any area element points
50. What is a Gaussian surface ? Give
radially outwards, so it is parallel to ⃗ , i.e.,
its importance.
θ = 0°.
Gaussian surface. Any hypothetical
∴ Flux through area ⃗⃗⃗⃗ is
closed surface enclosing a charge is
dϕ = ⃗ . ⃗⃗⃗⃗ = E dS cos 0° = EdS called the Gaussian surface of that
charge. It is chosen to evaluate the
Total flux through surface S is surface integral of the electric field
produced by the charge enclosed by it,
ϕ ∮ ∮ ∮
which, in turn, gives the total flux through
the surface.
Total area of sphere
Importance. By a clever choice of
Gaussian surface, we can easily find the
electric fields produced by certain
or ϕ symmetric charge configurations which
are otherwise quite difficult to evaluate by
This proves Gauss's theorem. the direct application of Coulomb's law
and the principle of superposition.
For Your Knowledge
1.34 COULOMB'S LAW FROM
 Gauss's theorem is valid for a closed GAUSS'S THEOREM
surface of any shape and for any
general charge distribution. 51. Deduce Coulomb's law from
 If the net charge enclosed by a closed Gauss's theorem.
surface is zero (q = 0), then flux Deduction of Coulomb's law from
through it is also zero. Gauss's theorem. As shown in Fig. 1.86,
ϕ consider an isolated positive point charge

56
MITTAL INSTITUTE

q. We select a spherical surface S of wh r θ is th ng whi h the normal to


radius r centred at charge q as the the outward drawn normal to surface area
Gaussian surface. makes with the field ⃗ .
2. According to Gauss's theorem, the
total electric flux through a closed
surface S enclosing charge q is

ϕ ∮ ⃗ ⃗⃗⃗⃗

3. Flux density =
Units Used
Fig. 1.86 Applying Gauss's theorem to a
2
point charge. Electric flux ϕE is in Nm C-1 and flux
density in NC-1.
By symmetry, ⃗ has same magnitude at
all points on S. Also ⃗ and ⃗⃗⃗⃗ at any Constant Used
point on S are directed radially Permittivity constant of free space is
outward. Hence flux through area ⃗⃗⃗⃗ is
C2 N_1m-2
⃗ ⃗⃗⃗⃗
Example 57. If ⃗ ̂ ̂ ̂ , calculate
Net flux through closed surface S is
the electric flux through a surface of area
∮⃗ ∮ ∮ 20 units in Y-Z plane. [Haryana 97]
Solution. Electric field vector, ⃗ ̂ ̂
= E × total surface area of S = E × ̂
4πr2 As the area vector in the Y-Z plane
Using Gauss's theorem, points along outward drawn normal i.e.,
along positive X-direction, so
= 20 ̂
or Flux, ⃗ ⃗ ̂ ̂ ̂ ̂

units.
or Example 58. A circular plane sheet of
radius 10 cm is placed in a uniform
The force on the point charge qQ if electric field of 5 × 105 NC-1, making an
placed on surface S will be angle of 60° with the field. Calculate
electric flux through the sheet.
Solution. Here r = 10 cm = 0.1 m, E = 5
×105 NC-1
This proves the Coulomb's law.
As the angle between the plane sheet and
Examples based on
the electric field is 60°, angle made by the
Electric Flux and Gauss's Theorem normal to the plane sheet and the electric
Formulae Used i d is θ = 90° - 60° = 30°

1. Electric flux through a plane surface Flux,


area S held in a uniform electric field ⃗ is
ϕE = ⃗ . = ES os θ Nm2 C-1.
Example 59. A cylinder is placed in a
uniform electric field ⃗ with its axis parallel

57
MITTAL INSTITUTE

to the field. Show that the total electric flux Solution, (a) By symmetry, the flux
through the cylinder is zero. through each of the six faces of the cube
will be same when charge q is placed at
Solution. The situation is shown in Fig.
its centre.
1.87.

(b) When charge q is placed at one


vertex, the flux through each of the three
faces meeting at this vertex will be zero,
as ⃗ is parallel to these faces. As only
one-eighth of the flux emerging from the
Fig. 1.87 charge q passes through the remaining
three faces of the cube, so the flux
Flux through the entire cylinder,
through each such face is
∫ ⃗ ⃗⃗⃗⃗ ∫ ⃗ ⃗⃗⃗⃗ ∫ ⃗ ⃗⃗⃗⃗
left plane right plane curved
face face surface Example 63. The electric field components
in Fig. 1.88 are Ex = a x1/2, Ey = Ez = 0, in
∫ ∫ which α = 800 N / Cm2. Calculate (i) the
∫ flux through the cube and (ii) the
∫ ∫ charge within the cube. Assume that a =
0.1 m [NCERT]

Example 60. Calculate the number of


electric lines of force originating from a
charge of 1 C.
Solution. The number of lines of force
originating from a charge of 1 C
= Electric flux through a closed
surface enclosing a charge of 1
C
Fig. 1.88
Solution. (i) The electric field is acting
only in X-direction and its Y-and Z-
Example 61 .A positive charge of 17.7 μC components are zero. For the four non-
is placed at the centre of a hollow sphere
of radius 0.5 m. Calculate the flux density shaded faces, the angle between ⃗ and
through the surface of the sphere. ∆⃗ is + π/2. So ux ϕ = ⃗ . ∆ ⃗ is zero
Solution. From Gauss's theorem, through each of these faces.
The magnitude of the electric field at
Flux, the left face is
Flux density [x = a at the left
face] Flux, ϕ ⃗⃗⃗ ⃗

Example 62. Calculdate the electric flux [θ for the left face]
through each of the six faces of a closed The magnitude of the electric field at the
cube of length l, if a charge q is placed (a) right face is
at its centre and (b) at one of its vertices.

58
MITTAL INSTITUTE

[ at the right face] The outward flux through the left face is
Flux, ⃗ ⃗
2
[θ for the right face] (0.05) .̂ ̂ .
Net flux through the cube . [ .̂ ̂ 1]
On the right face :
⃗ ̂
2
√ √ ̂ (0.05) ̂
= 1.05 Nm2 C_1. The outward flux through the right face is
(ii) By Gauss's theorem, the total = ⃗ . = +1.57 Nm2 C-1.
charge inside the cube is
(ii) For any point on the side of the
cylinder ⃗ ,
Example 64. An electric field is uniform, ∴ Flux through the side of the cylinder,
and in the positive x direction for positive x
and uniform with the same magnitude in = ⃗ . = E S cos 90° = 0.
the negative x direction for negative x. It is (iii) Net outward flux through the cylinder,
given that
= 1.57 + 1.57 + 0 = 3.14 Nm2 C-1.
⃗ = 200 ̂ NC for x > 0
-1
(iv) By Gauss's theorem, the net charge
and ⃗ = - 200 ̂ NC-1 for x < 0. inside the cylinder is
A right circular cylinder of length 20 q = ε0 = 8.854 × 10-12 × 3.14 = 2.78 ×
-11
cmand radius 5 cm has its centre at the 10 C.
origin and its axis along the x-axis so that
one face is at x = +10 cmand the other is Example 65. You are given a charge +
at x = -10 cm Qat the origin 0 (Refer to Fig. 1.90).
Consider a sphere S with centre (2, 0, 0)
(i) What is the net outward flux of radius √ m. Consider another sphere
through each flat face ? of radius √ m centered at the origin.
(ii) What is the flux through the side of Consider the spherical caps (i) PSQ (ii)
the cylinder ? PRQ (iii) PWQ, with normals outward to
the respective spheres, and (iv) the flat
(iii) What is the net outward flux through circle PTQ with normal along the x-axis.
the cylinder ?
(a) What is the sign of electric flux through
(iv) What is the net charge inside the
each of the surfaces (i)-(iv) ?
cylinder ? [NCERT]
(b) What is the relation between the
magnitudes of fluxes through surfaces
(i)-(iv) ?
(c) Calculate the flux through the surface
(ii) directly. Assume that the area of
the cap (ii) is A. [NCERT]

Fig. 1.89
Solution, (i) On the left face : ⃗ = -200 ̂
NC-1,
2
̂ (0.05)

59
MITTAL INSTITUTE

Fig. 1.90 Fig. 1.91

Solution. For the charge + Q situated Solution. The neutral coin and the
at origin O, the field ⃗ points along +vex- outside charges q4 and q5 make no
direction i.e., towards right. contribution towards the net charge
enclosed by surface S. Applying Gauss's
(a) The outward drawn normal on cap theorem, we get
PSQ points towards left while it points
towards right for caps PRQ, PWQ and ϕ
circle PTQ. So the flux is negative for
(i) and positive for the rest.
(b) The same electric field lines crossing
(i) also cross (ii), (iii). Also, by Gauss's = -666.67 Nm2 C-1.
law, the fluxes through (iii) and (iv) add
upto zero. Hence, all magnitudes of Example 67. S1 and S2 are two concentric
fluxes are equal. spheres enclosing charges Q and 2Q
respectively as shown in Fig. 1.92.
(c) Given area of the cap (ii) = A Electric
field through cap (ii) is (i) What is the ratio of the electric flux
through S1 and S2 ?
(ii) How will the electric flux through the

sphere S1 change, if a medium of
= 4.5 × 109 Q NC-1 dielectric constant K is introduced in
Electric flux through the cap (ii) is the space inside S1 in place of air ?
= EA
= 4.5 × 109 QA NC_1m2.
Example 66. Figure 1.91 shows five
charged lumps of plastic and an
electrically neutral coin. The cross-section
of a Gaussian surface S is indicated. What
is the net electric flux through the surface
if Fig. 1.92
2
q1 = q4 = + 3.1 n C, q = q5= - 5.9 nC (iii) How will the electric flux through
and q3 = -3.1 wC ? sphere S: change, if a medium of
dielectric constant K is introduced in
the space inside S2 in place of air ?
[CBSE OD 02, 14,14C]
Solution, (i) By Gauss's Theorem,
Flux through S1 is ϕ

Flux through S2 is ϕ

60
MITTAL INSTITUTE

Calculate (i) the flux through the cube, and (ii) , because the charge
(ii) the charge inside the cube. enclosed is the same as in the case (i).
[CBS
E OD 08] 7. From Gauss's theorem, total flux
through entire spherical surface is
[Ans. (i) ϕE = 0.656 Nm-2C-1 (ii) q = 5.8
×10-12 C]
11. A uniform electric field ⃗ = Ex ̂ N / C From symmetry considerations, flux
for x > 0 and ⃗ = - Ex ̂ N / C for x < 0 are through the hemispherical surface is
given. A right circular cylinder of length l
cm and radius r cm has its centre at the
origin and its axis along the x-axis. Find
out the net outward flux. Using Gauss's 8. (i) Flux through the curved surface of
law write the expression for the net charge the cylinder is zero.
within the cylinder.
Magnitude of the electric field at the
[CBSE D 08C] left face,
HINTS E = 50 × 1 = 50NC-1
1. ⃗ ̂ 4 ̂ ̂ ̂ ∴ Flux through the left face,
Flux, ⃗ ⃗ ̂ ̂ . ̂ ϕL = EScos θ = 50×25×10-4 cos 180°
= -1250 × 10-4Nm2C-1
2. ⃗ ⃗ ̂ ̂ ̂ × 105. ̂ ̂ Magnitude of the electric field at the
right face,
E = 50 × 2 = 100NC-1
. ∴ Flux through the right face,
3. (i) Normal to the area points in the ϕE = 100 × 25 × 10-4 cos 0°
direction 01 the electric field, θ = 2500 × 10-4Nm2C-1
Net flux through the cylinder,
ϕE = ϕL + ϕR = (2500-1250) ×10-4Nm2C-1
(ii) cos = 1250 × 10-4Nm2C-1
= 1.250 ×10-1Nm2C-1.
4. (i)
(ii) Total charge enclosed by the cylinder,
q = ε0ϕE = 8.854 × 10-12 × 1250 × 10-4C
(ii) = 11067.5 × 10-16C = 1.107 pC.
Ratio of electric flux through S1 and S2 is
ϕ ⁄
5. Flux through each phase of the cube
ϕ
(ii) If a medium of dielectric constant K is
introduced in the space inside S1, then flux
through S1 becomes

6. (i) ϕ ∮ ⃗ ⃗⃗⃗⃗ = ∮ ⃗⃗⃗⃗ = ∮ ⃗ ⃗⃗⃗⃗
k

61
MITTAL INSTITUTE

(iii) The flux through S1 does not change determine the electric flux due to this
with the introduction of dielectric medium charge through hemispherical surface.
inside the sphere S2. ( )
Problems For Practice
1. If the electric field is given by
⃗ = 8 ̂ + 4 ̂ + 3 ̂ NC-1. calculate the electric
flux through a surface of area 100 m2 lying
in the X-Y plane. (Ans. 300 Nm2C-1)
2. The electric field in a certain region of
space is (5 ̂ + 4 ̂ - 4 ̂ ) × 105 NC-1. Fig. 1.93
Calculate electric flux due to this field over
8. A hollow cylindrical box of length 1 m
an area of (2 ̂ - )̂ × 10-2 m2.
and area of cross-section 25 cm2 is placed
(Ans. 6 × 103 Nm2 C-1) in a three dimensional coordinate system
as shown in Fig. 1.94. The electric field in
3. Consider a uniform electric field ⃗ = 3 ×
103 ̂ NC-1. the region is given by ⃗ = 50x ,̂ where E is
in NC-1 and x is in metres.
Calculate the flux of this field through a
square surface of area 10cm2 when
(i) its plane is parallel to the y-z plane, and
(ii) the normal to its plane makes a 60°
angle with the x-axis. [CBSE D 13C]
[Ans. (i) 30Nm2C-1 (ii) 15Nm2C-1]
4. Given a uniform electric field ⃗ = 5 × 10 Fig. 1.94
̂ NC-1, find the flux of this field through a Find
square of 10 cm on a side whose plane is
parallel to the Y-Z plane. What would be (i) net flux through the cylinder,
the flux through the same square if the (ii) charge enclosed by the cylinder.
plane makes a 30° angle with the X-axis ? [CBSE D 13]
[CBSE D 14] 9. The electric field in a region is given by
[Ans. (i) 50 Nm2C-1 (ii) 25 Nm2C-1] ⃗ = .̂ Find the charge contained in the
5. A point h rg o 17.7 μC is o t d t cubical volume bounded by the surfaces x
the centre of a cube of side 0.03 m. Find = 0, x = a, y = 0, y = a, z = 0 and z = a.
the electric flux through each face of the Take E0 = 5 × 103 NC-1, a = 1 cm and b =
cube. [Himachal 93] 2 cm. (Ans. 2.2 × 10-12 C)
(Ans. 3.3 × 105 Nm2 C-1) 10. The electric field components due to a
charge inside the cube of side 0.1 m are
6. A spherical Gaussian surface encloses as shown.
a charge of 8.85 × 10-8 C. (i) Calculate the
electric flux passing through the surface, Ex = αx, wh r α = 500 N/C-m
(ii) If the radius of the Gaussian surface is Ey = 0, Ez = 0.
doubled, how would the flux change ?
[CBSE D 01, F 07]
(Ans. (i) 104 Nm2 C-1 (ii) No change]
7. A charge q is situated at the centre of
an imaginary hemispherical surface, as
shown in Fig. 1.93. Using Gauss's
theorem and symmetry considerations,

62
MITTAL INSTITUTE

52. Apply Gauss's theorem to calculate


the electric field of a thin infinitely long
straight line of charge, with a uniform
charge density of  Cm-1.
Electric field due to an infinitely long
straight charged wire. Consider a thin
infinitely long straight wire having a
uniform linear charge density  Cm-1. By
symmetry, the field ⃗ of the line charge is
directed radially outwards and its
magnitude is same at all points equidistant
from the line charge. To determine the
field at a distance r from the line charge,
Fig. 1.95 we choose a cylindrical Gaussian surface
of radius r, length l and with its axis along
the line charge. As shown in Fig. 1.97, it
has curved surface S1 and flat circular
ends S2 and S3. Obviously, ⃗⃗⃗⃗ 1 || ⃗ , ⃗⃗⃗⃗ 2 
⃗ and ⃗⃗⃗⃗ 3  ⃗ . So only the curved surface
contributes towards the total flux.

9. ϕE = ϕL + ϕR

= 0.25 Nm2 C-1.


q = ε0 ϕE = 8.85 × 10-12 × 0.25 = 2.2 × 10-12
C.

Fig. 1.97 Cylindrical Gaussian surface for


line charge.
∮ ⃗ . ⃗⃗⃗⃗ ∫ ⃗ ⃗⃗⃗⃗⃗⃗ ∫ ⃗ ⃗⃗⃗⃗⃗⃗ ∫ ⃗
⃗⃗⃗⃗⃗⃗
Fig. 1.96 ∫ ∫
11. Proceed as in Example 64 on page ∫
1.52.

(i) ϕ ( ) ( )

= 2πr2Ex(10)-4 Nm2C-1. area of the curved surface

(ii) q = ε0E = 2πr2ε0Ex(10)-4 C. or ϕ

1.35 FIELD DUE TO AN INFINITELY Charge enclosed by the Gaussian


LONG CHARGED WIRE surface,
Using Gauss's theorem, , we get

63
MITTAL INSTITUTE

or or ∴ 2EA = or E =
Thus the electric field of a line charge Clearly, E is independent of r, the
is inversely proportional to the distance distance from the plane sheet.
from the line charge.
(i) If the sheet is positively h rg d (σ >
1.36 ELECTRIC FIELD DUE TO A 0), the field is directed away from it.
UNIFORMLY CHARGED INFINITE
PLANE SHEET (ii) If the sheet is negatively h rg d (σ <
0), the field is directed towards it.
53. Apply Gauss's theorem to calculate
the electric field due to an infinite plane For a finite large planar sheet, the
sheet of charge. above formula will be approximately valid
in the middle regions of the sheet, away
Electric field due to a uniformly from its edges.
charged infinite plane sheet. As shown in
Fig. 1.98, consider a thin, infinite plane 54. Two infinite parallel planes have
sheet of charge with uniform surface uni or h rg d nsiti s o σ1 and σ2.
h rg d nsity σ. W wish to u t its Determine the electric field at points (i) to
electric field at a point P at distance r from the left of the sheets, (ii) between them,
it. and (iii) to the right of the sheets.
Electric field of two positively charged
parallel plates. Fig. 1.99 show's two thin
plane parallel sheets of charge having
uni or h rg d nsiti s σ1 nd σ2 with σ1
> σ2 > 0. Suppose ̂ is a unit vector
pointing from left to right.

Fig. 1.98 Gaussian surface for a uniformly


charged infinite plane sheet.
By symmetry, electric field E points
outwards normal to the sheet. Also, it
must have same magnitude and opposite
direction at two points P and F equidistant
from the sheet and on opposite sides. We
choose cylindrical Gaussian surface of
cross- sectional area A and length 2r with
its axis perpendicular to the sheet.
Fig. 1.99
As the lines of force are parallel to the
curved surface of the cylinder, the flux In the region I : Fields due to the two
through the curved surface is zero. The sheets are
flux through the plane-end faces of the ⃗⃗⃗⃗ ⃗⃗⃗⃗
̂ ̂
cylinder is
= EA+ EA=2 EA From the principle of superposition, the
total electric field at any point of region is
Charge enclosed by the Gaussian
surface, ̂
⃗⃗⃗⃗ ⃗⃗⃗⃗ ⃗⃗⃗⃗
Q = σA
In the region II: Fields due to the two
According to Gauss's theorem,
sheets are
⃗⃗⃗⃗ ̂ ⃗⃗⃗⃗ ̂

64
MITTAL INSTITUTE

̂
∴ Total field, ⃗⃗⃗⃗⃗ ∴ Total field, ⃗

In the region III: Fields due to the two Thus the electric field between two
sheets are oppositely charged plates of equal charge
density is uniform which is equal to and
⃗⃗⃗⃗ ̂ ⃗⃗⃗⃗ ̂ is directed from the positive to the
̂
negative plate, while the field is zero on
∴ Total field, ⃗ the outside of the two sheets. This
arrangement is used for producing uniform
55. Two infinite parallel planes have electric field.
uniform charge densities ± σ. Determine
the electric field in (i) the region between 1.37 FIELD DUE TO A UNIFORMLY
the planes, and (ii) outside it. CHARGED THIN SPHERICAL SHELL

Electric field of two oppositely charged 56. Apply Gauss's theorem to show
plane parallel plates. As shown in Fig. that for a spherical shell, the electric field
1.100, consider two plane parallel sheets inside the shell vanishes, whereas outside
having uniform surface charge densities of it, the field is as if all the charge had been
± σ. Suppose ̂ be a unit vector pointing concentrated at the centre.
from left to right. Electric field due to a uniformly
charged thin spherical shell. Consider a
thin spherical shell of charge of radius R
with uniform surface charge density σ.
From symmetry, we see that the electric
field ⃗ at any point is radial and has same
magnitude at points equidistant from the
centre of the shell i.e., the field is
spherically symmetric. To determine
electric field at any point P at a distance r
from O, we choose a concentric sphere of
radius r as the Gaussian surface.

Fig. 1.100
In the region I : Fields due to the two
sheets are
̂ ̂
⃗⃗⃗⃗ ⃗⃗⃗⃗

Total field,
̂ ̂
⃗⃗⃗⃗ ⃗⃗⃗⃗ ⃗⃗⃗⃗

In the region II: Fields due to the two


sheets are
Fig. 1.101 Gaussian surface for outside
̂ ̂ points of a thin spherical shell of
⃗⃗⃗⃗ ⃗⃗⃗⃗
charge.
̂ ̂
∴ Total field, ⃗⃗⃗⃗⃗ ̂ (a) When point P lies outside the
spherical shell. The total charge q inside
In the region III: Fields due to the two the Gaussian surface is the charge on the
sheets are shell of radius R nd r 4πR2.
̂ ̂ ∴ q = 4πR2 σ

65
MITTAL INSTITUTE

Flux through the Gaussian surface, ϕE = E or E=0 [For r <


× 4 πr2 R]
By Gauss's theorem, Hence electric field due to a uniformly
charged spherical shell is zero at all points
ϕE =
inside the shell.
∴ E × 4πr2 = Figure 1.102(b) shows how E varies
with distance r from the centre of the shell
or E= [For r > of radius R. E is zero from r = 0 to r = R ;
R] and beyond r = R, we have
This field is the same as that produced ∝
by a charge q placed at the centre O.
Hence for points outside the shell, the field
due to a uniformly charged shell is as if
the entire charge of the shell is
concentrated at its centre.
(b) When point P lies on the spherical
shell. The Gaussian surface just encloses
the charged spherical shell.
Applying Gauss's theorem,
E × 4πR2 =

or E= [For r = R] Fig. 1.102 (b) Variation of E with r for a


spherical shell of charge.
or E= [∵ q = 4πR2 σ]

(c) When point P lies inside the


spherical shell. As is clear from Fig.
1.102(a), the charge enclosed by the
Gaussian surface is zero, i.e.,
q=0

Fig. 1.103 (b) Gaussian surface for inside


points of an insulating sphere of
charge.

Examples based on
Applications of Gauss's Theorem
Fig. 1.102 (a) Gaussian surface for inside
Formulae Used
points of a thin spherical shell of
charge. 1. Electric field of a long straight wire of
uniform linear charge density
Flux through the Gaussian surface,
<lemda>,
ϕE = E × 4πr2 
E=
Applying Gauss's theorem,
where r is the perpendicular distance of
the observation point from the wire.
E × 4πr2 = 0 2. Electric field of an infinite plane sheet of
uni or sur h rg d nsity σ,

66
MITTAL INSTITUTE

E= f = E × charge on unit length of wire 2 =


E2
3. Electric field of two positively charged  
parallel plates with charg d nsiti s σ1 or f= .
nd σ2 such that σ1 > σ2 > 0.
Example 69. An electric dipole consists of
E=± (σ1 + σ2) (Outside the plates) charges ±2 × 10-8C, separated by a
distance of 2 mm. It is placed near a long
E= (σ1 - σ2) (Inside the plates) line charge of density 4.0 × 10-4 Cm-1, as
shown in Fig. 1.104, such that the
4. Electric field of two equally and negative charge is at a distance of 2 cm
oppositely charged parallel plates, from the line charge. Calculate the force
acting on the dipole.
E= 0 (For outside points)
E= (For inside points)

5. Electric field of a thin spherical shell of


h rg d nsity σ nd r dius R,
E= For r > R (Outside points)

E= 0 For r < R (Inside points)


E= For r = R (At the surface)

Here q = 4π R2 σ.
6. Electric field of a solid sphere of uniform Fig. 1.104
charge density p and radius R : Solution. Electric field due to a line
E= For r > R (Outside points) charge at distance r from it,

E= For r < R (Inside points)

E= For r = R (At the surface) Force exerted by this field on charge q,



3
Here q = π R
Units Used Force exerted on negative charge (r =
0.02 m),
Here charges are in coulomb, r and R in
metre,  in Cm-1, σ in Cm-2, in Cm-3 and
electric field E in NC-1 or Vm-1.
Example 68. Two long straight parallel = 7.2 N, acting towards the line charge
wires carry charges 1 and 2 per unit Force exerted on positive charge (r = 2.2
length. The separation between their axes × 10-2 m),
is d. Find the magnitude of the force
exerted on unit length of one due to the
charge on the other.
Solution. Electric field at the location of = 6.5 N, acting away from the line
wire 2 due to charge on 1 is charge
 Net force on the dipole,
E=
F = F1 - F2 = 7.2 - 6.5
Force per unit length of wire 2 due to
the above field = 0.7 N, acting towards the line charge.

67
MITTAL INSTITUTE

Example 70. (a) An infinitely long


positively charged wire has a linear
charge density  Cm-1. An electron is
revolving around the wire as its centre
with a constant velocity in a circular plane Example 72. A charged particle having a
perpendicular to the wire. Deduce the charge of -2.0 × 10-6 C is placed close to a
expression for its kinetic energy, (b) Plot a non-conducting plate having a surface
graph of the kinetic energy as a function of charge density of 4.0 × 10-6 Cm-2. Find the
charge density . [CBSE F 13] force of attraction between the particle
and the plate.
Solution. The electrostatic force
exerted by the line charge on the electron Solution. Here q = -2.0× 10-6C,
provides the centripetal force for the σ = 4.0 × 10-6Cm-2
revolution of electron.
Field produced by charged plate,
∴ Force exerted by electric field =
Centripetal force

eE = Force of attraction between the


charged particle and the plate,
Here v is the orbital velocity of the
electron ∴

But E = = 0.45 N.
  Example 73. A particle of mass 9 × 10-5g
∴ or
is kept over a large horizontal sheet of
Kinetic energy of the electron will be charge density 5 × 10-5 Cm-2. What charge
should be given to the particle, so that if
 released, it does not fall ?
Solution. Here m = 9 × 10-5g = 9 × 10-8 kg,
(b) As Ek  , the graph of kinetic
σ = 5 × 10-5 Cm-2
energy Ek vs. charge density  will be a
straight line as shown in Fig. 1.105. The particle must be given a positive
charge q. It will not fall if
Upward force exerted on the = Weight of
the particle particle by electric field
or qE = mg
or
Fig. 1.105
or
Example 71. A charge of 17.7 × 10-4 C is
distributed uniformly over a large sheet of
area 200 m2. Calculate the electric field
intensity at a distance of 20 cm from it in
= 3.12 × 10-13 C.
air.
Example 74. A large plane sheet of
[CBSE OD 03C]
charge having surface charge density 5.0
Solution. Surface charge density of the × 10-16 Cm-2 lies in the X-Y plane. Find the
sheet, electric flux through a circular area of
radius 0.1 m, if the normal to the circular
area makes an angle of 60° with the Z-
axis.
Electric field at a distance of 20 cm from it Given that: ε0 = 8.85 × 10-12 C2 N-1m-2.
in air,

68
MITTAL INSTITUTE

So ution. H r σ = 5.0 × 10-16 Cm-2, r = (Ans. 1800 Vm-1)


0.1 , θ = 60°
3. An infinitely long wire is stretched
Field due to a plane sheet of charge, horizontally 4 metre above the surface
of the earth. It carries a charge 1μC
E= per cm of its length. Calculate its
electric field at a point on the earth's
Flux through circular area,
surface vertically below the wire.
ϕ (Ans. 4.5 × 103 Vm-1)
4. Two large metal plates each of area 1
m2 are placed facing each other at a
distance of 10 cm and carry equal and
opposite charges on their faces. If the
electric field between the plates is 100
Example 75. A spherical conductor of NC-1 find the charge on each plate.
radius 12 cm has a charge of 1.6 × 10-7 C
distributed uniformly over its surface. (Ans. 8.85 ×10-10 C)
What is the electric field (i) inside the 5. An electron is revolving around a long
sphere, (ii) just outside the sphere, (Hi) at line charge having charge density 2 ×
a point 18 cm from the centre of the 10-8Cm-1. Find the kinetic energy of the
sphere ? [NCERT] electron, assuming that it is
Solution. Here q = 1.6 × 10-7 C, independent of the radius of electron's
orbit.
R = 12 cm = 0.12 m
(Ans. 2.88×10_17J)
(i) Inside the sphere, E = 0. This is
because the charge resides on the outer 6. A particle of mass 5 × 10-6g is kept
surface of the spherical conductor. over a large horizontal sheet of charge
density 4×10-6Cm-2. What charge
(ii) Just outside the sphere, r = R = should be given to this particle, so that
0.12 m. Here the charge may be assumed if released, it does not fall down. How
to be concentrated at the centre of the many electrons should be removed to
sphere. give this charge ?
∴ (Ans. 2.16 ×10-13C, 1.355 × 106)
7. A spherical shell of metal has a radius
of 0.25 m and carries a charge of 0.2
μC. C u t th tri i d int nsity
(iii) At a point 18 cm from the centre, r = at a point (i) inside the shell, (ii) just
18 cm = 0.18 m. outside the shell and (iii) 3.0 m from
the centre of the shell. [Ans. (i) 0 (ii)
∴ 2.88 × 104 NC-1 (iii) 200 NC-1]
= 4.44 × 104 NC-1. HINTS
Problems For Practice 1.
1. An infinite line charge produces a field Cm-1.
of
2. Here ,
9×104NC-1 at a distance of 4 cm. Calculate
the linear charge density. [Haryana 01] ∴
-7 -1
(Ans. 2 × 10 Cm ) Vm-1
2. A cylinder of large length carries a
charge of 2 ×10-8Cm-1. Find the 3. .
electric field at a distance of 0.2 m
4.
from it.

69
MITTAL INSTITUTE

∴ C.
Number of electrons required to be
5. From Example 70, removed,
-6
× 10 .

J. 7. (i) Electric field at any point inside the


shell = 0.
6. Upward electric force on particle
(ii)
= Weight of the particle

or (iii)

VERY SHORT ANSWER CONCEPTUAL PROBLEMS


Problem 1. The electric charge of any Solution. No. The positive charge of a
body is actually a surplus or deficit of body is due to deficit of electrons while the
electrons. Why not protons ? negative charge is due to surplus of
electrons. Hence the mass of the
Solution. Electrons are loosely bound
negatively charged sphere will be slightly
to atoms and can be readily exchanged
more than that of the positively charged
during rubbing. Protons are firmly bound
spheres.
inside the nucleus. They cannot be easily
detached. Hence electric charge of any Problem 5. A positively charged rod
body is just a surplus or deficit of electrons repels a suspended object. Can we
and not protons. conclude that the object is positively
charged ?
Problem 2. When a glass rod is
rubbed with silk, both acquire charges. Solution. Yes, the object is positively
What is the source of their electrification ? charged. Repulsion is the surest test of
electrification.
Solution. For the electrification of a
body, only electrons are responsible. Problem 6. A positively charged rod
During rubbing electrons are transferred attracts a suspended object. Can we
from glass rod to silk. The glass rod conclude that the object is negatively
acquires a positive charge and silk charged ?
acquires an equal negative charge.
Solution. No. A positively charged rod
Problem 3. Is the mass of a body can attract both a neutral object and a less
affected on charging ? positively charged object.
Solution. Yes. Electrons have a Problem 7. How does a positively
definite mass. The mass of a body slightly charged glass rod attract a neutral piece
increases if it gains electrons while the of paper ?
mass decreases if the body loses
Solution. The positively charged rod
electrons.
induces negative charge on the closer end
Problem 4. Two identical metallic and positive charge on the
spheres of exactly equal masses are
taken. One is given a positive charge q
coulombs and other an equal negative
charge. Are their masses after charging
equal ? [IIT]

70
MITTAL INSTITUTE

Problem 12. A comb run through one's


dry hair attracts small bits of paper. Why ?
What happens if the hair is wet or if it is a
rainy day ? [NCERT]
Solution. When the comb runs through
dry hair, it gets charged by friction. The
Fig. 1.106 molecules in the paper get polarized by
the charged comb, resulting in a net force
farther end of the paper. The rod exerts of attraction. If the hair is wet, or if it is
greater attraction than repulsion on the rainy day, friction between hair and the
paper because negative charge is closer comb reduces. The comb does not get
to the rod than the positive charge. Hence charged and thus it will not attract small
the rod attracts the piece of paper. bits of paper.
Problem 8. Can two like charges Problem 13. Ordinary rubber is an
attract each other ? If yes, how ? insulator. But the special rubber tyres of
Solution. Yes. If one charge is larger aircrafts are made slightly conducting.
than the other, the larger charge induces Why is this necessary ? [NCERT]
equal and opposite charge on the nearer Solution. During landing, the tyres of
end of the body with smaller charge. The aircraft may get highly charged due to
opposite induced charge is larger than the friction between tyres and the air strip. If
small charge initially present on it. the tyres are made slightly conducting,
Problem 9. Why do the gramophone they will lose the charge to the earth
records get covered with dust easily ? otherwise too much of static electricity
accumulated may produce spark and
Solution. The gramophone records get result in fire.
charged due to the rubbing action of the
needle. So they attract the dust particles Problem 14. Vehicles carrying
from the air. inflammable materials usually have
metallic ropes touching the ground during
Problem 10. An ebonite rod held in motion. Why ? [Himanchal 98 ; Punjab 99
hand can be charged by rubbing with ; NCERT]
flannel but a copper rod cannot be
charged like this. Why ? [Himachal 97] Solution. Moving vehicle gets charged
due to friction. The inflammable material
Solution. Ebonite rod is insulating. may catch fire due to the spark produced
Whatever charge appears on it due to by charged vehicle. When metallic rope is
rubbing, stays on it. Copper is good used, the charge developed on the vehicle
conductor. Any charge developed on it is transferred to the ground and so the fire
flows to the earth through our body. So is prevented.
copper rod cannot be charged like this. It
can be charged by providing it a plastic or Problem 15. An inflated balloon is
rubber handle. charged by rubbing with fur. Will it stick
readily to a conducting wall or to an
Problem 11. Electrostatic experiments insulating wall ? Give reason. [Roorkee]
do not work well on humid days. Give
reason. Solution. It will stick readily to the
conducting wall. It induces an equal
Solution. Electrostatic experiments amount of charge on the conducting wall
require accumulation of charges. and much smaller charge on insulating
Whatever charges appear during the wall. So a huge force of attraction acts
experimentation, they are drained away between the balloon and the conducting
through humid air which is more wall.
conducting than dry air due to the
presence of a larger number of charged Problem 16. A metal sphere is fixed on
particles in it. a smooth horizontal insulating plate.
Another metal sphere is placed a small

71
MITTAL INSTITUTE

distance away. If the fixed sphere is given


a charge, how will the other sphere react ?
Solution. The charge on the fixed This is not an integer. So a body
sphere induces unlike charge at the closer cannot have a charge of 0.8 × 10-19 C.
end and like charge on the far end of the
Problem 22. If the distance between
free sphere. Net attraction acts on the free
sphere and so it gets accelerated towards two equal point charges is doubled and
the fixed sphere. their individual charges are also doubled,
what would happen to the force between
Problem 17. Is there some way of them ?
producing high voltage on your body
without getting a shock ? Solution. The original force between
the two charges is
Solution. If we stand on an insulating
surface and touch the live wire of a high
power supply, a high potential is
developed on our body, without causing When the individual charges and the
any shock. distance between them are doubled, the
Problem 18. A charged rod attracts force becomes
bits of dry cork which after touching the
rod, often jump away from it violently. Why
?
Hence the force will remain same.
Solution. The charged rod attracts the
bits of dry cork by inducing unlike charge Problem 23. The electrostatic force
at their near ends and like charge at their between two charges is a central force.
far ends. When the cork bits touch the rod, Why ?
they share the charge of the rod of the Solution. The electrostatic force
same sign and so get strongly repelled between two charges acts along the line
away. joining the two charges. So it is a central
Problem 19. What does q1 + q2 = 0 force.
signify in electrostatics ? [CBSE OD 01C] Problem 24. How is the Coulomb force
Or between two charges affected by the
presence of a third charge ?
Two charges q1 and q2, separated by a
small distance satisfy the equation q1 + q2 Solution. The Coulomb force between
= 0. What does it tell about the charges ? two charges does not depend on the
[CBSE F 03] presence of a third charge.
Solution. The equation signifies that Problem 25. Two equal balls having
the electric charges are algebraically equal positive charge ‘q’ coulombs are
additive and here q1 and q2 are equal and suspended by two insulating strings of
opposite. equal length. What would be the effect on
the force when a plastic sheet is inserted
Problem 20. Name the experiment between the two ? [CBSE OD 14]
which established the quantum nature of
electric charge. [CBSE OD 98] Solution. The force between the two
balls decreases because (Plastic) > 1
Solution. Millikan's oil drop experiment
and F 1/ .
for determining electronic charge.
Problem 26. Force between two point
Problem 21. Can a body have a
charges kept at a distant d apart in air is
charge of 0.8 × 10-19 C ? Justify your
F. If these charges are kept at the same
answer by comment ? [Himachal 99C]
distance in water, how does the electric
Solution. The charge on any body is force between them change ? [CBSE OD
always an integral multiple of e. Here 11]

72
MITTAL INSTITUTE

Solution. Dielectric constant for water, Solution. The positive charge induced
= 80 on the neck of the tube will accelerate the
electron towards the neck.

Problem 31. Why should a test charge
Thus the force in water is 1/ 80 times be of negligibly small magnitude ?
the original force in air. Solution. The magnitude of the test
Problem 27. The dielectric constant of charge must be small enough so that it
water is 80. What is its permittivity ? does not disturb the distribution of the
[Haryana 97C] charges whose electric field we wish to
measure otherwise the measured field will
Solution. Dielectric constant, = be different from the actual field.
∴ P r ittivity, ε = ε0 = 8.854 × 10-12 × 80 Problem 32. In defining electric field
due to a point charge, the test charge has
= 7.083 ×10-10C2N-1m-2. to be vanishingly small. How this condition
Problem 28. Give an example to can be justified, when we know that
illustrate that electrostatic forces are much charge less than that on an electron or a
stronger than gravitational forces. proton is not possible ?
Solution. A charged glass rod can lift a Solution. Because of charge
piece of paper against the gravitational quantisation, the test charge q0 cannot go
pull of the earth on this piece. This shows below e. However, in macroscopic
that the electrostatic force on the piece of situations, the source charge is much
paper is much greater than the larger than the charge on an electron or
gravitational force on it. proton, so the limit q0 → 0 for the test
charge is justified.
Problem 29. Two electrically charged
particles, having charges of different Problem 33. What is the advantage of
magnitude, when placed at a distance 'd' introducing the concept of electric field ?
from each other, experience a force of Solution. By knowing the electrical
attraction 'F'. These two particles are put field at a point, the force on a charge
in contact and again placed at the same placed at that point can be determined.
distance from each other.
Problem 34. How do charges interact
What is the nature of new force ?
between them ?
Solution. The electric field of one
Is the magnitude of the force of charge exerts a force on the other charge
interaction between them now more or and vice versa.
less than F ? [CBSE Sample Paper 11]
Charge Electric field Charge.
Solution. When the two particles are
put in contact, they share the difference of Problem 35. An electron and a proton
charge identically. Hence the two particles are kept in the same electric field. Will
repel, with a force less than F. they experience same force and have
same acceleration ?
Problem 30. An electron moves along
a metal tube with variable cross-section, Solution. Both electron and proton will
as shown in Fig. 1.107. How will its experience force of same magnitude, F =
velocity change when it approaches the eE. Since a proton has 1836 times more
neck of the tube ? mass than an electron, so its acceleration
will be 1/1836 times that of the electron.
Problem 36. Why direction of an
electric field is taken outward (away) for a
positive charge and inward (towards) for a
Fig. 1.107 negative charge ?

73
MITTAL INSTITUTE

Solution. By convention, the direction Alternatively, electrostatic field is a


of electric field is the same as that of force conservative field. The work done in
on a unit positive charge. As this force is moving a charge along a closed path must
outward in the field of a positive charge, be zero. Hence, electrostatic field lines
and inward in the field of a negative cannot form closed loops.
charge, so the directions are taken
Problem 42. Do (he electric lines of
accordingly.
force really exist ? What is about the field
Problem 37. A charged particle is free they represent ?
to move in an electric field. Will it always
Solution. Lines of force do not really
move along an electric field ? [IIT]
exist. These are hypothetical curves used
Solution. The tangent at any point to to represent an electric field. But the
the line of force gives the direction of electric field which they represent is real.
electric field and hence of force on a
Problem 43. Draw lines of force to
charge at that point. If the charged particle
represent a uniform electric field. [CBSE
starts from rest, it will move along the line
OD 95]
of force. If it is in motion and moves
initially at an angle with the line of force, Solution. The lines of force of a uniform
then resultant path is not along the line of electric field are equidistant Uniform
force. electric field, parallel lines as shown in
Fig. 1.108.
Problem 38. A small test charge is
released at rest at a point in an
electrostatic field configuration. Will it
travel along the line of force ? [NCERT]
Solution. Not necessarily. The test
charge will move along the line of force
only if it is a straight line. This is because
Fig. 1.108 Uniform electric field.
a line of force gives the direction of
acceleration and not that of velocity. Problem 44. Fig. 1.109 shows electric
lines of force due to point charges q1 and
Problem 39. Why do charges reside
q2 placed at points A and B respectively.
on the surface of the conductor ?
Write the nature of charge on them.
Solution. Charges lie at the ends of [CBSE F 03]
lines of force. These lines of force have a
tendency to contract in length. The lines of
force pull charges from inside a conductor
to its outer surface.
Problem 40. Why is electric field zero
inside a charged conductor ?
Solution. This is because charges Fig. 1.109
reside on the surface of a conductor and Solution. As the lines of force are
not inside it. pointing towards a. as well as so both q1
Problem 41. Why do the electrostatic and q2 must be negative charges.
field lines not form closed loops ? [CBSE Problem 45. A positive point charge
OD 14,15] (+Q) is kept in the vicinity of an uncharged
Solution. Electrostatic field lines start conducting plate. Sketch electric field lines
from a positive charge and end on a originating from the point charge on to the
negative charge or they fade out at infinity surface of the plate. [CBSE OD 09,17C]
in case of isolated charges without Solution. Starting from the charge +Q,
forming any closed loop. the lines of force will terminate at the
metal plate, inducing negative charge on
it. At all positions, the lines of force will be

74
MITTAL INSTITUTE

perpendicular to the metal surface, as


shown in Fig. 1.110.

Fig. 1.112
Problem 48. In the electric field shown
Fig. 1.110 in Fig. 1.113, the electric field lines on the
Problem 46. Why is it necessary that left have twice the separation as that
the field lines from a point charge placed between those on the right. If the
in the vicinity of a conductor must be magnitude of the field at point A is 40 NC-
1
normal to the conductor at every point. , calculate the force experienced by a
[CBSE F 09] proton placed at point A Also find the
magnitude of electric field at point B.
Solution. If the field lines are not normal,
then the field ⃗ would have a
tangential component which will make
electrons move along the surface
creating surface currents and the
conductor will not be in equilibrium.
Problem 47. Fig. 1.111 shows two large
metal plates, P1 and P2, tightly held Fig. 1.113
against each other and placed between
Solution. Force on proton at point A,
two equal and unlike point charges
perpendicular to the line joining them. F = eEA = 1.6 × 10-19 × 40 = 6.4 ×10-18
N As the separation between the lines of
(i) What will happen to the plates when
force at point B is twice of that at point A,
they are released ?
so
(ii) Draw the pattern of the electric field
lines for the system. [CBSE F 09] .
Problem 49. The electric lines of force
tend to contract lengthwise and expand
laterally. What do they indicate ?
Solution. The lengthwise contraction
indicates attraction between unlike
charges while lateral expansion indicates
repulsion between like charges.
Problem 50. A point charge placed at
any point on the axis of an electric dipole
Fig. 1.111 at some large distance experiences a
Solution. force F. What will be the force acting on
the point charge when its distance from
(i) When released, the two plates tend to
the dipole is doubled? [CPMT91]
move apart slightly due to the charges
induced in them. Solution. At any axial point of a dipole,
electric field varies as
(ii) The pattern of the electric field lines for
the system is shown in Fig. 1.112.
∝ ∝

75
MITTAL INSTITUTE

∴ When the distance of the point Fig. 1.115


charge is doubled, the force reduces to F / surface 'S'. What is the electric flux due to
8. this configuration through the surface 'S' ?
Problem 51. Consider the situation [CBSE D 10]
shown in Fig. 1.114. What are the signs of Solution.
q1 and q2 ? If the lines are drawn in
proportion to the charge, what is the ratio
q1/q2?
Solution. Here q1 is a negative charge
and q2 is a positive charge.
Problem 55. Two charges of magnitudes -
2Q and +Q are located at points (a, 0) and
(4a, 0) respectively. What is the electric
= 1:3. flux due to these charges through a
sphere of radius '3a' with its centre at the
origin ? [CBSE OD 13]
Solution. Only the charge -2 Q is
enclosed by the sphere of radius 3a. By
Gauss's theorem.

Fig. 1.114
Problem 56. A point charge +Q is placed
Problem 52. An arbitrary surface
at the centre O of an uncharged hollow
encloses a dipole. What is the electric flux
spherical conductor of inner radius 'a' and
through this surface ? [Exemplar Problem]
outer radius 'b'. Find the following :
Solution. As the total charge of a
(a) The magnitude and sign of the charge
dipole is zero, so by Gauss's theorem, the
induced on the inner and outer
electric flux through the closed surface is
surfaces of the conducting shell.
zero.
(b) The magnitude of electric field vector
Problem 53. The force on an electron
kept in an electric field in a particular at a distance (i) r = , and (ii) r = 2b, from
direction is F. What will be the magnitude the centre of the shell.
and direction of the force experienced by [CBSE SP 18]
a proton at the same point in the field ?
Mass of the proton is 1836 times the mass
of the electron. [CBSE F07]
Solution. A proton has charge equal
and opposite to that of an electron. Hence
the proton will experience a force equal
and opposite to that of F.
Problem 54. Figure 1.115 shows three
charges + 2q, -q and + 3q. Two charges + Fig. 1.116
2q and -q are enclosed within a Solution, (a) The charge +Q at the
centre induces charge - Q on the inner
surface of the shell and charge + Q on the
outer surface of the shell.
(b) (i) Imagine a concentric spherical
surface of radius r = as the Gaussian
surface. By symmetry, E will have same

76
MITTAL INSTITUTE

magnitude at all points on this surface and ϕE = or E × 4πr2 =


will point radially outwards.
∴ Flux through the Gaussian surface, ∴

ϕE = E × 4πr2 (ii) For outside points like r = 2b, the


Charge enclosed by the Gaussian field is similar to that of a point charge.
surface = + Q.
By Gauss's theorem,
SHORT ANSWER CONCEPTUAL PROBLEMS
Problem 1. Five balls, numbered 1 to 5
are suspended using separate threads.
Pairs (1,2), (2,4), (4,1) show electrostatic
attraction, while pairs (2,3) and (4, 5)
show repulsion. What is the nature of
charge on ball 1 ?
Solution. Repulsion is the surest test
of electrification. So balls 2,3,4 and 5 are
electrically charged. Pairs (2,3) and (4, 5)
are similarly charged. As pair (2,4) shows
attraction, so balls 2 and 4 are oppositely
charged. If balls (2, 3) are positively
charged, then (4, 5) be negatively charged
or vice versa. The ball 1 is attracted both
Fig. 1.117
by 2 and 4 separately i.e., by both kinds of
charges. Hence the ball 1 is neutral. Solution. Let the original charge on
sphere A be q and that on B be q'. At a
Problem 2. A charged metallic sphere
distance r between their centres, the
A is suspended by a nylon thread. Another
magnitude of the electrostatic force on
charged metallic sphere B carried by an
each is given by
insulating handle is brought close to A
such that the distance between their
centres is 10 cm as shown in Fig.
1.117(a). The resulting repulsion of A is
noted (for example, by shining a beam of neglecting the sizes of spheres A and B in
light and measuring the deflection of its comparison to r. When an identical but
shadow on a calibrated screen). Spheres uncharged sphere C touches A, the
A and B are touched by uncharged charges redistribute on A and C and, by
spheres C and D, respectively as shown symmetry, each sphere carries a charge
in Fig. 1.117(b). C and D are then (q / 2). Similarly, after D touches B, the
removed and B is brought closer to A to a redistributed charge on each is (q’/2). If
distance of 5.0 cm between their centres, now the separation between A and B is
as shown in Fig. 1.117(c). What is the halved, the magnitude of the electrostatic
expected repulsion of A on the basis of force on each is
Coulomb's law ? Spheres A and C and
spheres B and D have identical sizes.
Ignore the sizes of A and B in comparison
to the separation between their centres. Thus the electrostatic force on A, due
to B, remains unaltered.
[NCERT]
Problem 3. Compare electrostatic and
gravitational interactions.
Solution.Similarities between electrostatic and gravitationa

77
MITTAL INSTITUTE

1. Both forces act according to similar (i) The tangent at any point on the curve
laws : gives the direction of the electric field
at that point.
and
(ii) The relative closeness of the lines of
2. Both are conservative forces. force indicates the relative strength of
electric field at different points.
3. Both are central forces.
Problem 6. Represent the surface
Dissimilarities between electrostatic and
distribution of charge for a square metal
gravitational interactions :
plate by using dashes in such a way that
1. Electrostatic interactions may be the greater the surface density of charge,
attractive or repulsive while the farther away are the dashes from the
gravitational interactions are always plate.
attractive.
Solution. The distribution of surface
2. Electrostatic interactions depend on charge density on a square metal plate is
the nature of the medium while as shown in Fig. 1.118. As the surface
gravitational interactions do not charge density is proportional to the
depend on the nature of the medium. curvature and curvature is maximum at
the comers and zero at plane surface, so
3. Electrostatic interactions are much
dashes are equidistant from the straight
stronger than gravitational interactions.
portion and far-away from the corners.
Problem 4. Distinguish between
electric charge and mass.
Solution.

Electric Charge Mass

1. Electric charge can Mass is always


be positive, positive.
negative or zero.
Quantisation of
2. Electric charge is mass is yet not Fig. 1.118
always quantised. established. Problem 7. Two point electric charges
3. Charge on a body Mass of a body of unknown magnitude and sign are
does not change increases with its placed a distance 'd' apart. The electric
with its speed. speed. field intensity is zero at a point, not
between the charges but on the line
4. Electric charge is Mass is not
joining them. Write the essential
always conserved. conserved by itself
conditions for this to happen. [CBSE D 97]
as it can be
changed into Solution. (i) The two charges must
energy and vice have opposite signs.
versa. (ii) The magnitude of the charge lying
near the point of zero electric field
Problem 5. What is an electric line of intensity must be smaller than the
force ? What is its importance ? [Punjab magnitude of the other charge.
98C]
Problem 8. The electric field E due to a
Solution. An electric line of force may point charge at any point is defined as
be defined as the path straight or curved,
along which a unit positive charge would , where q is the test charge and F
tend to move if free to do so. is the force acting on it. What is the
physical significance of in this field
Importance :
expression ? Draw the electric lines of a

78
MITTAL INSTITUTE

point charge Q when (i) Q > 0 and (ii) Q <


* +
0. [CBSE D 07]
Solution. The indicates that the test The charge Q will be in equilibrium if
charge q is small enough so that its the force exerted by q3 is equal and
presence does not affect the opposite to the combined force exerted by
distribution of source charge and q1 and q2.
hence does not change the value of ∴ Force exerted by q3 on Q = - F
electric field which we wish to
measure. * +
For electric lines of force of point
charge Q, see Fig. 1.74 and Fig. 1.75. ⃗⃗⃗
* +
Problem 9. A charge Q located at a point
is in equilibrium under the combined The above vector gives the direction of
electric field of three charges q1, q2, q3. If the force on Q due to q3.
the charges q1, q2 are located at points Problem 10. Two point charges + q
⃗⃗⃗ and ⃗⃗⃗ respectively, find the direction of and - q are placed a 'd' distance apart.
the force on Q, due to q3 in terms of q1, q2, Draw the line on which the resultant field
⃗⃗⃗ , ⃗⃗⃗ and . is parallel to the line joining the two
[CBSE Sample Paper 08] charges.
Solution. The forces exerted by ql and [CBSE OD 96C]
q2 on Q are shown in Fig. 1.119. Solution. As shown in Fig. 1.120, the
resultant at all points lying on the
perpendicular bisector (dotted line) is
parallel to the line joining the charges + q
and - q.

Fig. 1.119
Fig. 1.120
According to Coulomb's law, force
Problem 11. Draw a diagram to show
exerted by q1 on Q is
lines of force in a plane containing two
⃗⃗⃗ equal point charges of opposite sign
separated by a small distance. Giving
reason, indicate on the diagram a point
where a small positive charge experiences
a force parallel to the line joining the two
Force exerted by q2 on Q is charges. [CBSE D 93C]

⃗⃗⃗ Solution. As shown in Fig. 1.120,


electric field at any point P on the
equatorial line is parallel to the line joining
According to the principle of superposition,
the two charges. So if a small positive
total force exerted by q1 and q2 on Q is
charge is placed at such a point, it will
⃗⃗⃗ ⃗⃗⃗

79
MITTAL INSTITUTE

experience a force parallel to the line


joining the two charges.
Problem 12. What is meant by the
statement that the electric field of a point
charge has spherical symmetry whereas
that of an electric dipole is cylindrically
symmetric ? [Haryana 96]
Solution. The electric field due to a
point charge q at distance r from it is given
by

Clearly, the magnitude of field ⃗ will be


same at all points on the surface of a
sphere of radius r drawn around the point
charge and does not depend on the
direction of . Hence the field due to a
point charge is spherically symmetric.
Electric field at distance r on the
equatorial line of an electric dipole of
dipole moment p is given by

Fig. 1.121 Electric dipole (a) parallel (b)


The electric field E is same at all perpendicular to uniform electric field ⃗ .
points which lie on a cylinder of radius r
with its axis on the dipole axis and the (b) The two equal and opposite forces
field pattern looks same in all planes -qE and +qE constitute a couple and
passing through the dipole axis. We say hence a torque acts on the dipole, given
that the electric field of an electric dipole is by
cylindrically symmetric.  = pE sin 90° = q . 2aE [∵ p = - q .
Problem 13. An electric dipole free to 2a]
move is placed in a uniform electric field. This torque rotates the dipole about an
Explain alongwith diagram its motion axis perpendicular to the electric field and
when it is placed, passing through the midpoint of the dipole.
(a) parallel to the field, Problem 14. An electric dipole is a pair
(b) perpendicular to the field. of equal and opposite charges, separated
by a small fixed distance between them.
[CBSE Sample Paper 1990] The dipole is free to move. What is the
Solution, (a) Since the line of action of action on it, when it is placed in
the two forces passes through the same (i) a uniform electric field, and
point, the net force and the net torque (ii) a non-uniform electric field ?
acting on the dipole is zero. So no motion
is produced when a dipole is placed Solution, (i) In a uniform electric field,
parallel to the electric field. an electric dipole experiences two equal,
opposite and parallel forces at its two
ends. The net force on it is zero but it
experiences a torque due to which it
rotates about an axis perpendicular to the
electric field and passing through its mid-
point.

80
MITTAL INSTITUTE

(ii) In a non-uniform electric field, an


electric dipole experiences two unequal
and non-parallel forces at its two ends.
The two forces add up to give a resultant
force and a torque. So the dipole will
accelerate linearly in the direction of the
resultant force and also accelerate
angularily in the direction of the resultant
torque.
Problem 15. An electric dipole of Fig. 1.122
dipole moment is placed in a uniform (i) Wh n θ = 90°, torque is maximum [Fig.
electric field ⃗ . Write the expression for 1.122(a)].  sin 90° = pE
the torque  experienced by the dipole.
Identify two pairs of perpendicular vectors (ii) Wh n θ = 30° or 150°, torqu is
in the expression. Show diagrammatically half the maximum value [Fig. 1.122(b)].
the orientation of the dipole in the field for  = p E sin (30° or 150°)
which the torque is (i) maximum (ii) half
the maximum value (iii) zero. [CBSE 
Sample Paper 08]
Solution. Torque experienced by the (iii) Wh n θ = 0° or 180°, torqu is
minimum [Fig. 1.122(c)]
electric dipole of dipole moment in a
uniform electric field ⃗ is given by  p E sin (0° or 180°) = 0

= × ⃗ Problem 16. Two small identical


electrical dipoles AB and CD, each of
The pairs of perpendicular vectors are dipole moment 'p' are kept at an angle of
: 120° as shown in Fig. 1.123. What is the
1.  and 2.  and ⃗ resultant dipole moment of this
combination ? If this system is subjected
to electric field ( ⃗ ) directed along +X
direction, what will be the magnitude and
direction of the torque acting on this ?
[CBSE Dll]

Fig. 1.123
Solution. The directions of the two
dipole moments and their resultant are
shown in Fig. 1.124.

Fig. 1.124

81
MITTAL INSTITUTE

Given pA = pC = p Fig. 1.125


Resultant dipole moment, Find
√ (i) the force on the charge at the centre of
shell and at the point A,

√ (ii) the electric flux through the shell.


( )
[CBSE D 15]
This dipole moment acts along the Solution. Net force on the charge Q/2,
bisector of AOC i.e., at an angle of 30° placed at the centre of the shell is zero.
with +X direction.
Force on the charge 2Q kept at point A
∴ Torque,  = pE sin 30° = pE at distance r from the centre is

By right hand rule, the torque  acts


into the plane of paper along Z-direction.
Problem 17. Define electric flux. Write
its SI units. A spherical balloon carries a
charge that is uniformly distributed over its (ii) Electric flux through the shell,
surface. As the balloon is blown up and
increases in size, how does the total ϕ
electric flux coming out of the surface
change ? Give reason. Problem 19. Two thin concentric and
[CBSE D 07] coplanar spherical shells, of radii a and b
(b > a) carry charges, q and Q,
Solution. The electric flux through a respectively. Find the magnitude of the
given surface area is the total number of electric field, at a point distant x, from their
electric lines of force passing normally common centre for
through that area.
(i) 0 < x < a (ii) a ≤ x < b (iii) b ≤ x < ∞
It is given by
[CBSE F 15, D 16C]
∆ϕE = ⃗ . ⃗
Solution. (i) For 0 < x < a, the charge
= E S os θ enclosed by Gaussian surface I is zero.
SI unit of electric flux = Nm2C-1
As the balloon is blown up and
increases in size, the total charge on its
surface remains constant. Hence, by
Gauss's theorem, the total electric flux
coming out of its surface remains
unchanged.
Problem 18. A thin metallic spherical shell
of radius R carries a charge Q on its
surface. A point charge is placed at its Fig. 1.126
centre C and another charge +2Q is By Gauss's theorem,
placed outside the shell at a distance x
from the centre as shown in Fig. 1.125. ∮⃗ ⃗


(ii) For ≤ x < b, the net charge
enclosed by the Gaussian surface II is q.

82
MITTAL INSTITUTE

∴ or Solution. (i) For points inside the


balloon, E = 0.
(iii) For b ≤ x < ∞, th n t h rg
(ii) As the balloon is blown up, surface
enclosed by the Gaussian surface III is (q
h rg d nsity σ d r s s nd so th
+ Q).
field, (E = σ/ε0) on its surface decreases.

(iii) For points outside the balloon,
or

Problem 20. A spherical rubber


balloon carries a charge that is uniformly As the balloon is blown up, the charge
distributed over its surface. As the balloon enclosed by the Gaussian surface
is blown up ; how does E vary for points (i) remains same, so E does not change.
inside the balloon, (ii) on the surface of the
balloon and (iii) outside the balloon ?

HOTS Problems on Higher Order Thinking Skills


Problem 1. A metallic spherical shell pairs of charges (ii) angle between the
has an inner radius R1 and outer radius dipole axes of these two dipoles.
R2. A charge Q is placed at the centre of
[CBSE Sample Paper 13]
the spherical cavity. What will be the
surface charge density on (i) the inner Solution. As the two dipoles have
surface and (ii) the outer surface ? equal dipole moments, so
[Exemplar Problem] (i) qa = Q ’ 
Solution. Charge -Qis induced on the
(ii) their dipole axes must have same
inner surface and charge + Q is induced
direction i.e., θ = 0°
on the outer surface of the cavity.
Problem 4. Sketch the electric field
∴ Surface charge density on the inner lines for a uniformly charged hollow
surface = cylinder as shown in Fig. 1.127.

Surface charge density on the outer


surface =

Problem 2. The dimensions of an atom


are of the order of an Angstrom. Thus
there must be large electric fields between Fig. 1.127
the protons and electrons. Why, there is
the electrostatic field inside a conductor Solution.
zero ?
[Exemplar Problem]
Solution. The electric fields of protons
and electrons bind the atoms to a neutral
entity. Fields are caused by excess
charges, there can be no excess charge
on the inner surface of an isolated
conductor.
Problem 3. Two dipoles, made from
charges ±q and ±Q, respectively, have
equal dipole moments. Give the (i) ratio
between the 'separations' of these two

83
MITTAL INSTITUTE

(i) near A (ii) near B (iii) near C (iv)


nowhere. Justify your answer.

Fig. 1.128 Fig. 1.129

Problem 5. Two point charges placed Solution, (a) Charges A and C are
at a distance r in air exert a force F on positive because the lines of force are
each other. At what distance will these emerging out from them.
charges experience the same force F in a (b) Charge C has the largest
medium of dielectric constant ? [PMT AP magnitude because the maximum number
90] of field lines are associated with it.
Solution. (c) (?) Near A No neutral point can
exist between unlike charges A and B or
between B and C. The neutral point exists
…(i) between like charges A and C. Also, the
neutral point will be closer to the charge
with smaller magnitude. Hence, electric
…(ii) field is zero near charge A
Dividing (1) by (ii), we get, Problem 8. A glass rod rubbed with
silk is brought close to two uncharged
spheres in contact with each other,
inducing charges on them as shown in
or
√ Fig. 1.130. Describe what happen when
Problem 6. A force F is acting between
two charges placed some distance apart
in vacuum. If a brass rod is placed
between these two charges, how does the
force change ?
Fig. 1.130
Solution. For any metal, =∞
(i) the spheres are slightly separated, and

(ii) the glass rod is subsequently removed,
i.e., in the presence of brass rod, the and finally
force between the two charges becomes
(iii) the spheres are separated far apart ?
zero.
Solution.
Problem 7. Figure 1.129 shows the
electric lines around three charges A, B
and C.
(a) Winch charge is positive ?
(b) Which charge has the largest
magnitude ? Why ?
Fig. 1.131
(c) In which region or regions of the
picture could the electric field be zero (i) When the spheres are slightly
? separated with the glass rod
undisturbed, there is little change in

84
MITTAL INSTITUTE

the distribution of charges, as shown Therefore, deflection of charged


in Fig. 1.131(a). particle in time t in y-direction is
(ii) When the glass rod is removed, there h = 0 × t + at2 = t2
is redistribution of charge on each
sphere. Their positive and negative i.e., h∝
charges will face each other, as shown As the particle 3 suffers maximum
in Fig. 1.131(b). deflection in y-direction, so it has highest
(iii) The charge on each sphere gets charge to mass (q / m) ratio.
uniformly distributed as shown in Fig. (a) Negative charge.
1.131(c).
(b) (iii) and (iv). The particles must have
Problem 9. Figure 1.132 shows tracks charges of same sign and same e / m
of three charged particles in a uniform ratio.
electrostatic field. Give the signs of the
three charges. Which particle has the (c) To find e/m, we measure the vertical
highest charge to mass ratio ? displacement h as the particle crosses
the capacitor plates.
Time taken by a particle to cross the
capacitor plates,

Fig. 1.132 ( )
(a) Suppose that a particle is attracted
towards the positive plate : what must Problem 10. Figure 1.133 shows three
the charge on it be ? different patterns of electric field lines. In
each pattern, a proton is released from
(b) Suppose, two particles have identical rest at point P and then accelerated
curved trajectories. Which of the towards the point Q by the electric field.
following are necessarily true ? Rank the patterns according to the linear
(i) They have same charge ; (ii) They have momentum of the proton when it reaches
same mass; (iii) The charges have the Q, greatest first.
same sign; (iv) They have the same elm
ratio.
(c) You are given the initial velocity v of a
beam particle and the length of the
capacitor l. What other measurement
would enable one to find e/m ?
[NCERT ; CBSE D 01C]
Solution. Particles 1 and 2 have
negative charges because they are being
deflected towards the positive plate of the
electrostatic field.
Particle 3 has positive charge because
it is being deflected towards the negative
plate. Fig. 1.133
Acceleration acting on charge q in y- Solution. The lines of force near point
direction, P are closest to each other in pattern (c)
and farthest apart in pattern (a).
Consequently, electric field near point P is
strongest in case (c) and weakest in case
(a).

85
MITTAL INSTITUTE

∴ Momentum of proton at point Q in


pattern (c)
> Momentum of proton at point Q in
pattern (b)
> Momentum of proton at point Q in
pattern (a)
Problem 11. Plot a graph showing the
variation of coulomb force (F) versus
( ), where r is the distance between the
two charges of each pair of charges : Fig. 1.135
(1μC, 2 μC) and (2μC, -3μC). Interpret the
But ⃗⃗⃗ ̂ ̂
graphs obtained.
⃗⃗⃗ ̂ ̂
[CBSE OD 11]
( ̂ ̂) ( ̂ ̂)
Solution, (i) As F ∝ , so F versus ∴⃗ * +
graph is a straight line for both pairs of
charges. Problem 13. A point charge is placed
at the centre of spherical Gaussian
(ii) For th p ir (1 μC, 2 μC), F is surface. How will electric flux ϕE change if
repulsive, the graph OA has a +ve slope.
For th p ir (2μC, -3μC), F is attractive, (i) the sphere is replaced by a cube of
the graph OB has a -ve slope. same or different volume,
(Hi) Clearly, Fattraction = 3Frepulsion, the (ii) a second charge is placed near, and
slope of OB is three times the slope of outside, the original sphere,
OA. (iii) a second charge is placed inside the
sphere, and
(iv) the original charge is replaced by an
electric dipole ?
Solution. By Gauss's law, ϕE =
∮ ⃗ ⃗⃗⃗⃗

(i) ϕE does not change because it depends


only on the total charge enclosed by
the Gaussian surface and not its
shape or size.
Fig. 1.134
(ii) ϕE does not change because the total
Problem 12. Two point charges, q1
flux is determined by the charge inside
and q2 are located at points (a,0,0) and
the surface and not on the charge
(0,b,0) respectively. Find the electric field,
outside.
due to both these charges, at the point
(0,0, c). (iii) ϕE will change because the total
charge inside the surface has
[CBSE Sample Paper 13]
changed.
Solution. Net electric field at the point
(iv) ϕE becomes zero, because a dipole
(0,0, c)due to the charges q1 and q2 is
consists of two equal and opposite
charges and so the net charge inside
the surface is zero.
⃗ ⃗⃗⃗⃗ ⃗⃗⃗⃗ * ⃗⃗⃗ ⃗⃗⃗ + Problem 14. Four point charges are
placed at the four corners of a square in
the two ways (i) and (ii) as shown in Fig.

86
MITTAL INSTITUTE

1.136. Will the (i) electric field (ii) electric Fig. 1.138
potential, at the centre of the square, be
Solution. Let each side of the
the same or different in the two
equilateral triangle be d.
configurations and why ? [CBSE Sample
Paper 08] Dipole moment along CA = qd
Dipole moment along CB = qd
By parallelogram law, the net dipole
moment acts along diagonal CD. Its
magnitude is


Fig. 1.136
This dipole moment acts along the
Solution, (i) In Fig. 1.137(i), the electric bisector of the angle at charge -2q.
fields ⃗ and ⃗ get added and also ⃗
Problem 16. Two similar balls each
and ⃗ get added. Hence there is a net having mass m and charge q are hung
field at the centre O. In Fig. 1.137(ii), the from a silk thread of length l. Prove that
equilibrium separation,

( )

when each thread makes a small angle θ


with the vertical.
Fig. 1.137 Solution. Refer to Fig. 1.139.
According to Coulomb's law, the electro-
fields ⃗ and ⃗ are equal and opposite static repulsion between the two balls will
and also ⃗ and ⃗ are equal and be
opposite. Hence the net field at the centre
is zero.
(ii) In both figures 1.137(i) and (ii), the
potentials due to positive charges are +ve For s ngu r disp nt θ, w
and those due to negative charges are have
equally negative. Hence the net potential
at the centre is zero. ≃

Problem 15. Three charges +q, +q and


-2q are placed at the vertices of an
equilateral triangle. What is the dipole
moment of the system ?

Fig. 1.139
∴ Restoring force on each ball

87
MITTAL INSTITUTE

= mg sin θ mg ≃ Acceleration,

For equilibrium,

∴ T = 2π√ √

or * + Problem 19. Four particles, each


having a charge q, are placed on the
Problem 17. Five point charges, each vertices of a regular pentagon. The
of value + q coulomb are placed on five distance of each comer from the centre is
vertices of a regular hexagon of side L 'a'. Determine the electric field at the
metres. Find the magnitude of force on a centre of the pentagon.
charge - q coulomb placed at the centre of
the hexagon. [IIT 92] Solution. Suppose the four charges
are placed at the comers A, B, C and D of
Solution. The situation is shown in Fig. the pentagon ABODE. If we place a
1.140. Forces on charge –q at O due to charge q at the comer E also, then by
charges at A and D are equal and symmetry, the total electric field at O must
opposite and hence cancel out. be zero. Thus, the field at the centre O
due to the charges A, B, C and D is equal
and opposite to the field due to the charge
q at E alone.

Fig. 1.140
Similarly, the forces due to charges at Fig. 1.141
B and E cancel out.
Electric field at O due to charge q at E
The only force on charge -q at O is
exerted by charge +q at C. It is given by , along EO

newton, along OC. ∴ Electric field at O due to the charges at


A, B, C and D
Problem 18. A simple pendulum
onsists o α s sph r o ss , along OE.
suspended by a thread of length l. The
sphere carries a positive charge q. The Problem 20. An infinite number of
pendulum is placed in a uniform electric charges, each equal to q are placed
field of strength E directed vertically along X-axis at x = 1, x = 2, x = 4, x =
upwards. With what period will the 8, …… and so on.
pendulum oscillate if the electrostatic force (i) Find the electric field at the point x = 0
acting on the sphere is less than the due to this set up of charges.
gravitational force ? Assume oscillations
(ii) What will be the electric field, if in the
to be small.
above set up, the consecutive charges
Solution. The sphere experiences an have opposite signs.
upward pull qE.
[IIT 95]
∴ Net force on the sphere
= mg - qE, in downward direction

88
MITTAL INSTITUTE

Solution, (i) At the point x = 0, the electric Component of E1 along PR = E1 os α


fields due to all the charges are in the
Component of E1 along PQ = E1 sin α
same x-direction and hence get added up.
Component of E2 along PR = E2 os β
* +
Component of E2 along PS = E2 sin β
* + The resultant field will be parallel to
PR if the components El sin α nd E2 sin β
(ii) (ii) Electric field at x = 0 is are equal and opposite or each
component is zero. Hence
* +
(i) When E1 sin α = E2 sin β
* +

.
or r1 = r2
Problem 21. Two point charges + q
and - q are placed distance d apart. What (ii) When E1 sin α = E2 sin β = 0, w h v
are the points at which the resultant sin α = sin β = 0
electric field is parallel to the line joining
the two charges ? [IIT] i.e., a = 0° or 180° nd β = 0 nd 180°.

Solution. As shown in Fig. 1.142, Thus the resultant intensity will be


suppose the charges +q and. -q are parallel to the line joining A and B if (i) P
located at points A and B distance d apart. lies on the perpendicular bisector of AB, or
(ii) P lies on either side of AB i.e., a = 0°
nd β = 180°.
Problem 22. Eight identical point
charges of q coulomb each are placed at
the corners of a cube of each side 0.1 m.
Calculate the electric field at the centre G
of the cube. Calculate the field at the
centre when one of the comer charges is
removed.
Solution. Length of each side, l = 0.1
m
Distance of each comer from the
centre of the cube is
Fig. 1.142
√ √
Let P be a point such that AP = r1 and √
BP = r2.
When all the eight point charges are
Electric field at P due to charge + q is placed at the corners, electric fields of the
, diagonally opposite charges cancel out in
pairs.
along AP produced.
∴ Net field at the centre = 0.
Electric field at P due to charge - q is
When a charge is removed from one
, corner, the electric fields due to three
pairs of charges cancel out. However, the
along PB produced. charge at the remaining comer creates
Draw PR | | AB and PQ, PS ± PR. I α field,
nd β r th ng s d by AP and BP
with AB, then

89
MITTAL INSTITUTE

(i) For which surface is the electric flux


( √ ) zero ?
(ii) Over which surface is ⃗ zero ?
The field points towards the comer
(iii) Over which surface is | ⃗ | constant ?
having no charge.
Problem 23. If the total charge (iv) Over which surface does | ⃗ | change
enclosed by a surface is zero, does it ?
imply that the electric field everywhere on Solution. Electric field of a line charge,
the surface is zero ? Conversely, if the 
electric field everywhere on the surface is E= , acting radially outward
zero, does it imply that the charge inside
is zero ? [Exemplar Problem] (i) At the two plane end faces,

Solution. As q =0, so from the Gauss's ∫⃗ ⃗ .


theorem, we have (ii) For any finite distance from the line
∮⃗ ⃗ charge, ⃗ cannot be zero.
(iii) At all points of the curved surface, | ⃗ |
Clearly, the above equation does not is constant.
imply that ⃗ is necessarily zero at all
points on the Gaussian surface. It may (iv) At the plane end faces, | ⃗ | decreases
also be possible that ⃗ is non-zero but it is with the increase in distance r.
perpendicular to d at all points on the Problem 25. A small metal sphere
surface, even then the integral carrying charge + Q is located at the
centre of a spherical cavity in a large
∮⃗ would be zero. uncharged metal sphere as shown in Fig.
However, the converse is true. If ⃗ is 1.144. Use Gauss's theorem to find
zero at every point on Gaussian surface, electric field at points P1 and P2. [CBSE D
then from Gauss's theorem 05, OD 14C]

∮⃗ ⃗

we get, q=0
i.e., no net charge is enclosed by the
Gaussian surface.
Problem 24. Figure 1.143 slimes a
cylindrical Gaussian surface for an in Fig. 1.144
finitely long thin straight wire of uniform
linear charge density. Solution, (i) To determine the electric
field at point P1, consider a concentric
spherical surface of radius r1 as the
Gaussian surface. By symmetry, the field
E will have same magnitude at all points
on this surface and will point radially
outward.
∴ Flux through the Gaussian surface,

Fig. 1.143
Charge enclosed by Gaussian surface = +
Answer the following: Q
By Gauss's theorem,

90
MITTAL INSTITUTE

or or through the cube, (ii) net charge enclosed


within the cube. [CBSE Sample Paper 08]

(ii) As the electric field inside a


conductor is zero, so field at point P2 = 0.
Problem 26. A spherical conducting
shell of inner radius r1 and outer radius r2
h s h rg 'Q'. A h rg ‘q’ is placed at
the centre of the shell.
Fig. 1.146
(a) What is the surface charge density on
the (i) inner surface, (ii) outer surface Solution, (i) The electric field is acting
of the shell ? only in X-direction and its Y and Z-
components are zero. So the flux passes
(b) Write the expression for the electric only through faces M and N.
field at a point x > r2 from the centre of
the shell. [CBSE OD 10] The magnitude of the electric field at
the face M(x =0) is
Solution. The charge q at the centre
induces charge –q on the inner surface of EM = 5 Ax + 2 B = 5 × 10 × 0 + 2 × 5 =
the shell and charge +q on the outer 10 NC-1
surface of the shell. Flux,
M = EMl2 os θ =10 × (0.10)2 ×
cos180°
= -0.1 Nm2C-1
The magnitude of the electric field at
the face N (x =10 cm) is
EN = 5 Ax + 2 B = 5 × 10 × 0.10 + 2 × 5
= 15 NC-1
Fig. 1.145
Flux, N = EN l2 os θ
(a) (i) Surface charge density on the
inner surface, = 15 × (0.10)2 × cos 0° = 0.15
2 -1
Nm C
Net flux through the cube,
(ii) Surface charge density on the outer ϕE = ϕM + ϕN = -0.1 + 0.15 = 0.05 Nm2C_1.
surface,
(ii) Total charge enclosed within the
cube,
q = ε0 ϕE = 8.854 × 10-12 × 0.05 = 4.43
(b) For x > r2, the field is similar to that × 10-13 C.
of a point charge. Problem 28. A charge Q is distributed
uniformly on a ring of radius r. A sphere of
equal radius r is constructed with its
centre at the periphery of the ring, as
Problem 27. Electric field in Fig. 1.146 shown in Fig. 1.147. Find the electric flux
is directed along +X direction and given by through the sphere.
Ex = 5 Ax + 2 B, where E is in NC-1 and x
is in metre, A and B are constants with
dimensions. Taking A = 10 NC-1m-1 and B
= 5 NC-1, calculate : (i) the electric flux

91
MITTAL INSTITUTE

must be -Ze. If p is the negative charge


density, then we must have

or
By spherical symmetry of the charge
distribution the electric field ⃗ depends
only on radial distance and not on the
direction of . It should point radially
Fig. 1.147 inwards or outwards. So we imagine a
Solution. Clearly, OA = OO = O' A = r. spherical Gaussian surface of radius r
centered at the nucleus.
Thus ∆OAO’ is equilateral. Hence
AOO’ = 60° and AOB = 120°. (i) For r < R. Flux through the
Obviously, one-third portion AO’ B of the Gaussian surface,
ring lies in the sphere. E = E × 4πr2
∴ Charge enclosed by the sphere, q = Charge enclosed by the Gaussian
surface,

From Gauss's theorem, electric flux q = Positive nuclear charge


through the sphere is + Negative charge in a sphere of
radius r
ϕ
( )
Problem 29. An early model of an
atom considered it to have a positively
charged point nucleus of charge Ze, ( )
surrounded by a uniform density of
negative charge up to a radius R. The Applying Gauss's theorem, E = q/ε0,
atom as a whole is neutral. For this model, we get
what is the electric field at a distance r
from the nucleus ? [NCERT] ( )
Solution. Figure 1.148 shows the
charge distribution for the given model of or ( ) (r <
the atom.
R)
The field E points radially outward.
(ii) For r > R. As the atom is neutral,
the total charge enclosed by the Gaussian
surface is zero. By Gauss's theorem,
E × 4 πr2 = 0
or E = 0. (r> R)
(iii) At r = R. Both of the above cases
Fig. 1.148 An early model of atom.
give the same result: E = 0.
As the atom is neutral, the total
negative charge in a sphere of radius R
NCERTEXERCISE
1.1. What is the force between two Ans. Here q1 = 2 × 10-7C, q2 = 3 × 10-
7
small charged spheres having charges of C,
2 × 10-7 Cand 3 ×10-7 C placed 30 cm
r = 30 cm = 0.30 m
apart in air ?
According to Coulomb's law,

92
MITTAL INSTITUTE

= 2.287 ×1039.
The factor ke2 / Gmemp represents the
(repulsive). ratio of electrostatic force to the
gravitational force between an electron
1.2. The electrostatic force on a small
and a proton. Also, the large value of the
sphere of charge 0.4 μC due to another
ratio signifies that the electrostatic force is
small sphere of charge -0.8μC in air is 0.2
much stronger than the gravitational force.
N. (i) What is the distance between two
spheres ? (ii) What is the force on the 1.4. (i) Explain the meaning of the
second sphere due to the first ? statement 'electric charge of a body is
quantised.'
Ans. (i) Here q1 = 0.4 μC = 0.4 × 10-6
C (ii) Why can one ignore quantisation of
electric charge when dealing with
q2 = - 0.8 μC = - 0.8 × 10-6 C, F = 0.2 N, r
macroscopic i.e., large scale charges
=?
?
As Ans. (i) Quantisation of electric charge
means that the total charge (q) of a body
∴ is always an integral multiple of a basic
charge (e) which is the charge on an
electron. Thus q = ne, where n = 0, ± 1, ±
2, ± 3, ……...........................

or cm. (ii) While dealing with macroscopic


charges (q = ne), we can ignore
(ii) The two charges mutually exert quantisation of electric charge. This is
equal and opposite forces. because e is very small and nis very large
∴ Force on the second sphere due to and so q behaves as if it were continuous
i.e., as if a large amount of charge is
the first = 0.2 N (attractive).
flowing continuously.
1.3. Check that the ratio ke2/Gmemp is
dimensionless. Look up a table of physical 1.5. When a glass rod is rubbed with a
silk cloth, charges appear on both. A
constants and determine the value of this
similar phenomenon is observed with
ratio. What does this ratio signify ?
many other pairs of bodies. Explain how
Ans. [ ] this observation is consistent with the law
of conservation of charge.
unit
Ans. It is observed that the positive
As the ratio k e2 / Gmm has no unit, so charge developed on the glass rod has
it is dimensionless. the same magnitude as the negative
Now k = 9 × 109Nm2C-2 charge developed on silk cloth. So total
charge after rubbing is zero as before
G = 6.67 × 10-11 Nm2 kg-2 rubbing. Hence the law of conservation of
e = 1.6 × 10-19 kg charge is being obeyed here.
me = 9.1 × 10-31 kg 1.6. Four point charges qA = 2 μC, qB =
-5 μC, qC = 2 μC, qD = -5 μC are located at
the corners of a square ABCD of side

Fig. 1.150
and mp = 1.66 × 10-27 kg
( )

93
MITTAL INSTITUTE

gradually with increasing distance from it


and becomes zero at infinity i.e., electric
field cannot vanish abruptly. So a line of
force cannot have sudden breaks, it must
be a continuous curve.
(b) If two lines of force intersect, then
there would be two tangents and hence
two directions of electric field at the point
of intersection, which is not possible.
1.8. Two point charges qA = + 3 μC
Fig. 1.149 and qB = -3 μC are located 20 cm apart in
10 cm. What is the force on a charge of 1 vacuum, (i) Find the electric field at the
μC p d t th ntr o th squ r ? midpoint O of the line AB joining the two
charges, (ii) If a negative test charge of
Ans. Here

magnitude 1.5 × 10-9 C is placed at the
centre, find the force experienced by the
test charge. [CBSE OD
√ cm √ 03]
For s x rt d on th h rg o 1 μC Ans. The directions of the fields EA
located at the centre are and EB due to the charges qA and qB at the
midpoint P are as shown in Fig. 1.150.
⃗⃗⃗⃗
√ Electric field at the midpoint O due to
qA,
, along ⃗⃗⃗⃗⃗

⃗⃗⃗⃗

, along
, along ⃗⃗⃗⃗⃗
Electric field at the midpoint O due to qB,
⃗⃗⃗⃗

, along ⃗⃗⃗⃗⃗
, along

√ Resultant field at the midpoint is

, along ⃗⃗⃗⃗⃗⃗
, along
Clearly, ⃗⃗⃗⃗ ⃗⃗⃗⃗ and ⃗⃗⃗⃗ ⃗⃗⃗⃗
(ii) Force on a negative charge of 1.5 × 10-
H n tot or on 1 μC h rg is 9
C placed at the midpoint O,
⃗⃗⃗⃗ ⃗⃗⃗⃗ ⃗⃗⃗⃗ ⃗⃗⃗⃗
⃗⃗⃗⃗ ⃗⃗⃗⃗ ⃗⃗⃗⃗ ⃗⃗⃗⃗ zero N. , along
1.7. (al) An electrostatic field line is a The force on a negative charge acts in
continuous curve. That is, a field line a direction opposite to that of the electric
cannot have sudden breaks. Why not ? field.
(b) Explain why two field lines never 1.9. A system has two charges qA =
cross each other at any point ? [Punjab 2.5 ×10-7 C and qB = -2.5 × 10-7 C, located
01, 02; CBSE D 05, 03 ; OD 14] at points A (0,0, -15 cm) and B (0,0, +15
Ans. (a) Electric lines of force exist cm) respectively. What is the total charge
throughout the region of an electric field. and electric dipole moment of the system
The electric field of a charge decreases ?

94
MITTAL INSTITUTE

Ans. Clearly, the two charges lie on Z-


axis on either side of the origin and at 15
cm from it, as shown in Fig. 1.151. Since polythene has negative charge,
-7 so electrons are transferred from wool to
∴ 2a = 30 cm = 0.30 m, q = 2.5 × 10 C
polythene during rubbing.
(ii) Yes, there is a transfer of mass
from wool to polythene because each
electron has a finite mass of 9.1 × 10-31
kg.
Mass transferred
= me × n = 9.1 × 10-31 × 2 × 1012
= 1.82 × 10-18 kg
Clearly, the amount of mass
transferred is negligibly small.
1.12. (a) Two insulated charged
Fig. 1.151 copper spheres A and B have their
centres separated by a distance of 50 cm.
Total charge = qA + qB = 2.5 × 10-7 - 2.5
What is the mutual force of electrostatic
× 10-7 = 0
repulsion if the charge on each is 6.5 × 10-
7
Dipole moment, C ? The radii of A and B are negligible
compared to the distance of separation,
p - q × 2a = 2.5 × 10-7 × 0.30
(b) What is the force of repulsion if each
= 0.75 × 10-7 Cm sphere is charged double the above
amount, and the distance between them is
The dipole moment acts in the
halved ?
direction from B to A i.e., along negative
Z-axis. Ans. Refer to the solution of Example
9 on page 1.12.
1.10. An electric dipole with dipole
moment 4 × 10-9 Cm is aligned at 30° with 1.13. Suppose the spheres A and B in
the direction of a uniform electric field of Exercise 1.12 have identical sizes. A third
magnitude 5 × 104 NC-1 Calculate the sphere of the same size but uncharged is
magnitude of the torque acting on the brought in contact with the first, then
dipole. brought in contact with the second, and
finally removed from both. What is the
Ans. Here p = 4 × 10“9 C , θ = 30°, E
new force of repulsion between A and B ?
= 5 × 104 NC-1
Ans. Refer to the solution of Example
∴ Torque,  = pE sin θ
10 on page 1.12.
= 4 × 10-9 × 5 ×104 × sin 30° 1.14. Figure 1.152 shows tracks of
-4
= 10 Nm. three charged particles in a uniform
electrostatic field. Give the signs of the
1.11. A polythene piece rubbed with
three charges. Which particle has the
wool is found to have a negative charge of
highest charge to mass ratio ?
3.2 × 10-7C. (i) Estimate the number of
electrons transferred, (ii) Is there a
transfer of mass from wool to polythene ?
Ans. (i) Here q = 3.2 × 10-7 C, e = 1.6
× 10-19 C
As q = ne, therefore Fig. 1.152
Number of electrons transferred, Ans. Refer to the solution of Problem 9
on page 1.73.

95
MITTAL INSTITUTE

1.15. Consider a uniform electric field : say that the net charge inside the box is
zero.
⃗ = 3 × 103 ̂ NC-1 (i) What is the flux
of this field through a square of 10 cm on 1.18. A point charge +10 μC is a
a side whose plane is parallel to the Y-Z- distance 5 cm directly above the centre of
plane ? (ii) What is the flux through the a square of side 10 cm as shown in Fig.
same square if the normal to its plane 1.153(a). What is the magnitude of the
makes a 60° angle with the X-axis ? electric flux through the square ? (Hint:
Think of the square as one face of a cube
Ans. (i) Normal to a plane parallel to Y- with edge 10 cm)
Z plane points in X-direction, so
Ans. We can imagine the square as
 = 0.10 × 0.10 ̂ m2 = 0.01 ̂ m2 face of a cube with edge 10 cm and with
Electric flux, the charge of + 10 μC p d t its ntr ,
as shown in Fig. 1.153(b).
ϕE = ⃗ .  = 3 × 103 ̂ . 0.01 ̂
= 30 .̂ ̂ = 30 Nm2C-1.
(ii) H r θ = 60°
∴ ϕE = E∆S os 60° = 3 × 103 × 0.01 cos
60°
= 30 × = 15 Nm2C-1.
1.16. Consider a uniform electric field : Fig. 1.153
⃗ = 3 × 10 ̂ NC . What is the net flux of
3 -1
Symmetry of six faces of a cube about
this field through a cube of side 20 cm its centre ensures that the flux ϕS through
oriented so that its faces are parallel to the each square face is same when the
coordinate planes ? charge q is placed at the centre.
Ans. The flux entering one face ∴ Total flux,
parallel to Y-Z plane is equal to the flux
leaving other face parallel to Y-Z plane. ϕE = 6 × ϕS =
Flux through other faces is zero. Hence
net flux through the cube is zero. or ϕS = × 10 × 10-6 × 4π × 9 × 109
1.17. Careful measurement of the = 1.88 ×105 Nm2C_1.
electric field at the surface of a black box
indicates that the net outward flux through 1.19. A point charge of 2.0 μC is at the
the surface of the box is 8.0 × 103 Nm2C1. centre of a cubic Gaussian surface 9.0 cm
(i) What is the net charge inside the box ? on edge. What is the net electric flux
(ii) If the net outward flux through the through the surface ?
surface of the box were zero, could you Ans. Here q = 2.0 μ C = 2.0 × 10-6C,
conclude that there were no charges
ε0 = 8.85 × 10-12C2N-1m-2
inside the box ? Why or why not ?
By Gauss's theorem, electric flux is
Ans. (i) ϕE = 8.0 × 103 Nm2C-2
Using Gauss theorem, ϕ = 2.26 × 105 Nm2 C-1.

ϕ 1.20. A point charge causes an electric


flux of -1.0 × 103 Nm2 C-1 to pass through
Charge, q = ε0. ϕE = 8.0 × 103 × a spherical Gaussian surface of 10.0 cm
radius centred on the charge, (i) If the
= 0.07 × 10-6 = 0.07 μC radius of the Gaussian surface were
doubled, how much flux would pass
(ii) No, we cannot say that there are no
charges at all inside the box. We can only

96
MITTAL INSTITUTE

through the surface ? (ii) What is the value Ans. E = 9 × 104 NC-1, r = 2cm = 0.02
of the point charge ? m
Ans. (i) ϕE = -103 Nm2C-1, because the 
Electric field of a line charge, E =
charge enclosed is the same in both the
cases. ∴ Linear charge density,
(ii) Charge,
= 2πε0Er = 2π × × 9 × 104 × 0.02
q = ε0 ϕE
= 0.01 × 10-5 Cm-1 = 0.1 μC -1
.
3
= ×(- 1.0 × 10 )
1.24. Two large, thin metal plates are
= - 8.84 × 10-9 C = - 8.84 nC. parallel and close to each other. On their
inner faces, the plates have surface
1.21. A conducting sphere of radius 10 charge densities of opposite signs and of
cm has an unknown charge. If the electric magnitude 17.0 ×10-22 Cm-2. What is E (a)
field 20 cm from the centre of the sphere to the left of the plates, (b) to the right of
is 1.5 × 103 NC-1 and points radially the plates, and (c) between the plates ?
inward, what is the net charge on the
sphere ? Ans. H r σ = 17.0 × 10-22 Cm-2
Ans. Electric field at the outside points (a) On the left, the fields of the two plates
of a conducting sphere is are equal and opposite, so E = Zero.

E= (b) On the right, the fields of the two


plates are equal and opposite, so E =
Zero.
∴ q = 4πε0Er2 = × 1.5 × 103 × (0.20)2
C (c) Between the plates, the fields due to
both plates are in same direction. So
= 6.67 × 10-9 C = 6.67 nC the resultant field is
As the field acts inwards, the charge q   
must be negative.
∴ q = - 6.67 nC. = 19.2 × 10-10 NC-1.

1.22. A uniformly charged conducting 1.25. An oil drop of 12 excess


sphere of 2.4 m diameter has a surface electrons is held stationary under a
charge density of 80.0 μC/ 2. (i) Find the constant electric field of 2.55 × 104 Vm-1 in
charge on the sphere, (ii) What is the total Millikan's oil drop experiment. The density
electric flux leaving the surface of the of the oil is 1.26 g cm-3. Estimate the
sphere ? [CBSE D 09C] radius of the drop. (g = 9.81 ms-2; e = 1.60
× 10-19 C)
Ans. Here R = 1.2 m Ans. Force on the oil drop due to electric
σ = 80.0μC -2
= 80 × 10-6 Cm -2 field = qE = neE

(i) Charge on the sphere is Weight of oil drop

q = 4π R2 σ = 4 × 3.14 ×(1.2)2 × 80 × = mg = volume × density × g = π r3 g


10-6C
= 1.45 ×10-3 C.
(ii) Flux,
ϕE = = 1.45 × 10-3 × 4π × 9 × 109

= 1.6 ×108 Nm2 C-1. Fig. 1.154

1.23. An infinite line charge produces The field E must act vertically
a field of 9 × 104 NC-1 at a distance of 2 downward so that the negatively charged
cm. Calculate the linear charge density.

97
MITTAL INSTITUTE

oil drop experiences an upward force and


balances the weight of the drop.
When the drop is held stationary,
Weight of oil drop
= Force on the oil drop due to
electric field
or ∴

Now,

kg

mm.
1.26. Which among the curves shown
in Fig. 1.155, cannot possibly represent
electrostatic field lines ?
Ans. Only Fig. 1.155(c) is right and the
remaining figures cannot represent the
electrostatic field lines.
Figure 1.155(A) is wrong because field
lines must be normal to a conductor.
Figure 1.155(b) is wrong because lines
of force cannot start from a negative
charge.

Fig. 1.155
Figure 1.155(c) is right because it
satisfies all the properties of lines of force.
Figure 1.155(d) is wrong because lines
of force cannot intersect each other.
Figure 1.155(c) is wrong because
electrostatic field lines cannot form closed
loops.
1.27. In a certain region of space,
electric field is along the Z-direction

98
MITTAL INSTITUTE

throughout. The magnitude of electric field (ii) Another conductor B with charge q
is, however, not constant but increases is inserted into the cavity keeping B
uniformly along the positive Z-direction at insulated from A. Show that the total
the rate of 105 NC-1m-1. What are the force charge on the outside surface of A is Q+ q
and torque experienced by a system [Fig. 1.157(b)],
having a total dipole moment equal to 10-7
(iii) A sensitive instrument is to be
C m in the negative Z-direction ?
shielded from the strong electrostatic
Ans. The situation is shown in Fig. fields in its environment. Suggest a
1.156. possible way.
As the electric field changes uniformly
in the positive Z-direction, so

As the system has a total dipole


moment in the negative Z-direction, so
Pz = - 10-7 Cm, px = 0, py = 0
Fig. 1.157
Ans. (i) Refer answer to Q.25(6) on
page 2.25.
(ii) Consider a Gaussian surface inside
the conductor but quite close to the cavity.
Inside the conductor, E = 0.

Fig. 1.156
In a non-uniform electric field, the
force on the dipole will be
Fig. 1.158
By Gauss's theorem,

= 0 + 0 – 10-7 ×105 = -10-2 N ϕ ∮ ⃗ ⃗⃗⃗⃗


The negative sign shows that the force
i.e., the total charge enclosed by the
on the dipole acts in the negative Z-
Gaussian surface must be zero. This
direction.
requires a charge of -q units to be induced
As the dipole moment p acts in the on inner surface of conductor A. But an
negative Z-direction while the electric field equal and opposite charge of +q units
E acts in the positive Z-dir tion, so θ = must appear on outer surface A so that
180°. charge on the surface of A is Q + q.
Torque,  = pE sin 180° = pE × 0 = 0. Hence the total charge on the surface
of A is Q + q.
1.28. (i) A conductor A with a cavity
[Fig. 1.157(a)] is given a charge Q. Show (iii) The instrument should be enclosed
that the entire charge must appear on the in a metallic case. This will provide an
outer surface of the conductor. electrostatic shielding to the instrument.

99
MITTAL INSTITUTE

1.29. A hollow charged conductor has a [Hint. Use Coulomb's law directly and
tiny hole cut into its surface. Show that the evaluate the necessary integral.]

electric field in the hole is ̂, where ̂ is Ans. Refer to the solution of Example
the unit vector in the outward normal 47 on page 1.37.
direction, and σ is the surface charge
1.31. It is now believed that protons
density near the hole.
and neutrons are themselves built out of
Ans. Consider the charged conductor more elementary units called quarks. A
with the hole filled up, as shown by proton and a neutron consists of three
shaded portion in Fig. 1.159. Applying quarks each. Two types of quarks, the so
Gauss's theorem, we find that field just called 'up' quark (denoted by u) of charge

outside is ̂ and is zero inside. This field + (2/3) e, and the 'down' quark (denoted
by d) of charge (-1/3) e, together with
can be viewed as the superposition of the
electrons build up ordinary matter.
field E2 due to the filled up hole plus
Suggest a possible quark composition of a
proton and neutron.
Ans. Charge on 'up’ quark (u) = + e

Charge on 'down’ quark (d) = e


Charge on a proton = e
Charge on a neutron = 0
Let a proton contain x 'up' quarks and
(3 - x) 'down' quarks. Then total charge on
a proton is
Fig. 1.159 ux + d (3 - x) = e
the field due to the rest of the charged
or
conductor. Since inside the conductor the
field vanishes, the two fields must be
or
equal and opposite, i.e.,
E1 - E2 = 0 ...(1) or x = 2 and 3 – x = 3 - 2
=1
And outside the conductor, the fields
are added up : Thus a proton contains 2 'up' quarks

and 1 'down' quark. Its quark composition
...(2) should be : uud.
Adding equations (1) and (2), we get Let a neutron contain y 'up' quarks and
  (3 - y) 'down' quarks. Then total charge on
or a neutron must be
Hence the field due to the rest of the uy + d (3 - y) = 0
conductor or the field in the hole is
or

̂
or
where n is a unit vector in the outward or y = 1 and 3 - y = 3 - 1
normal direction. =2
1.30. Obtain the formula for the Thus a neutron contains 1 'up' quark
electric field due to a long thin wire of and 2 'down' quarks. Its composition
uniform linear charge density  without should be : udd.
using Gauss's law.
1.32. (a) Consider an arbitrary
electrostatic field configuration. A small

100
MITTAL INSTITUTE

test charge is placed at a null point (i.e., the two charged plates is shown in Fig.
where ⃗ = 0) of the configuration. Show 1.160.
that the equilibrium of the test charge is
necessarily unstable.
(b) Verify this result for the simple
configuration of two charges of the same
magnitude and sign placed a certain
distance apart.
Ans. (a) We can prove it by Fig. 1.160
contradiction. Suppose the test charge Force on the charge - q in the upward
placed at null point be in stable direction is
equilibrium. Since the stable equilibrium
requires restoring force in all directions, ma = qE
therefore, the test charge displaced
∴ Acceleration,
slightly in any direction will experience a
restoring force towards the null point. That
Time taken to cross the field, t =
is, all field lines near the null point should
be directed towards the null point. This Vertical deflection at the far edge of
indicates that there is a net inward flux of the plate will be
electric field through a closed surface
around the null point. But, by Gauss's law,
the flux of electric field through a surface
enclosing no charge must be zero. This
contradicts our assumption. Hence the Like the motion of a projectile in
test charge placed at the centre must be gravitational field, the path of a charged
necessarily in unstable equilibrium. particle in an electric field is parabolic.

(b) The null point lies on the midpoint 1.34. Suppose that the particle in
of the line joining the two charges. If the Exercise 1.33 is an electron projected with
test charge is displaced slightly on either velocity vx = 2.0 × 106 ms-1. If E between
side of the null point along this line, it will the plates separated by 0.5 cm is 9.1 ×
experience a restoring force. But if it is 102 N/C, where will the electron strike the
displaced normal to this line, the net force upper plate ?
takes it away from the null point. That is, (| e | = 1.6 × 10-19 C, me = 9.1 × 10-31 kg).
no restoring force acts in the normal
direction. But stable equilibrium demands Ans. Here y = 0.5 cm = 0.5 × 10-2 m,
restoring force in all directions, hence test v× = 2.0 × 106 ms-1, E = 9.1 × 102 NC-1, L =
charge placed at null point will not be in ?
stable equilibrium.
From the above exercise, the vertical
1.33. A particle of mass mand charge deflection of an electron is given by
(- q) enters the region between the two
charged plates initially moving along x-
axis with speed vx (like particle 1 in Fig.
1.152). The length of plate is Land a

uniform electric field E is maintained
between the plates. Show that the vertical
deflection of the particle at the far edge of
the plate is qEL2/(2m ).
Compare this motion with motion of a
projectile in gravitational field. or cm.

Ans. The motion of the charge - q in


the region of the electric field E between
.

101
MITTAL INSTITUTE

MULTIPLE CHOICE QUESTIONS

1. A body can be negatively charged by


(a) giving excess of electrons to it
(b) removing some electrons from it
(c) giving some protons to it
(d) removing some neutrons from it.
[AIIMS 1998] figure. The net electrostatic force exerted
by the eight Cs+ ions on the Cl- ion is
2. The number of electrons for one
coulomb of charge is (a) (b)
(a) 6.25 ×1018 (b) 6.25 × 1019
(c) (d) zero.
(c) 6.25 × 1021 (d) 6.25 × 1023
[AIIMS 1999] [AIIMS 2004]

3. SI unit of permittivity is 7. Three charges +4q, Q and q are


placed in a straight line of length l at
(a) C2m2N2 (b) C2m-2N-1 points at distances 0, 1/2, and l
(c) C2m2N-1 (d) C-1m2N-2 respectively. What should be Q in order to
make the net force on q to be zero ?
[CBSE 04 ; AIIMS 12]
(a) -q (b) -2 q
4. A charge Q is divided into two parts
of q and Q-q. If the coulomb repulsion (c) (d) 4q
between them when they are separated is
[AIIMS 1980]
to be maximum, the ratio of should be
8. Three charges are placed at the
(a) 2 (b) 1/2 vertices of an equilateral triangle of side a
as shown in the figure. The
(c) 4 (d) 1/4
[AIIMS 11]
5. Two charged spheres separated at
a distance d exert a force F on each other.
If they are immersed in a liquid of
dielectric constant 2, then the force (if all
conditions are same) is force experienced by the charge placed at
(a) FI2 (b) F the vertex A in a direction normal to BC is
(c) 2 F (d) 4 F (a) (b)
[AIIMS 1997 ; VMMC 04 ]
(c) zero (d)
6. In the basic CsCl crystal structure,
Cs+ and Cl- ions are arranged in a bcc [AIIMS 2003]
configuration as shown in the
9. Three charged particles are
collinear and are in equilibrium, then
(a) all the charged particles have the
same polarity
(b) the equilibrium is unstable

102
MITTAL INSTITUTE

(c) all the charged particlescannot have 13. Four point +ve charges of same
the same polarity magnitude (Q) are placed at four comers
of a rigid square frame as shown in the
(d) both (b) and (c) are correct.
figure. The plane of the frame is perpen-
[AIIMS 2014]
10. An electron of mass me, initially at
rest, moves through a certain distance in a
uniform electric field in time t1. A proton of
mass mp also initially at rest, takes time t2
to move through an equal distance in this
uniform electric field. Neglecting the effect
of gravity, the ratio t2 / t1 is nearly equal to
dicular to Z-axis. If a -ve point charge is
(a) 1 (b) √ placed at a distance z away from the
above frame, the
(c) √ (d) 1836 (a) -ve charge oscillates along the Z-
axis
[AIIMS 15] (b) it moves away from the frame
11. Four charges are arranged at the (c) it moves slowly towards the frame and
comers of a square as shown in the figure. stays in the plane of the frame
The direction of electric field at the centre
of the square is along (d) it passes through the frame only
once.
[AIIMS 2005]
14. Two identical conductors of copper
and aluminium are placed in an identical
electric field. The magnitude of induced
charge m the aluminium will be
(a) DC (b) BC (a) zero (b) grater than in
copper
(c) AB (d) AD
(c) less than in copper
[AIIMS 2009]
(d) equal to that of copper.
12. A small uncharged metallic sphere
is positioned exactly at a point midway [AIIMS 1999]
between two equal and opposite point 15. The spatial distribution of the
charges. If the sphere is slightly displaced electric field due to two charges (A, B) is
towards the positive charge and released, shown in the figure.
then [AIIMS
11]
(a) it will oscillate about its original
position.
(b) it will move further towards the positive
charge. Which one of the following statements is
(c) its electric potential energy will correct ?
decrease and kinetic energy will (a) A is +ve and B -ve and | A | > | B|
increase.
(b) A is -ve and B +ve and | A | = | B|
(d) its total energy remains constant but is
non-zero. (c) Both are + ve but A > B
(d) Both are -ve but A> B

103
MITTAL INSTITUTE

[AIIMS 2006] (a) (b)


16. The point charges Q and -2 Q are
placed some distance apart. If the electric (c)
field at the location of Q is E, then the
electric field at the location of -2 Q will be (d) zero [AIIMS
2003]
(a) – (b) – 22. Two infinitely long parallel
(c) - E (d) - 2 E conducting plates having surface charge
d nsiti s + σ nd -σ r sp tiv y r
[AIIMS 2001] separated by a small distance. The
17. An electric dipole placed in a non- medium between the plates is vacuum. If
uniform electric field experiences ε0 is the dielectric permittivity of vacuum,
then the electric field in the region
(a) both a torque and a net force between the plates is
(b) only a force but no torque (a) 0Vm-1 (b) Vm-1
(c) only a torque but no net force
(c) Vm-1 (d) Vm-1
(d) no torque and no net force.
[AIIMS 2003] [AIIMS 2005]
18. Let Ea be the electric field due to a 23. Two parallel large thin metal
dipole in its axial plane distant l and let Eq sheets have equal surface charge
be the field in the equatorial plane distant densities of 2.56 × 10-11 Cm-3 of opposite
l. The relation between Ea and Eq is signs. The electric field between these
sheets is
(a) Ea = Eq (b) Ea = 2 Eq
(a) 1.5 NC-1 (b) 1.5 × 10-10 NC-1
(c) Eq = 2 Ea (d) Ea = 3Eq
(c) 3 NC-1 (d) 3×10-10NC-1
[AIIMS 2000]
[AIIMS 2006]
19. A charge q is located at the centre
of a cube. The electric flux through any 24. The electric field due to a uniformly
face is charged sphere of radius R as a function
of the distance from its centre is
(a) (b) represented graphically by

(c) (d)

[AIIMS 2014]
20. In air the value of the total electric
flux emitted from unit positive charge is
(a) ε0 (b) (ε0)-1
( ) (4πε0)-1 (d) 4πε0
[AIIMS 12]
21. Shown below is a distribution of
charges. The flux of electric field due to
these charges through the surface S is

[AIIMS 2004]
25. Two concentric conducing thin
spherical shells A and B having radii rA
and rB(rB > rA) are charged to QA and –

104
MITTAL INSTITUTE

QB(|QB| > | QA|). The electric field along a Assertions and Reasons
line passing through the centre is
Directions :
(a)
In the following question, a statement of
assertion (A) is followed by a statement of
reason [R].
Mark the correct choice as :
(a) If both assertion and reason are true
and reason is the correct explanation
of the assertion.
(b)
(b) If both assertion and reason are true
but reason is not the correct
explanation of the assertion.
(c) If assertion is true but reason is false.
(d) If both assertion and reason are false.
26. Assertion If the bob of a simple
(c) pendulum is kept in a horizontal electric
field, its period of oscillation will remain
same.
Reason. If bob is charged and kept in
horizontal electric field, then the time
period will be decreased.
[AIIMS 12]
(d)

[AIIMS 2005]

Answers and Explanations


1. (a) A body can be negatively
charged by giving excess of electrons to it.
2. (a) ∴

6. (d) The net force exerted by any diagonally 5. (a)


opposite pair of ions on central ion is
zero. 6. (d) The net force exerted by any
diagonally opposite pair of Cs+ ions on
3. (b) central Cl- ion is zero.
∴ 7. (a)

SI units of permittivity =

4. (a)
As the net force on q is zero, so
x is constant. For maximum force, or Q = -q.

105
MITTAL INSTITUTE

8. (c) The forces on charge + Q at the upward direction. The charge -q


vertex A are shown in the figure. begins to oscillate about O along Z-axis.
14. (d) The magnitude of charge
induced in aluminium is equal to that of
copper.
15. (a) The lines of force start from
charge A and end on charge B. So A is +
ve and B is -ve.
16. (a) Electric field of -2Q at the
location of charge Q,
The resultant force on charge + Q at A
is

Electric field of Q at the location of -2 Q,
This force acts parallel to CB. Hence
the component normal to BC will be zero.
9. (d) The three charged particles 17. (a) In a non-uniform electric field,
cannot be in stable equilibrium and cannot an electric dipole experiences both a
have the same polarity. torque and a net force.

10. (b) √ 18. (b) Electric at any axial point is


twice the electric field at the same
distance along the equatorial line
For same s, ∝


∴ √ √ √ 19. (d)
11. (a) The resultant fields due to the 20. (b)
diagonally opposite charges will act as
shown in the figure. Hence the resultant 21. (d) Charges present outside the
electric field at the centre of the square is closed surface do not contribute towards
alone DC. electric flux.

22. (c)

12. (b) At the midpoint, the forces on


the sphere due to the +ve and -ve charges
are equal and opposite. So, net force = 0. 23. (c) ≃ .
When the sphere is slightly displayed
24. (b) For points inside a charged
towards the +ve charge, force due to +ve
sphere,
charge is more than that due to -ve
charge. So the sphere will move further E∝r
towards the +ve charge.
And for points outside a charged
13. (a) At the centre of the square sphere,
frame, E = 0 and V = a constant +ve
value. Tire charge -q held above the frame E∝
will be attracted towards the centre O. As
it crosses O, a force begins to act on it in Hence option (b) is correct.

106
MITTAL INSTITUTE

25.(a) Inside the inner spherical shell 26. (c) When the uncharged bob is
A, E = 0 . At r = rA, the field is constant. As placed in an electric field, its time period
QB is -ve and | QB| > | QA |, the field in the does not change. Assertion is true.
space between A and B decreases with
For the charged bob, time period
the increase in the value of x and
increases in an electric field because of
becomes constant at the surface of B.
the increase in restoring force. Reason is
Outside the shell B, the field E decreases
false.
from a negative value to zero. Hence
option (a) is correct.

1. When air is replaced by a dielectric Now the strings are rigidly clamped at half
medium of dielectric constant , the the height.
maximum force of attraction between two
charges separated by a distance
(a) decreases times
(b) remains unchanged
(c) increases times
2 The equilibrium separation between the
(c) decreases times. balls now becomes
[CBSE 1999]
(a) (√ ) (b) ( )
2. An electron is moving round the √
nucleus of a hydrogen atom in a circular (c) ( ) (d) ( )

orbit of radius r, The coulomb force
between the two is 5. Point charges + 4q, -q and +4q are
kept on the X-axis at points x = 0, x = a
(a) – ̂ (b) and x = 2a respectively.
(a) Only -q is in stable equilibrium.
(c) – (d) ̂
(b) None of the charges is in
[CBSE 2003] equilibrium.
3. Two positive ions, each carrying a (c) All the charges are in unstable
charge q, are separated by a distance d. If equilibrium.
F is the force of repulsion between the
ions, the number of electrons missing from (d) All the charges are in stable
each ion will be (e being the charge of an equilibrium. [CBSE 1992]
electron) 6. Figure gives electric lines of force
due to two charges q1 and q2 What are the
(a) (b)√ signs of the two charges ?

(c) √ (d)

[CBSE Pre 2010]


4. Two pith balls carrying equal
charges are suspended from a common
point by strings of equal length, the
equilibrium separation between them is r. (a) Both are negative

107
MITTAL INSTITUTE

(b) Both are positive 11. A semi-circular arc of radius a is


charged uniformly and the charge per unit
(c) q1 is positive but q2 is negative
length is λ. The electric field at the centre
(d) q1 is negative but q2 is positive. is
[CBSE 1994] (a) (b)
7. There is an electric field in the X-
direction. If the work done in moving a (c) (d)
charge of 0.2 C through a distance of 2 m
along a line making an angle of 60° with [CBSE 2000]
X-axis is 4 J, then what is the value of E ? 12. A point charge +q is placed at the
(a) √ NC-1 (b) 4NC-1 midpoint of a cube of side l. The electric
flux emerging from the cube is
(c) 5 NC-1 (d) 20 NC-1
(a) zero (b)
[CBSE 1995]
8. Torque acting on electric dipole of (c) (d)
dipole moment placed in uniform electric
field ⃗ is [CBSE 1993, 96]

(a) × ⃗ (b) .⃗ 13. A charge q μC is p d t the


centre of a cube of side 0.1 m. Then the
(c) × (⃗ × ) (d) ⃗ . / electric flux diverging from each face of
[CBSE 2001] this cube is

9. A point Plies on the perpendicular (a) (b)


bisector of an electric dipole of dipole
moment p. If the distance of P from the (c) (d)
dipole is r (much larger than the size of
the dipole), then the electric field at P is [CBSE 2001]
proportional to
14. A charge Q is placed at the comer
(a) p-1 and r-2 (b) p and r-2 of a cube. The electric flux through all the
(c) p2 and r-3 (d) p and r-3 six faces of the cube is

[CBSE 1998] (a) (b)


10. Three point charges + q, -2q and (c) (d)
+ q are placed at points (x = 0, y = a, z =
03 (x = 0, y = 0, z = 0) and (x = a, y =0, z [CBSE 2000]
=0) and (x =a, y =0, z =0) respectively.
The magnitude and direction of the 15. An electric charge q is placed at
electric dipole moment vector of this one of the comers of a cube of side a. The
charge assembly are electric flux on one of its faces will be

(a) √ qa along the line joining the (a) (b)


points (x = 0, y = 0, z = 0) and (x = a, y
= a, z =0). (c) (d)
(b) qa along the line joining the points [CBSE 1993]
(x = 0, y = 0, z = 0) and (x = a, y = a, z
=0). 16. A square surface of side L metres is in
the plane of the paper. A uniform electric
(c) √ qa along + x direction. ⃗ (volt m-1), also in the plane of the paper,
is limited only to the
(d) √ qa along +y direction.
[CBSE 2007]

108
MITTAL INSTITUTE

[CBSE Pre 2010]


19. A charge Q is enclosed by a
Gaussian spherical surface of radius R. If
the radius is doubled, then the outward
electric flux will (a) increase four times (b)
be reduced to half (c) remain the same (d)
be doubled
lower half of the square surface as shown
[CBSE Pre 2011]
in the figure. The electric flux (in SI units)
associated with the surface is 20. A hollow insulated conducting
sphere is giv n positiv h rg o 10 μC
(a) EL2 (b) What will be the electric field at the centre
of the sphere, if its radius is 2 m ?
(c) (d) (a) zero (b) 5μC -2

-2 -2
[CBSE 2006] (c) 20 μC (d) 32 μC
17. What is the flux through a cube of [CBSE 1998]
side ' ’ if a point charge q is at one of its
21. A hollow cylinder has a charge q
corners ?
coulomb within it. If ϕ is the electric flux in
(a) (b) unit of volt metre

(c) (d)

[AIPMT Pre 12]


18. A square surface of side L metre in associated with the curved surface B, the
the plane of the paper is placed in a flux linked with the plane surface A in unit
uniform electric field E (volt/m) of volt metre will be
(a) ( ϕ) (b)

(c) (d) ϕ

[CBSE 2007]
22. The electric field in a certain region
is acting radially outward and is given by E
ting ong th s p n t n ng θ = Ar. A charge contained in a sphere of
with the horizontal side of the square as radius ‘ ’ centered at the origin of the field,
shown in the figure. The electric flux linked will be given by
to the surface, in units of volt m, is ( ) Aε0a2 (b) 4πε0Aa3
(a) EL2 (b) EL2 cos θ ( ) ε0Aa3 (c) 4πε0Aa2
2
(c) EL sin θ (d) zero [AIPMT 15]

Answers and Explanations


1. (a) Here ̂ = is unit vector along line joining
2. (c) Charge on an electron = -e electron to the nucleus. The negative sign
shows that the force is of attraction.
Charge on nucleus of hydrogen = + e
3. (c) [∵
∴ ̂

109
MITTAL INSTITUTE

8. (a) Torque on a dipole,


or √
= × ⃗
4. (b) In first case :
9. (d) At any far away equatorial point of
an electric dipole,
i.e., E ∝ pr-3.
 Fe = mg tan
10. (a) The given charge assembly is
equivalent to two dipoles. One dipole of
charges -q and +q has dipole moment p1
along +ve X-direction and other

In second case :
F’e = mg tan θ'

dipole of charges -q and +q has dipole


moment along + ve Y-direction. The
resultant dipole moment has magnitude,

√ √ √

is directed along ⃗⃗⃗⃗⃗ , where P is



(a,a,0).
11. (c) Refer to the solution of
Example 49 on page 1.39.
12. (b) By Gauss' theorem, ϕE =

∴ 13. ϕ .

5. (c) The net force on each charge is 14. (b) When a charge Q is placed at
zero. Therefore, ah the charges are in one corner of the cube, only one-eighth of
equilibrium. If we slightly displace the the flux emerging from charge Q passes
charge -q to the right, the net force of through all the six faces of the cube.
attraction will further displace it to the right
i.e., away from its mean positive. The ϕ
equilibrium is, therefore, unstable.
6. (a) Both the charges are negatively 15. (d) When charge q is placed at one
charged because the lines of force are corner, the flux through each of the three
directed towards the two charges. faces meeting at this corner will be zero,
as ⃗ is parallel to these faces. One-eighth
7. (d) Here q = 0.2 C, s = 2 , θ = 60°, W = of the flux emerging from charge q passes
4J But W = Fs osθ = qEs osθ through the remaining three faces, so the
or E= flux through each such face is

= 20NC-1. ϕ

110
MITTAL INSTITUTE

16. (d) As electric field is parallel to the ϕ= ⃗ . = ES cos 90° = 0.


square surface, electric flux crossing this
surface will be zero. 19. (c) As the charge Q enclosed by
the Gaussian surface remains same, so
17. (b) When the charge q is placed at
one comer of the cube, only one-eighth of ϕ = q / ε0, will also remain same.
the flux emerging from the charge q 20. (a) Electric field is zero at all points
passes through the cube. inside a hollow charged conducting
sphere.
∴ ϕ
21. (a) Let the electric flux linked with
18. (d) As the field E lies in the plane surfaces A, B and C be ϕA, ϕB and ϕC
of the surface, it is perpendicular to the respectively.
normal to this surface.

111

You might also like